74
Insurance Case Digests Atty. Bathan-Basuel 2-S, 2013-2014 1 CASES: INSURANCE 1. Secs 1-9 Travellers v. CA [G.R. No. 82036. May 22, 1997] Travellers Insurance & Surety Corporation vs. Hon. Court of Appeals & Vicente Mendoza Facts: Vicente Mendoza, Jr. as heir of his mother (Feliza Vineza de Mendoza) who was killed in a vehicular accident, filed an action for damages against the erring taxicab driver (Rodrigo Dumlao), the owner (Armando Abellon) of the taxicab (Lady Love Taxi with Plate No. 438-HA Pilipinas Taxi 1980) and the alleged insurer of the vehicle which featured in the vehicular accident. The erring taxicab was allegedly covered by a third- party liability insurance policy issued by petitioner Travellers Insurance & Surety Corporation. Petitioner was included in the complaint as the compulsory insurer of the said taxicab under Certificate of Cover No. 1447785-3. The trial court rendered judgment in favor of private respondent and ordered Rodrigo Dumlao, Armando Abellon and petitioner to pay private respondent death indemnity, moral damages, exemplary damages, attorney’s fees and other litigation expenses, jointly and severally. The decision was affirmed by the CA and the subsequent MR was denied. Hence this petition. ISSUE: Whether petitioner is liable to private respondent? HELD : NO. I. The right of the person injured to sue the insurer of the party at fault (insured), depends on whether the contract of insurance is intended to benefit third persons also or on the insured. And the test applied has been this: Where the contract provides for indemnity against liability to third persons, then third persons to whom the insured is liable can sue the insurer. Where the contract is for indemnity against actual loss or payment, then third persons cannot proceed against the insurer, the contract being solely to reimburse the insured for liability actually discharged by him thru payment to third persons, said third persons’ recourse being thus limited to the insured alone.” The trial court did not distinguish between the private respondent’s cause of action against the owner and the driver of the Lady Love taxicab and his cause of action against petitioner. The former is based on torts and quasi-delicts while the latter is based on contract. Confusing these two sources of obligations as they arise from the same act of the taxicab fatally hitting private respondent’s mother, and in the face of overwhelming evidence of the reckless imprudence of the driver of the Lady Love taxicab, the trial court brushed aside its ignorance of the terms and conditions of the insurance contract and forthwith found all three - the driver of the taxicab, the owner of the taxicab, and the alleged insurer of the taxicab - jointly and severally liable for actual, moral and exemplary damages as well as attorney’s fees and litigation expenses. This is clearly a misapplication of the law by the trial court, and respondent appellate court grievously erred in not having reversed the trial court on this ground. “While it is true that where the insurance contract provides for indemnity against liability to third persons, such third persons can directly sue the insurer, however, the direct liability of the insurer under indemnity contracts against third-party liability does not mean that the insurer can be held solidarily liable with the insured and/or the other parties found at fault. The liability of the insurer is based on contract; that of the insured is based on tort.” II. At the time of the vehicular incident which resulted in the death of private respondent’s mother, during which time the Insurance Code had not yet been amended by Batas Pambansa (B.P.) Blg. 874, Section 384 provided as follows: “Any person having any claim upon the policy issued pursuant to this chapter shall, without any unnecessary delay, present to the insurance company concerned a written notice of claim setting forth the amount of his loss, and/or the nature, extent and duration of the injuries sustained as certified by a duly licensed physician. Notice of claim must be filed within six months from date of the accident, otherwise, the claim shall be deemed waived. Action or suit for recovery of damage due to loss or injury must be brought in proper cases, with the Commission or the Courts within one year from date of accident, otherwise the claimant’s right of action shall prescribe” [emphasis and underscoring supplied]. It is significant to note that the aforecited Section 384 was amended by B.P. Blg. 874 to categorically provide that “action or suit for recovery of damage due to loss or injury must be brought in proper cases, with the Commissioner or the Courts within one year from denial of the claim, otherwise the claimant’s right of action shall prescribe” [emphasis ours]. We have certainly ruled with consistency that the prescriptive period to bring suit in court under an insurance policy, begins to run from the date of the insurer’s rejection of the claim filed by the insured, the beneficiary or any person claiming under an insurance contract. This ruling is premised upon the compliance by the persons suing under an insurance contract, with the indispensable requirement of having filed the written claim mandated by Section 384 of the Insurance Code before and after its amendment. Absent such written claim filed by the person suing under an insurance contract, no cause of action accrues under such insurance contract, considering that it is the rejection of that claim that triggers the running of the one-year prescriptive period to bring suit in court, and there can be no opportunity for the insurer to even reject a claim if none has been filed in the first place, as in the instant case. WHEREFORE , the instant petition is HEREBY GRANTED.

Insurance Case Digests (1)

Embed Size (px)

DESCRIPTION

Digests

Citation preview

Page 1: Insurance Case Digests (1)

Insurance Case Digests Atty. Bathan-Basuel

2-S, 2013-2014 1

CASES: INSURANCE 1. Secs 1-9

Travellers v. CA

[G.R. No. 82036. May 22, 1997] Travellers Insurance & Surety Corporation vs. Hon. Court of Appeals & Vicente Mendoza

Facts: Vicente Mendoza, Jr. as heir of his mother (Feliza Vineza de Mendoza) who was

killed in a vehicular accident, filed an action for damages against the erring taxicab driver (Rodrigo Dumlao), the owner (Armando Abellon) of the taxicab (Lady Love Taxi with Plate No. 438-HA Pilipinas Taxi 1980) and the alleged insurer of the vehicle which featured in the vehicular accident. The erring taxicab was allegedly covered by a third-party liability insurance policy issued by petitioner Travellers Insurance & Surety Corporation. Petitioner was included in the complaint as the compulsory insurer of the said taxicab under Certificate of Cover No. 1447785-3.

The trial court rendered judgment in favor of private respondent and ordered Rodrigo Dumlao, Armando Abellon and petitioner to pay private respondent death indemnity, moral damages, exemplary damages, attorney’s fees and other litigation expenses, jointly and severally.

The decision was affirmed by the CA and the subsequent MR was denied.

Hence this petition.

ISSUE: Whether petitioner is liable to private respondent?

HELD: NO.

I. The right of the person injured to sue the insurer of the party at fault (insured), depends on whether the contract of insurance is intended to benefit third persons also or on the insured. And the test applied has been this: Where the contract provides for indemnity against liability to third persons, then third persons to whom the insured is liable can sue the insurer. Where the contract is for indemnity against actual loss or payment, then third persons cannot proceed against the insurer, the contract being solely to reimburse the insured for liability actually discharged by him thru payment to third persons, said third persons’ recourse being thus limited to the insured alone.”

The trial court did not distinguish between the private respondent’s cause of action against the owner and the driver of the Lady Love taxicab and his cause of action against petitioner. The former is based on torts and quasi-delicts while the latter is based on contract. Confusing these two sources of obligations as they arise from the same act of the taxicab fatally hitting private respondent’s mother, and in the face of overwhelming evidence of the reckless imprudence of the driver of the Lady Love taxicab, the trial court brushed aside its ignorance of the terms and conditions of

the insurance contract and forthwith found all three - the driver of the taxicab, the owner of the taxicab, and the alleged insurer of the taxicab - jointly and severally liable for actual, moral and exemplary damages as well as attorney’s fees and litigation expenses. This is clearly a misapplication of the law by the trial court, and respondent appellate court grievously erred in not having reversed the trial court on this ground.

“While it is true that where the insurance contract provides for indemnity against liability to third persons, such third persons can directly sue the insurer, however, the direct liability of the insurer under indemnity contracts against third-party liability does not mean that the insurer can be held solidarily liable with the insured and/or the other parties found at fault. The liability of the insurer is based on contract; that of the insured is based on tort.”

I I . At the time of the vehicular incident which resulted in the death of private respondent’s mother, during which time the Insurance Code had not yet been amended by Batas Pambansa (B.P.) Blg. 874, Section 384 provided as follows:

“Any person having any claim upon the policy issued pursuant to this chapter shall, without any unnecessary delay, present to the insurance company concerned a written notice of claim setting forth the amount of his loss, and/or the nature, extent and duration of the injuries sustained as certified by a duly licensed physician. Notice of claim must be filed within six months from date of the accident, otherwise, the claim shall be deemed waived. Action or suit for recovery of damage due to loss or injury must be brought in proper cases, with the Commission or the Courts within one year from date of accident, otherwise the claimant’s right of action shall prescribe” [emphasis and underscoring supplied].

It is significant to note that the aforecited Section 384 was amended by B.P. Blg. 874 to categorically provide that “action or suit for recovery of damage due to loss or injury must be brought in proper cases, with the Commissioner or the Courts within one year from denial of the claim, otherwise the claimant’s right of action shall prescribe” [emphasis ours].

We have certainly ruled with consistency that the prescriptive period to bring suit in court under an insurance policy, begins to run from the date of the insurer’s rejection of the claim filed by the insured, the beneficiary or any person claiming under an insurance contract. This ruling is premised upon the compliance by the persons suing under an insurance contract, with the indispensable requirement of having filed the written claim mandated by Section 384 of the Insurance Code before and after its amendment. Absent such written claim filed by the person suing under an insurance contract, no cause of action accrues under such insurance contract, considering that it is the rejection of that claim that triggers the running of the one-year prescriptive period to bring suit in court, and there can be no opportunity for the insurer to even reject a claim if none has been filed in the first place, as in the instant case.

WHEREFORE, the instant petition is HEREBY GRANTED.

Page 2: Insurance Case Digests (1)

Insurance Case Digests Atty. Bathan-Basuel

2-S, 2013-2014 2

Gulf Resorts v. Phil Charter

GULF RESORTS, INC. VS. PHILIPPINE CHARTER INSURANCE CORPORATION G.R. No. 156167 May 16, 2005 Puno, J.; FACTS: Plaintiff Gulf Resorts is the owner of the Plaza Resort situated at Agoo, La Union and had its properties in said resort insured originally with the American Home Assurance Company or AHAC-AIU. In the first four insurance policies issued by AHAC-AIU the risk of loss from earthquake shock was extended to only the plaintiff’s two swimming pools, thus, “earthquake shock endt.” Subsequently, AHAC-AIU issued in the plaintiff’s favor Policy number 206-4182383-0 covering the period March 14, 1988 to March 14, 1989 and in the said policy, the earthquake endorsement clause was deleted and the entry under Endorsements/Warranties at the time of issue read that plaintiff renewed its policy with AHAC-AIU for the period of March 14, 1989 to March 14, 1990 under policy number 206-4568061-9 which carried the entry under “Endorsement/Warranties at Time of Issue” which read “Endorsement to Include Earthquake Shock” in the amount of P10,700 and paid P42,658.14 as premium thereof.

The plaintiff agreed to insure with defendant the properties covered by AHAC-AIU Policy number 206-4568061-9 provided that the policy wording and rates in the said policy be copied in the policy to be issued by defendant. The defendant then issued Policy number 31944 to plaintiff covering the period of March 14, 1990 to March 14, 1991 for P10,700,600 for a total premium of P45,159.92. In Policy number 31944 issued by defendant, the shock endorsement provide:

“In consideration of the payment by the insured to the company of the sum included additional premium the Company agrees, notwithstanding what is stated in the printed conditions of this policy due to the contrary, that this insurance covers loss or damage to shock to any of the property insured by this Policy occasioned by or through or in consequence of earthquake.”

That in the said policy, the word “included” was deleted. On July 16, 1990 an earthquake struck Central Luzon and Northern Luzon and the plaintiff’s properties covered by Policy No. 31944 issued by defendant, including the two swimming pools in its Agoo Playa Resort were damaged. After the earthquake, petitioner advised respondent that it would be making a claim under its Insurance Policy No. 31944 for damages on its properties. Respondent instructed petitioner to file a formal claim, then assigned the investigation of the claim to an independent claims adjuster. On July 30, 1990, respondent through its adjuster, requested petitioner to submit various documents in support of its claim. Subsequently, the adjuster rendered a decision saying “except for the swimming pools, all affected items have no coverage for earthquake shocks.” Petitioner then filed its formal demand for settlement for the damage to all its properties in the Agoo Playa Resort.

The regional trial court ruled in favor of respondent stating that only the 2 swimming pools had earthquake shock coverage. Upon appeal, the appellate court affirmed the decision of the trial court. ISSUE: Whether or not the other properties in the Agoo Playa Resort are included in the earthquake shock coverage of the said insurance policy HELD: It is basic that all the provisions of the insurance policy should be examined and interpreted in consonance with each other. All its parts are reflective of the true intent of the parties. The policy cannot be construed piecemeal. Petitioner cannot focus on the earthquake shock endorsement to the exclusion of the other provisions. All the provisions and riders, taken and interpreted together, indubitably show the intention of the parties to extend earthquake shock coverage to the two swimming pools only. A careful examination of the premium recapitulation will show that it is the clear intent of the parties to extend earthquake coverage shock only to the 2 swimming pools. Section 2(1) of the Insurance Code defines a contract of insurance as an agreement whereby one undertakes for a consideration to indemnify another against loss, damage or liability arising from an unknown or contingent event. Thus, an insurance contract exists where the following elements concur:

1. the insured has an insurable interest 2. the insured is subject to a risk of loss by the happening of the designated

peril 3. the insurer assumes the risk 4. such assumption of risk is part of a general scheme to distribute actual

losses among a large group of persons bearing similar risk 5. in consideration of the insurer’s promise, the insured pays a premium

an insurance premium is the consideration paid an insurer for undertaking to indemnify the insured against a specified peril. In the subject policy, no premium payments were made with regard to earthquake shock coverage, except on the 2 swimming pools. There is no mention of any premium payable for the other resort properties with regard to earthquake shock. This is consistent with the history of petitioner’s previous insurance policies from AHAC-AIU. Hence, the judgment of the Court of Appeals is affirmed.

New World v. NYK Fil-Japan

FACTS: Petitioner New World International Development (Phils.), Inc. (New World) bought from DMT Corporation (DMT) through its agent, Advatech Industries, Inc. (Advatech) three emergency generator sets worth US$721,500.00.

Page 3: Insurance Case Digests (1)

Insurance Case Digests Atty. Bathan-Basuel

2-S, 2013-2014 3

DMT shipped the generator sets by truck from Wisconsin, United States, to LEP Profit International, Inc. (LEP Profit) in Chicago, Illinois. From there, the shipment went by train to Oakland, California, where it was loaded on S/S California Luna V59, owned and operated by NYK Fil-Japan Shipping Corporation (NYK) for delivery to petitioner New World in Manila. NYK issued a bil l of lading, declaring that it received the goods in good condition. NYK unloaded the shipment in Hong Kong and transshipped it to S/S ACX Ruby V/72 that it also owned and operated.

On its journey to Manila, however, ACX Ruby encountered typhoon Kadiang whose captain filed a sea protest on arrival at the Manila South Harbor on October 5, 1993 respecting the loss and damage that the goods on board his vessel suffered. Marina Port Services, Inc. (Marina), the Manila South Harbor arrastre or cargo-handling operator, received the shipment on October 7, 1993. Upon inspection of the three container vans separately carrying the generator sets, two vans bore signs of external damage while the third van appeared unscathed. An examination of the three generator sets in the presence of petitioner New World’s representatives, Federal Builders (the project contractor) and surveyors of petitioner New World’s insurer, Seaboard–Eastern Insurance Company (Seaboard), revealed that all three sets suffered extensive damage and could no longer be repaired.

New World demanded recompense for its loss from respondents NYK, DMT, Advatech, LEP Profit, LEP International Philippines, Inc. (LEP), Marina, and Serbros. While LEP and NYK acknowledged receipt of the demand, both denied l iabil ity for the loss. Since Seaboard covered the goods with a marine insurance policy, New World sent it a formal claim dated November 16, 1993. Replying on February 14, 1994, Seaboard required petitioner New World to submit to it an itemized list of the damaged units, parts, and accessories, with corresponding values, for the processing of the claim. But petitioner New World did not submit what was required of it, insisting that the insurance policy did not include the submission of such a l ist in connection with an insurance claim. Reacting to this, Seaboard refused to process the claim.

On October 11, 1994 New World filed an action for specific performance and damages against all the respondents before the Regional Trial Court (RTC) of Makati City. ISSUE: Whether CA erred in ruling that Seaboard’s request from petitioner New World for an itemized list is a reasonable imposition and did not violate the insurance contract between them; and

HELD: YES. Itemized listing is not substantially necessary.

The record shows that petitioner New World complied with the documentary requirements evidencing damage to its generator sets.

The marine open policy that Seaboard issued to New World was an all-risk

policy. Such a policy insured against all causes of conceivable loss or damage except when otherwise excluded or when the loss or damage was due to fraud or intentional misconduct committed by the insured. The policy covered all losses during the voyage whether or not arising from a marine peril.

The policy enumerated certain exceptions like unsuitable packaging, inherent

vice, delay in voyage, or vessels unseaworthiness, among others. But Seaboard had been unable to show that petitioner New World’s loss or damage fell within some or one of the enumerated exceptions.

Moreover, New World has submitted various documents, namely: (1) copy of the Supplier’s Invoice; (2) copy of the Packing List; (3) copy of the Bill of Lading; (4) the Delivery of Waybill Receipts 1135, 1222, and 1224; (5) original copy of Marine Insurance Policy MA-HO-000266; (6) copies of Damage Report from Supplier and Insurance Adjusters; (7) Consumption Report from the Customs Examiner; and (8) Copies of Received Formal Claim from the following:

a) LEP International Philippines, Inc.; b) Marina Port Services, Inc.; and c) Serbros Carrier Corporation.

Seaboard cannot pretend that the above documents are inadequate since they

were precisely the documents listed in its insurance policy. Being a contract of adhesion, an insurance policy is construed strongly against the insurer who prepared it. The Court cannot read a requirement in the policy that was not there.

White Gold v. Pioneer

WHITE GOLD MARINE SERVICES, INC., petitioner vs. PIONEER INSURANCE AND SURETY CORPORATION and THE STEAMSHIP MUTUAL UNDERWRITING ASSOCIATION (BERMUDA) LTD., respondents Ponente: QUISUMBING FACTS: White Gold procured a protection and indemnity coverage for its vessels from The Steamship Mutual through Pioneer. It was issued a Certificate of Entry and Acceptance. Pioneer also issued receipts evidencing payments. When White Gold failed to fully pay its accounts, Steamship Mutual refused to renew the coverage.

Page 4: Insurance Case Digests (1)

Insurance Case Digests Atty. Bathan-Basuel

2-S, 2013-2014 4

Steamship Mutual filed a case against White Gold for collection of sum of money. White Gold filed a complaint before the Insurance Commission claiming that Steamship Mutual violated Sections 186 and 187 of the Insurance Code, while Pioneer violated Sections 299-301 in relation to Sections 302-303. It was dismissed, ruling that there was no need for Steamship Mutual to secure a license because it was not engaged in the insurance business, as a Protection and Indemnity Club (P & I Club). Pioneer also need not obtain another license as insurance agent and/or a broker for Steamship Mutual. Moreover, Pioneer was already licensed. CA affirmed. Hence, the petition. A P & I Club is “an association composed of ship owners in general who band together for the specific purpose of providing insurance cover on a mutual basis against liabilities incidental to ship owning that the members incur in favor of third parties.” As a P & I Club, Steamship Mutual’s primary purpose is to solicit and provide protection and indemnity coverage and for this purpose, it has engaged the services of Pioneer to act as its agent. ISSUE: Whether Steamship Mutual, a P & I Club, is engaged in the insurance business in the Philippines. YES Whether Pioneer needs a license as an insurance agent/broker for Steamship Mutual. YES HELD: Section 2 (2) of the Insurance Code enumerates what constitutes “doing an insurance business” or “transacting an insurance business”. The test to determine if a contract is an insurance contract or not, depends on the nature of the promise, the act required to be performed, and the exact nature of the agreement in the light of the occurrence, contingency, or circumstances under which the performance becomes requisite. It is not by what it is called. Basically, an insurance contract is a contract of indemnity. In it, one undertakes for a consideration to indemnify another against loss, damage or liability arising from an unknown or contingent event. In particular, a marine insurance undertakes to indemnify the assured against marine losses, such as the losses incident to a marine adventure. A mutual insurance company is a cooperative enterprise where the members are both the insurer and insured. Additionally, mutual insurance associations, or clubs, provide three types of coverage, namely, protection and indemnity, war risks, and defense costs. A P & I Club is “a form of insurance against third party liability, where the third party is anyone other than the P & I Club and the members.” By definition then, Steamship Mutual as a P & I Club is a mutual insurance association engaged in the marine insurance business. Thus, to continue doing business here, Steamship Mutual or through its agent Pioneer, must secure a license from the Insurance Commission. Since a contract of insurance

involves public interest, regulation by the State is necessary. Thus, no insurer or insurance company is allowed to engage in the insurance business without a license or a certificate of authority from the Insurance Commission. Although Pioneer is already licensed as an insurance company, it needs a separate license to act as insurance agent for Steamship Mutual. CA decision REVERSED AND SET ASIDE. Steamship Mutual and Pioneer are ORDERED to obtain licenses.

RP v. Del Monte Republic of the Phil ippines, (represented by Eduardo Malinis in his capacity as Insurance Commisioner) vs. Del Monte Motors Inc. G.R. No, . 156956 October 9, 2006 C.J. Panganiban Facts:

On January 15, 2002, the RTC rendered a decision in a civil case finding Vilfran Liner, Hilaria Villegas and Maura Villegas jointly and severally liable to pay Del Monte Motors lost P11,835,375.50 for service contracts with Del Monte. The Trial Court further ordered the execution of the decision against the counterbond posted by Vilfran and issued by Capital Insurance and Surety Co. Inc. (CISCO).

CISCO opposed the Motion for Execution claiming that they had no record or document regarding the alleged issuance of the counterbond, theus the bond was not valid and enforceable. However, the RTC released a motion for execution commanding the sheriff to levy the amount on the property of CISCO. To completely satisfy the amount, the Insurance Commissioner was also commanded to withdraw the security deposit filed by CISCO with the Commission according to Sec 203 of the Insurance Code.

Insurance Commissioner Malinis was ordered by the RTC to withdraw the security bond of CISCO for the payment of the insurance indemnity won by Del Monte Motor against Vilfran Liner, the insured. Malinis didn’t obey the order, so the respondent moved to cite him in contempt of Court. The RTC ruled against Malinis. It explained that the commissioner had no legal justification for his refusal to allow the withdrawal of CISCO’s security deposit. Hence, this petition. Issues: 1. Whether or not the security deposit held by the Insurance Commissioner pursuant to Section 203 of the Insurance Code may be levied or garnished in favor of only one insured. 2. Whether or not the Insurance Commissioner has power to withhold the release of the security deposit. Held: 1. NO.

Sec 203 of the Insurance Code provides, “no judgment creditor or other claimant shall have the right to levy upon any of the securities of the insurer held on

Page 5: Insurance Case Digests (1)

Insurance Case Digests Atty. Bathan-Basuel

2-S, 2013-2014 5

deposit pursuant to the requirement of the Commissioner.” The court also claimed that the security deposit shall be (1) answerable for all the obligations of the depositing insurer under its insurance contracts; (2) at all times free from any liens or encumbrance; and (3) exempt from levy by any claimant.

“To allow the garnishment of that deposit would impair the fund by decreasing it to less than the percentage of paid-up capital that the law requires to be maintained. Further, this move would create, in favor of respondent, a preference of credit over the other policy holders and beneficiaries.”

“Also, the securities are held as a contingency fund to answer for the claims against the insurance company by all its policy holders and their beneficiaries. This step is taken in the event that the company becomes insolvent or otherwise unable to satisfy the claims against it. Thus, a single claimant may not lay stake on the securities to the exclusion of all others. The other parties may have their own claims against the insurance company under other insurance contracts it has entered into.” 2. YES.

The Insurance Code has vested the Office of the Insurance Commission with both regulatory and adjudicatory authority over insurance matters. Under Sec 414 of the Insurance Code, "The Commissioner may issue such rulings, instructions, circulars, orders and decisions as he may deem necessary to secure the enforcement of the provisions of this Code.”

“The commissioner is authorized to (1) issue (or to refuse to issue) certificates of authority to persons or entities desiring to engage in insurance business in the Philippines;16 (2) revoke or suspend these certificates of authority upon finding grounds for the revocation or suspension; (3) impose upon insurance companies, their directors and/or officers and/or agents appropriate penalties -- fines, suspension or removal from office -- for failing to comply with the Code or with any of the commissioner's orders, instructions, regulations or rulings, or for otherwise conducting business in an unsafe or unsound manner.”

Included here is the duty to hold security deposits under Secs 191 and 202 of the Code for the benefit of policy holders. Sec 192, on the other hand, states: “the securities deposited as aforesaid shall be returned upon the company's making application therefor and proving to the satisfaction of the Commissioner that it has no further liability under any of its policies in the Philippines.”

He has been given great discretion to regulate the business to protect the public. Also “An implied trust is created by the law for the benefit of all claimants under subsisting insurance contracts issued by the insurance company.” He believed that the security deposit was exempt from execution to protect the policy holders.

Philhealth v. CIR PHILHEALTH CARE PROVIDERS vs. CIR G.R. No. 167330 September 18, 2009 PHILIPPINE HEALTH CARE PROVIDERS, INC., Petitioner,

COMMISSIONER OF INTERNAL REVENUE, Respondent. CORONA, J.: FACTS: On January 27, 2000, respondent Commissioner of Internal Revenue [CIR] sent Philhealth a formal demand letter and the corresponding assessment notices demanding the payment of deficiency taxes for the taxable years 1996 and 1997 in the total amount of P224,702,641.18. Philhealth protested the assessment and, as CIR did not act on the protest, Philhealth filed a petition for review in the Court of Tax Appeals (CTA) seeking the cancellation of the deficiency VAT and DST (documentary stamp tax) assessments. CTA rendered a decision ordering the Philhealth to pay the deficiency VAT for 1996 and 1997 and canceling the 1996 and 1997 deficiency DST assessment against it. CIR appealed the CTA decision to CA insofar as it cancelled the DST assessment claiming that petitioner’s health care agreement was a contract of insurance subject to DST under Section 185 of the 1997 Tax Code. CA reversed the decision of CTA and upheld that the petitioner’s health care agreement was in the nature of non-life insurance contract subject to DST. ISSUES:

1. WON the respondent is a Health Maintenance Organization (HMO) 2. WON petitioner, as an HMO, engaged in the business of insurance during the

pertinent taxable years 3. WON a health care agreement is an insurance contract.

HELD: 1. Philhealth is admittedly an HMO. Under RA 7875 (or "The National Health Insurance Act of 1995"), an HMO is “an entity that provides, offers or arranges for coverage of designated health services needed by plan members for a fixed prepaid premium.” 2. NO, Philhealth is not engaged in the business of insurance. Pursuant to the “principal object and purpose test” adopted by the Supreme Court, the primary purpose of a medical service corporation, such as Philhealth, is NOT the assumption of risk and indemnification, but rather to provide physicians who will render services to subscribers on a prepaid basis. In fact, a substantial portion of petitioner’s services covers preventive and diagnostic medical services intended to keep members from developing medical conditions or diseases. As an HMO, it is its obligation to maintain the good health of its members. Accordingly, its health care programs are designed to prevent or to minimize the possibility of any assumption of risk on its part. Thus, its undertaking under its agreements is not to indemnify its members against any loss or damage arising from a medical condition but, on the contrary, to provide the health and medical services needed to prevent such loss or damage. Petitioner, as an HMO, is not part of

Page 6: Insurance Case Digests (1)

Insurance Case Digests Atty. Bathan-Basuel

2-S, 2013-2014 6

the insurance industry. This is evident from the fact that it is not supervised by the Insurance Commission but by the Department of Health. 3. NO, a healthcare agreement is not an insurance contract. The agreements between the petitioner and its members do not possess all the elements of an insurance contract provided in Sec. 2(1) of the Insurance Code because the healthcare's primary purpose is to render service and not to assume risk and indemnify another. Also, there is nothing in petitioner's agreements that gives rise to a monetary liability on the part of the member to any third party-provider of medical services which might in turn necessitate indemnification from petitioner. The terms "indemnify" or "indemnity" presuppose that a liability or claim has already been incurred. There is no indemnity precisely because the member merely avails of medical services to be paid or already paid in advance at a pre-agreed price under the agreements. NOTE:

• “Principal object and purpose test” of United State jurisprudence, to wit: whether the assumption of risk and indemnification of loss (which are elements of an insurance business) are the principal object and purpose of the organization or whether they are merely incidental to its business. If these are the principal objectives, the business is that of insurance. But if they are merely incidental and service is the principal purpose, then the business is not insurance.

Philamcare v. CA

Philamcare v CA G.R. No. 125678. March 18, 2002 J. Ynares-Santiago Facts: Ernani Trinos applied for a health care coverage with Philam. He answered no to a question asking if he or his family members were treated to heart trouble, asthma, diabetes, etc. The application was approved for 1 year. He was also given hospitalization benefits and out-patient benefits. After the period expired, he was given an expanded coverage for Php 75,000. During the period, he suffered from heart attack and was confined at MMC. The wife tried to claim the benefits but the petitioner denied it saying that he concealed his medical history by answering no to the aforementioned question. She had to pay for the hospital bills amounting to 76,000. Her husband subsequently passed away. She filed a case in the trial court for the collection of the amount plus damages. She was awarded 76,000 for the bills and 40,000 for damages. The CA affirmed but deleted awards for damages. Hence, this appeal. Issue: WON a health care agreement is not an insurance contract; hence the “incontestability clause” under the Insurance Code does not apply.

Held: No. Petition dismissed. Ratio: Petitioner claimed that it granted benefits only when the insured is alive during the one-year duration. It contended that there was no indemnification unlike in insurance contracts. It supported this claim by saying that it is a health maintenance organization covered by the DOH and not the Insurance Commission. Lastly, it claimed that the Incontestability clause didn’t apply because two-year and not one-year effectivity periods were required. Section 2 (1) of the Insurance Code defines a contract of insurance as “an agreement whereby one undertakes for a consideration to indemnify another against loss, damage or liability arising from an unknown or contingent event.” Section 3 states: every person has an insurable interest in the life and health: (1) of himself, of his spouse and of his children. In this case, the husband’s health was the insurable interest. The health care agreement was in the nature of non-life insurance, which is primarily a contract of indemnity. The provider must pay for the medical expenses resulting from sickness or injury. While petitioner contended that the husband concealed material fact of his sickness, the contract stated that: “that any physician is, by these presents, expressly authorized to disclose or give testimony at anytime relative to any information acquired by him in his professional capacity upon any question affecting the eligibility for health care coverage of the Proposed Members.” This meant that the petitioners required him to sign authorization to furnish reports about his medical condition. The contract also authorized Philam to inquire directly to his medical history. Hence, the contention of concealment isn’t valid. They can’t also invoke the “Invalidation of agreement” clause where failure of the insured to disclose information was a grounds for revocation simply because the answer assailed by the company was the heart condition question based on the insured’s opinion. He wasn’t a medical doctor, so he can’t accurately gauge his condition. Henrick v Fire - “in such case the insurer is not justified in relying upon such statement, but is obligated to make further inquiry.” Fraudulent intent must be proven to rescind the contract. This was incumbent upon the provider. “Having assumed a responsibility under the agreement, petitioner is bound to answer the same to the extent agreed upon. In the end, the liability of the health care provider attaches once the member is hospitalized for the disease or injury covered by the agreement or whenever he avails of the covered benefits which he has prepaid.” Section 27 of the Insurance Code- “a concealment entitles the injured party to rescind a contract of insurance.” As to cancellation procedure- Cancellation requires certain conditions:

1. Prior notice of cancellation to insured;

Page 7: Insurance Case Digests (1)

Insurance Case Digests Atty. Bathan-Basuel

2-S, 2013-2014 7

2. Notice must be based on the occurrence after effective date of the policy of one or more of the grounds mentioned;

3. Must be in writing, mailed or delivered to the insured at the address shown in the policy;

4. Must state the grounds relied upon provided in Section 64 of the Insurance Code and upon request of insured, to furnish facts on which cancellation is based

None were fulfilled by the provider. As to incontestability- The trial court said that “under the title Claim procedures of expenses, the defendant Philamcare Health Systems Inc. had twelve months from the date of issuance of the Agreement within which to contest the membership of the patient if he had previous ailment of asthma, and six months from the issuance of the agreement if the patient was sick of diabetes or hypertension. The periods having expired, the defense of concealment or misrepresentation no longer lie.”

Eternal v. Philamlife ETERNAL VS. PHILAMLIFE

G.R. No. 166245

April 09, 2008

FACTS: Respondent Philamlife entered into an agreement denominated as Creditor Group Life Policy with petitioner Eternal Gardens Memorial Park Corporation (Eternal). Under the policy, the clients of Eternal who purchased burial lots from it on installment basis would be insured by Philamlife. The amount of insurance coverage depended upon the existing balance of the purchased burial lots.

The relevant provisions of the policy are:

ELIGIBILITY.

xx EVIDENCE OF INSURABILITY. xx LIFE INSURANCE BENEFIT. xx

EFFECTIVE DATE OF BENEFIT.

The insurance of any eligible Lot Purchaser shall be effective on the date he contracts a loan with the Assured. However, there shall be no insurance if the application of the Lot Purchaser is not approved by the Company.

xx Eternal was required under the policy to submit to Philamlife a list of all new lot

purchasers, together with a copy of the application of each purchaser, and the amounts of the respective unpaid balances of all insured lot purchasers. Eternal complied by submitting a letter dated December 29, 1982, containing a list of insurable balances of its lot buyers for October 1982. One of those included in the list as “new business” was a certain John Chuang. His balance of payments was 100K. on August 2, 1984, Chuang died.

Eternal sent a letter dated to Philamlife, which served as an insurance claim for Chuang’s death. Attached to the claim were certain documents. In reply, Philamlife wrote Eternal a letter requiring Eternal to submit the additional documents relative to its insurance claim for Chuang’s death. Eternal transmitted the required documents through a letter which was received by Philamlife.

After more than a year, Philamlife had not furnished Eternal with any reply to the latter’s insurance claim. This prompted Eternal to demand from Philamlife the payment of the claim for PhP 100,000. In response to Eternal’s demand, Philamlife denied Eternal’s insurance claim in a letter a portion of which reads:

The deceased was 59 years old when he entered into Contract #9558 and 9529 with Eternal Gardens Memorial Park in October 1982 for the total maximum insurable amount of P100,000.00 each. No application for Group Insurance was submitted in our office prior to his death on August 2, 1984

Eternal filed a case with the RTC for a sum of money against Philamlife, which decided in favor of Eternal, ordering Philamlife to pay the former 100K representing the proceeds of the policy.

CA reversed. Hence this petition.

ISSUE: WON Philamlife should pay the 100K insurance proceeds

HELD: Petition granted.

YES. An examination of the provision of the POLICY under effective date of benefit, would show ambiguity between its two sentences. The first sentence appears to state that the insurance coverage of the clients of Eternal already became effective upon contracting a loan with Eternal while the second sentence appears to require Philamlife to approve the insurance contract before the same can become effective.

It must be remembered that an insurance contract is a contract of adhesion which must be construed liberally in favor of the insured and strictly against the insurer in order to safeguard the latter’s interest

Page 8: Insurance Case Digests (1)

Insurance Case Digests Atty. Bathan-Basuel

2-S, 2013-2014 8

On the other hand, the seemingly conflicting provisions must be harmonized to mean that upon a party’s purchase of a memorial lot on installment from Eternal, an insurance contract covering the lot purchaser is created and the same is effective, valid, and binding until terminated by Philamlife by disapproving the insurance application. The second sentence of the Creditor Group Life Policy on the Effective Date of Benefit is in the nature of a resolutory condition which would lead to the cessation of the insurance contract. Moreover, the mere inaction of the insurer on the insurance application must not work to prejudice the insured; it cannot be interpreted as a termination of the insurance contract. The termination of the insurance contract by the insurer must be explicit and unambiguous.

Filipinas v. Christern

FILIPINAS COMPAÑIA DE SEGUROS, petitioner, vs. CHRISTERN, HUENEFELD and CO., INC., respondent.

G.R. No. L-2294 May 25, 1951

FACTS:

On October 1, 1941, the respondent corporation, Christern Huenefeld, & Co., Inc., after payment of corresponding premium, obtained from the petitioner ,Filipinas Cia. de Seguros, fire policy No. 29333 in the sum of P1000,000, covering merchandise contained in a building located at No. 711 Roman Street, Binondo Manila. On February 27, 1942, or during the Japanese military occupation, the building and insured merchandise were burned.

Respondent then claimed under its policy with the petitioner. The salvage goods were sold at public auction and, after deducting their value, the total loss suffered by the respondent was fixed at P92,650. The petitioner refused to pay the claim on the ground that the policy in favor of the respondent had ceased to be in force on the date the United States declared war against Germany, the respondent Corporation (though organized under and by virtue of the laws of the Philippines) being controlled by the German subjects and the petitioner being a company under American jurisdiction when said policy was issued on October 1, 1941. The petitioner, however, in pursuance of the order of the Director of Bureau of Financing, Philippine Executive Commission, dated April 9, 1943, paid to the respondent the sum of P92,650 on April 19, 1943.

After trial, the Court of First Instance of Manila dismissed the action filed by the petitioner to recover the amount from respondent. Upon appeal to the Court of Appeals, the judgment of the Court of First Instance of Manila was affirmed. Hence this petition.

ISSUE: Whether or not Filipinas Cia de Seguros can claim the amount it paid against respondent.

HELD:

1) In case of war, the control test applies to determine the nationality of a corporation.

The Court of Appeals overruled the contention of the petitioner that the respondent corporation became an enemy when the United States declared war against Germany, relying on English and American cases which held that a corporation is a citizen of the country or state by and under the laws of which it was created or organized. It rejected the theory that nationality of private corporation is determine by the character or citizenship of its controlling stockholders.

There is no question that majority of the stockholders of the respondent corporation were German subjects. This being so, we have to rule that said respondent became an enemy corporation upon the outbreak of the war between the United States and Germany. The English and American cases relied upon by the Court of Appeals have lost their force in view of the latest decision of the Supreme Court of the United States in Clark vs. Uebersee Finanz Korporation, decided on December 8, 1947, 92 Law. Ed. Advance Opinions, No. 4, pp. 148-153, in which the controls test has been adopted. This pronouncement was based on the situation that arise during the WWI and WWII.

2) The Phil ippine Insurance Law prohibits the granting of insurance against a public enemy.

The Philippine Insurance Law (Act No. 2427, as amended,) in section 8, provides that "anyone except a public enemy may be insured." It stands to reason that an insurance policy ceases to be allowable as soon as an insured becomes a public enemy.

Effect of war, generally. — All intercourse between citizens of belligerent powers which is inconsistent with a state of war is prohibited by the law of nations. Such prohibition includes all negotiations, commerce, or trading with the enemy; all acts which will increase, or tend to increase, its income or resources; all acts of voluntary submission to it; or receiving its protection; also all acts concerning the transmission of money or goods; and all contracts relating thereto are thereby nullified. It further prohibits insurance upon trade with or by the enemy, upon the life or lives of aliens engaged in service with the enemy; this for the reason that the subjects of one country cannot be permitted to lend their assistance to protect by insurance the commerce or property of belligerent, alien subjects, or to do anything detrimental too their country's interest. The purpose of war is to cripple the power and exhaust the resources of the enemy, and it is inconsistent that one country should destroy its enemy's property and repay in insurance the value of what has been so

Page 9: Insurance Case Digests (1)

Insurance Case Digests Atty. Bathan-Basuel

2-S, 2013-2014 9

destroyed, or that it should in such manner increase the resources of the enemy, or render it aid, and the commencement of war determines, for like reasons, all trading intercourse with the enemy, which prior thereto may have been lawful. All individuals therefore, who compose the belligerent powers, exist, as to each other, in a state of utter exclusion, and are public enemies. (6 Couch, Cyc. of Ins. Law, pp. 5352-5353.)

In the case of an ordinary fire policy, which grants insurance only from year, or for some other specified term it is plain that when the parties become alien enemies, the contractual tie is broken and the contractual rights of the parties, so far as not vested. lost. (Vance, the Law on Insurance, Sec. 44, p. 112.)

The respondent having become an enemy corporation on December 10, 1941, the insurance policy issued in its favor on October 1, 1941, by the petitioner (a Philippine corporation) had ceased to be valid and enforcible, and since the insured goods were burned after December 10, 1941, and during the war, the respondent was not entitled to any indemnity under said policy from the petitioner. However, elementary rules of justice (in the absence of specific provision in the Insurance Law) require that the premium paid by the respondent for the period covered by its policy from December 11, 1941, should be returned by the petitioner.

Constantino v. Asia Life Facts:

- Appeal consolidates two cases. - Asia life insurance Company (ALIC) was incorporated in Delaware. - For the sum of 175.04 as annual premium duly paid to ALIC, it issued Policy

No. 93912 whereby it insured the life of Arcadio Constantino for 20 years for P3T with Paz Constantino as beneficiary.

- First premium covered the period up to Sept. 26, 1942. No further premiums were paid after the first premium and Arcadio died on Sept. 22, 1944.

- Due to Jap occupation, ALIC closed its branch office in Manila from Jan. 2 1942-1945.

- On Aug. 1, 1938, ALIC issued Policy no. 78145 covering the lives of Spouses Tomas Ruiz and Agustina Peralta for the sum of P3T for 20 years. The annual premium stipulated was regularly paid from Aug. 1, 1938 up to and including Sept. 30, 1940.

- Effective Aug. 1, 1941, the mode of payment was changed from annually to quarterly and such quarterly premiums were paid until Nov. 18, 1941.

- Last payment covered the period until Jan. 31, 1942. - Tomas Ruiz died on Feb. 16, 1945 with Agustina Peralta as his beneficiary. - Due to Jap occupation, it became impossible and illegal for the insured to deal

with ALIC. Aside from this the insured borrowed from the policy P234.00 such that the cash surrender value of the policy was sufficient to maintain the policy in force only up to Sept. 7, 1942.

- Both policies contained this provision: All premiums are due in advance and any unpunctuality in making such payment shall cause this policy to lapse unless and except as kept in force by the grace period condition.

- Paz Constantino and Agustina Peralta claim as beneficiaries, that they are entitled to receive the proceeds of the policies less all sums due for premiums in arrears. They also allege that non-payment of the premiums were caused by the closing of ALIC’s offices during the war and the impossible circumstances by the war, therefore, they should be excused and the policies should not be forfeited.

- Lower court ruled in favor of ALIC. Issue: May a beneficiary in a life insurance policy recover the amount thereof although the insured died after repeatedly failing to pay the stipulated premiums, such failure being caused by war? Held: NO. Due to the express terms of the policy, non-payment of the premium produces its avoidance. In Glaraga v. Sun Life, it was held that a life policy was avoided because the premium had not been paid within the time fixed; since by its express terms, non-payment of any premium when due or within the 31 day grace period ipso fact caused the policy to lapse. When the life insurance policy provides that non-payment of premiums will cause its forfeiture, war does NOT excuse non-payment and does not avoid forfeiture. Essentially, the reason why punctual payments are important is that the insurer calculates on the basis of the prompt payments. Otherwise, malulugi sila. It should be noted that the parties contracted not only as to peace time conditions but also as to war-time conditions since the policies contained provisions applicable expressly to wartime days. The logical inference therefore is that the parties contemplated the uninterrupted operation of the contract even if armed conflict should ensue.

Great Pacific v. CA FACTS:

Great Pacific Life Assurance (Grepalife) assailed the decision of the CA, which affirmed the decision of the RTC (Misamis Oriental) when it ruled that Grepalife should pay Development Bank of the Philippines (DBP) as creditor of the insured Dr. Wilfredo Leuterio (Php 86,200.00).

A contract of GROUP LIFE INSURANCE was executed between Grepalife and DBP. Grepalife agreed to insure the lives of ELIGIBLE HOUSING LOAN MORTGAGORS of DBP.

Page 10: Insurance Case Digests (1)

Insurance Case Digests Atty. Bathan-Basuel

2-S, 2013-2014 10

Dr. Wilfredo Leutorio applied for membership and answered the question concerning his health conditions:

“7. Have you ever had, or consulted, a physician for a heart condition, high blood pressure, cancer, diabetes, lung, kidney or stomach disorder or any other physical impairment?

Answer: No. If so give details ___________.

8. Are you now, to the best of your knowledge, in good health?

Answer: [ x ] Yes [ ] No.”

Grepalife issued certificate insurance coverage of Dr. Leuterio, to the extent of his DBP mortgage indebtedness amounting to P86, 200.00 pesos. Dr. Leuterio DIED due to “massive cerebral hemorrhage.”

DBP submitted a death claim to Grepalife. Grepalife denied the claim alleging that Dr. Leuterio was not physically healthy when he applied for an insurance coverage on November 15, 1983. Grepalife insisted that Dr. Leuterio did not disclose he had been suffering from hypertension, which caused his death. Allegedly, such non-disclosure constituted concealment that justified the denial of the claim.

The widow of the late Dr. Leuterio, respondent Medarda V. Leuterio, filed a complaint with the Regional Trial Court of Misamis Oriental, Branch 18, against Grepalife for “Specific Performance with Damages.”

During the trial, Dr. Hernando Mejia, who issued the death certificate, was called to testify. Dr. Mejia’s findings, based partly from the information given by the respondent widow, stated that Dr. Leuterio complained of headaches presumably due to high blood pressure. The inference was not conclusive because Dr. Leuterio was not autopsied, hence, other causes were not ruled out.

Trial court rendered a decision in favor of respondent widow and against Grepalife. The widow appealed to the CA but CA sustained trial court’s decision. ISSUES:

1. Whether the Court of Appeals erred in holding petitioner liable to DBP as

beneficiary in a group life insurance contract from a complaint filed by the widow of the decedent/mortgagor? NO.

2. Whether the Court of Appeals erred in not finding that Dr. Leuterio concealed that he had hypertension, which would vitiate the insurance contract? NO.

3. Whether the Court of Appeals erred in holding Grepalife liable in the amount of eighty six thousand, two hundred (P86, 200.00) pesos without proof of the actual outstanding mortgage payable by the mortgagor to DBP. NO.

HELD: Background: INSURABLE INTEREST IN MORTGAGED PROPERTIES & PARTIES TO THIS TYPE OF CONTRACT. The rationale of a group insurance policy of mortgagors, otherwise known as the “mortgage redemption insurance,” is a device for the protection of both the mortgagee and the mortgagor.

a. On the part of the mortgagee, it has to enter into such form of contract so that in the event of the unexpected demise of the mortgagor during the subsistence of the mortgage contract, the proceeds from such insurance will be applied to the payment of the mortgage debt, thereby relieving the heirs of the mortgagor from paying the obligation.

b. Ample protection is given to the mortgagor under such a concept so that in the event of death; the mortgage obligation will be extinguished by the application of the insurance proceeds to the mortgage indebtedness.

Consequently, where the mortgagor pays the insurance premium under the group insurance policy, making the loss payable to the mortgagee, the insurance is on the mortgagor’s interest, and the mortgagor continues to be a party to the contract. In this type of policy insurance, the mortgagee is simply an appointee of the insurance fund, such loss-payable clause does not make the mortgagee a party to the contract. Section 8 of the Insurance Code provides:

“Unless the policy provides, where a mortgagor of property effects insurance in his own name providing that the loss shall be payable to the mortgagee, or assigns a policy of insurance to a mortgagee, the insurance is deemed to be upon the interest of the mortgagor, who does not cease to be a party to the original contract, and any act of his, prior to the loss, which would otherwise avoid the insurance, will have the same effect, although the property is in the hands of the mortgagee, but any act which, under the contract of insurance, is to be performed by the mortgagor, may be performed by the mortgagee therein named, with the same effect as if it had been performed by the mortgagor.”

1. PETITIONER’S CONTENTION: Petitioner alleges that the complaint was instituted by the widow of Dr. Leuterio, not the real party in interest, hence the trial court acquired no jurisdiction over the case. The policy stating that: “In the event of the debtor’s death before his indebtedness with the Creditor [DBP] shall have been fully paid, an amount to pay the outstanding indebtedness shall first be paid to the creditor and the balance of sum assured, if there

Page 11: Insurance Case Digests (1)

Insurance Case Digests Atty. Bathan-Basuel

2-S, 2013-2014 11

is any, shall then be paid to the beneficiary/ies designated by the debtor.” And since a policy of insurance upon life or health may pass by transfer, will or succession to any person, whether he has an insurable interest or not, and such person may recover it whatever the insured might have recovered, the widow of the decedent Dr. Leuterio may file the suit against the insurer, Grepalife.

2. PETITIONER’S CONTENTION: Concealment exists where the assured had knowledge of a fact material to the risk, and honesty, good faith, and fair dealing requires that he should communicate it to the assured, but he designedly and intentionally withholds the same. On the contrary the medical findings were not conclusive because Dr. Mejia did not conduct an autopsy on the body of the decedent. As the attending physician, Dr. Mejia stated that he had no knowledge of Dr. Leuterio’s any previous hospital confinement. Dr. Leuterio’s death certificate stated that hypertension was only “the possible cause of death.” It was considered as hearsay. The fraudulent intent on the part of the insured must be established to entitle the insurer to rescind the contract. Misrepresentation as a defense of the insurer to avoid liability is an affirmative defense and the duty to establish such defense by satisfactory and convincing evidence rests upon the insurer. In the case at bar, the petitioner failed to clearly and satisfactorily establish its defense, and is therefore liable to pay the proceeds of the insurance. 3. PETITIONER’S CONTENTION: Petitioner claims that there was no evidence as to the amount of Dr. Leuterio’s outstanding indebtedness to DBP at the time of the mortgagor’s death. Petitioner’s claim is without merit. A life insurance policy is a valued policy. Unless the interest of a person insured is susceptible of exact pecuniary measurement, the measure of indemnity under a policy of insurance upon life or health is the sum fixed in the policy. The mortgagor paid the premium according to the coverage of his insurance, which states that:

“The policy states that upon receipt of due proof of the Debtor’s death during the terms of this insurance, a death benefit in the amount of P86,200.00 shall be paid.

In the event of the debtor’s death before his indebtedness with the creditor shall have been fully paid, an amount to pay the outstanding indebtedness shall first be paid to the Creditor and the balance of the Sum Assured, if there is any shall then be paid to the beneficiary/ies designated by the debtor.” NOTE: (Supervening event) Court of Appeals’ decision was promulgated on May 17, 1993. In private respondent’s memorandum, she states that DBP foreclosed in 1995 their residential lot, in satisfaction of mortgagor’s outstanding loan. Considering this supervening event, the insurance proceeds shall inure to the benefit of the heirs of the deceased person or his beneficiaries. Equity dictates that DBP

should not unjustly enrich itself at the expense of another (Nemo cum alterius detrimenio protest). Hence, it cannot collect the insurance proceeds, after it already foreclosed on the mortgage. The proceeds now rightly belong to Dr. Leuterio’s heirs represented by his widow, herein private respondent Medarda Leuterio. PETITION DENIED. 2. Section 8, Additional Cases

Geagonia v. CA, G.R. No. 114427 February 6, 1995 FACTS: The petitioner is the owner of Norman's Mart located in the public market of San Francisco, Agusan del Sur. On 22 December 1989, he obtained from the private respondent fire insurance policy for P100,000. The period of the policy was from 22 December 1989 to 22 December 1990 and covered the following: "Stock-in-trade consisting principally of dry goods such as RTW's for men and women wear and other usual to assured's business. The petitioner declared in the policy under the subheading entitled CO-INSURANCE that Mercantile Insurance Co., Inc. was the co-insurer for P50,000.00. From 1989 to 1990, the petitioner had in his inventory stocks amounting to P392,130.50. The policy contained the following condition:

3. The insured shall give notice to the Company of any insurance or insurances already affected, or which may subsequently be effected, covering any of the property or properties consisting of stocks in trade, goods in process and/or inventories only hereby insured, and unless such notice be given and the particulars of such insurance or insurances be stated therein or endorsed in this policy pursuant to Section 50 of the Insurance Code, by or on behalf of the Company before the occurrence of any loss or damage, all benefits under this policy shall be deemed forfeited, provided however, that this condition shall not apply when the total insurance or insurances in force at the time of the loss or damage is not more than P200,000.00. On 27 May 1990, fire of accidental origin broke out at around 7:30 p.m. at the public market of San Francisco, Agusan del Sur. The petitioner's insured stock-in-trade were completely destroyed prompting him to file with the private respondent a claim under the policy. On 28 December 1990, the private respondent denied the claim because it found that at the time of the loss the petitioner's stocks-in-trade were likewise covered by fire insurance policies No. GA-28146 and No. GA-28144, for P100,000.00 each, issued by the Cebu Branch of the Philippines First Insurance Co., Inc. (hereinafter PFIC). 3 These policies indicate that the insured was "Messrs. Discount Mart (Mr. Armando Geagonia, Prop.)" with a mortgage clause reading:

MORTGAGE: Loss, if any shall be payable to Messrs. Cebu Tesing Textiles, Cebu City as their interest may appear subject to the terms of this policy. CO-INSURANCE DECLARED:

Page 12: Insurance Case Digests (1)

Insurance Case Digests Atty. Bathan-Basuel

2-S, 2013-2014 12

P100,000. — Phils. First CEB/F 24758. 4

The basis of the private respondent's denial was the petitioner's alleged violation of Condition 3 of the policy. PETITIONER’S CONTENTION: at the time he obtained the private respondent's fire insurance policy he knew that the two policies issued by the PFIC were already in existence; however, he had no knowledge of the provision in the private respondent's policy requiring him to inform it of the prior policies; this requirement was not mentioned to him by the private respondent's agent; and had it been mentioned, he would not have withheld such information. RESPONDENT’S CONTENTION: denied the allegations in the complaint and set up as its principal defense the violation of Condition 3 of the policy. INSURANCE COMMISSION: petitioner did not violate Condition 3 as he had no knowledge of the existence of the two fire insurance policies obtained from the PFIC. CA: reversed the decision of I.C. ISSUES: (a) whether the petitioner had prior knowledge of the two insurance policies issued by the PFIC when he obtained the fire insurance policy from the private respondent, thereby, for not disclosing such fact, violating Condition 3 of the policy, and [YES] (b) if he had, whether he is precluded from recovering therefrom. [NO] HELD: We agree with the Court of Appeals that the petitioner knew of the prior policies issued by the PFIC. His letter of 18 January 1991 to the private respondent conclusively proves this knowledge. His testimony to the contrary before the Insurance Commissioner and which the latter relied upon cannot prevail over a written admission made ante litem motam. It was, indeed, incredible that he did not know about the prior policies since these policies were not new or original. Policy No. GA-28144 was a renewal of Policy No. F-24758, while Policy No. GA-28146 had been renewed twice, the previous policy being F-24792. Condition 3 of the private respondent's Policy No. F-14622 is a condition which is not proscribed by law. Its incorporation in the policy is allowed by Section 75 of the Insurance Code which provides: "[a] policy may declare that a violation of specified provisions thereof shall avoid it, otherwise the breach of an immaterial provision does not avoid the policy." Such a condition is a provision which invariably appears in fire insurance policies and is intended to prevent an increase in the moral hazard. It is commonly known as the additional or "other insurance" clause and has been upheld as valid and as a warranty that no other insurance exists. Its violation would thus avoid the policy. However, in order to constitute a violation, the other insurance must be upon same subject matter, the same interest therein, and the same risk.”

As to a mortgaged property, the mortgagor and the mortgagee have each an independent insurable interest therein and both interests may be one policy, or each may take out a separate policy covering his interest, either at the same or at separate times. A mortgagor may, however, take out insurance for the benefit of the mortgagee, which is the usual practice. The mortgagee may be made the beneficial payee in several ways. He may become the assignee of the policy with the consent of the insurer; or the mere pledgee without such consent; or the original policy may contain a mortgage clause; or a rider making the policy payable to the mortgagee "as his interest may appear" may be attached; or a "standard mortgage clause," containing a collateral independent contract between the mortgagee and insurer, may be attached; or the policy, though by its terms payable absolutely to the mortgagor, may have been procured by a mortgagor under a contract duty to insure for the mortgagee's benefit, in which case the mortgagee acquires an equitable lien upon the proceeds. It is a cardinal rule on insurance that a policy or insurance contract is to be interpreted liberally in favor of the insured and strictly against the company, the reason being, undoubtedly, to afford the greatest protection which the insured was endeavoring to secure when he applied for insurance. It is also a cardinal principle of law that forfeitures are not favored and that any construction which would result in the forfeiture of the policy benefits for the person claiming thereunder, will be avoided, if it is possible to construe the policy in a manner which would permit recovery, as, for example, by finding a waiver for such forfeiture. With these principles in mind, we are of the opinion that Condition 3 of the subject policy is not totally free from ambiguity and must, perforce, be meticulously analyzed. Such analysis leads us to conclude that (a) the prohibition applies only to double insurance, and (b) the nullity of the policy shall only be to the extent exceeding P200,000.00 of the total policies obtained. The first conclusion is supported by the portion of the condition referring to other insurance "covering any of the property or properties consisting of stocks in trade, goods in process and/or inventories only hereby insured," and the portion regarding the insured's declaration on the subheading CO-INSURANCE that the co-insurer is Mercantile Insurance Co., Inc. in the sum of P50,000.00. A double insurance exists where the same person is insured by several insurers separately in respect of the same subject and interest. As earlier stated, the insurable interests of a mortgagor and a mortgagee on the mortgaged property are distinct and separate. Since the two policies of the PFIC do not cover the same interest as that covered by the policy of the private respondent, no double insurance exists. The non-disclosure then of the former policies was not fatal to the petitioner's right to recover on the private respondent's policy. When a property owner obtains insurance policies from two or more insurers in a total amount that exceeds the property's value, the insured may have an inducement to destroy the property for the purpose of collecting the insurance. The public as well as

Page 13: Insurance Case Digests (1)

Insurance Case Digests Atty. Bathan-Basuel

2-S, 2013-2014 13

the insurer is interested in preventing a situation in which a fire would be profitable to the insured. PETITION GRANTED. CA DECISION SET ASIDE.

Palileo v. Cosio, G.R. No. L-7667, November 28, 1955 Cherie Pali leo vs Beatriz Cosio G.R. No. L-7667 November 28, 1955 J. Bautista Angelo FACTS: Plaintiff Cherie Palileo obtained from defendant Beatriz Cosio a loan of P12,000 with these conditions: (a) that plaintiff shall pay to defendant an interest in the amount of P250 a month; (b) that defendant shall deduct from the loan certain obligations of plaintiff to third persons amounting to P4,550, plus the sum of P250 as interest for the first month; and (c) that after making the above deductions, defendant shall deliver to plaintiff only the balance of the loan of P12,000. Cherie paid Beatriz a sum of P2,250 interest corresponding for 9 months, which is more than the maximum interest authorized by law. To secure the payment of the loan, defendant Beatriz required plaintiff to sign a document known as "Conditional Sale of Residential Building", purporting to convey to defendant, with right to repurchase, a two-story building of strong materials belonging to plaintiff Cherrie. This document did not express the true intention of the parties which was merely to place said property as security for the payment of the loan. After the execution of the aforesaid document, defendant insured the building against fire with the Associated Insurance & Surety Co., Inc. for the sum of P15,000, the insurance policy having been issued in the name of defendant Beatriz. When the building was partly destroyed by fire, defendant Beatriz collected from the insurance company an indemnity of P13,107.00. Plaintiff Cherrie asked defendant that she be credited with the necessary amount to pay her obligation out of the insurance proceeds but defendant refused to do so. The lower court ruled in favor of plaintiff Cherrie, declaring the transaction as one of equitable mortgage to secure the 12,000-peso loan and ordering the defendant to credit the sum of P13,107 received by the defendant from the Associated Insurance & Surety Co., Inc. to the payment of plaintiff's obligation in the sum of P12,000.00 and lastly to return the overpaid interest in the sum of P810 and the balance of 1,107 pesos (difference between the amount of loan and the proceeds of the insurance). ISSUE: Whether the trial court is correct in in considering the obligation of plaintiff fully compensated by the insurance amount and in ordering defendant to refund to plaintiff the sum of P1,107 notwithstanding the fact that it was not proven that the insurance was taken for the benefit of the mortgagor

HELD: No. The rule is that "where a mortgagee, independently of the mortgagor, insures the mortgaged property in his own name and for his own interest, he is entitled to the insurance proceeds in case of loss, but in such case, he is not allowed to retain his claim against the mortgagor, but is passed by subrogation to the insurer to the extent of the money paid." (Vance on Insurance, 2d ed., p. 654) Or, stated in another way, "the mortgagee may insure his interest in the property independently of the mortgagor. In that event, upon the destruction of the property the insurance money paid to the mortgagee will not inure to the benefit of the mortgagor, and the amount due under the mortgage debt remains unchanged. The mortgagee, however, is not allowed to retain his claim against the mortgagor, but it passes by subrogation to the insurer, to the extent of the insurance money paid." "The general rule and the weight of authority is, that the insurer is thereupon subrogated to the rights of the mortgagee under the mortgage. This is put upon the analogy of the situation of the insurer to that of a surety." Consistent with the foregoing pronouncement, we therefore modify the judgment of the lower court as follows: (1) the transaction had between the plaintiff and defendant as shown in Exhibit A is merely an equitable mortgage intended to secure the payment of the loan of P12,000; (2) that the proceeds of the insurance amounting to P13,107.00 was properly collected by defendant who is not required to account for it to the plaintiff; (3) that the collection of said insurance proceeds shall not be deemed to have compensated the obligation of the plaintiff to the defendant, but bars the latter from claiming its payment from the former; and (4) defendant shall pay to the plaintiff the sum of P810.00 representing the overpayment made by plaintiff by way of interest on the loan. No pronouncement as to costs. 3. Sections 10-25

Insular Life Assurance Co., Ltd vs Ebrado, 80 SCRA 181 G.R. No. L-44059 October 28, 1977 THE INSULAR LIFE ASSURANCE COMPANY, LTD., plaintiff-appellee, vs. CARPONIA T. EBRADO and PASCUALA VDA. DE EBRADO, defendants-appellants. MARTIN, J.: FACTS: 1) On September 1, 1968, Buenaventura Cristor Ebrado was issued by The Life

Assurance Co., Ltd., Policy No. 009929 on a whole-life for P5,882.00 with a, rider for Accidental Death for the same amount. Buenaventura C. Ebrado designated T. Ebrado as the revocable beneficiary in his policy. He to her as his wife.

2) On October 21, 1969, Buenaventura C. Ebrado died as a result of an t when he was

hit by a failing branch of a tree.

Page 14: Insurance Case Digests (1)

Insurance Case Digests Atty. Bathan-Basuel

2-S, 2013-2014 14

3) As the policy was in force, The Insular Life Assurance Co., Ltd. Liable to pay the coverage in the total amount of P11,745.73, representing the face value of the policy in the amount of P5,882.00 plus the additional benefits for accidental death also in the amount of P5,882.00 and the refund of P18.00 paid for the premium due November, 1969, minus the unpaid premiums and interest thereon due for January and February, 1969, in the sum of P36.27.

4) Carponia T. Ebrado filed with the insurer a claim for the proceeds of the Policy as

the designated beneficiary therein, although she admits that she and the insured Buenaventura C. Ebrado were merely living as husband and wife without the benefit of marriage.

5) Pascuala Vda. de Ebrado also filed her claim as the widow of the deceased insured.

She asserts that she is the one entitled to the insurance proceeds, not the common-law wife, Carponia T. Ebrado.

6) In doubt as to whom the insurance proceeds shall be paid, the insurer, The Insular

Life Assurance Co., Ltd. commenced an action for Interpleader before the Court of First Instance of Rizal on April 29, 1970.

7) After the issues have been joined, a pre-trial conference was held on July 8, 1972.

During this conference, parties Carponia T. Ebrado and Pascuala Ebrado agreed and stipulated among others that: a. that the deceased Buenaventura Ebrado was married to Pascuala Ebrado with

whom she has six — (legitimate) namely; Hernando, Cresencio, Elsa, Erlinda, Felizardo and Helen, all surnamed Ebrado;

b. that during the lifetime of Buenaventura Ebrado, he was living with his common-wife, Carponia Ebrado, with whom she had 2 children although he was not legally separated from his legal wife;

8) The trial court rendered judgment declaring among others, Carponia T. Ebrado

disqualified from becoming beneficiary of the insured Buenaventura Cristor Ebrado and directing the payment of the insurance proceeds to the estate of the deceased insured. From this judgment, Carponia T. Ebrado appealed to the Court of Appeals, but the Appellate Court certified the case to the Supreme Court as involving only questions of law.

ISSUE: Whether a common-law wife named as beneficiary in the life insurance policy of a legally married man can claim the proceeds thereof in case of death of the latter HELD: No. It is quite unfortunate that the Insurance Act (RA 2327, as amended) or even the new Insurance Code (PD No. 612, as amended) does not contain any specific provision grossly resolutory of the prime question at hand. Section 50 of the Insurance Act which provides that "(t)he insurance shall be applied exclusively to the proper interest of the person in whose name it is made" cannot be validly seized upon to hold that the mm includes the beneficiary. The word "interest" highly suggests that the provision refers only to the

"insured" and not to the beneficiary, since a contract of insurance is personal in character. Otherwise, the prohibitory laws against illicit relationships especially on property and descent will be rendered nugatory, as the same could easily be circumvented by modes of insurance. Rather, the general rules of civil law should be applied to resolve this void in the Insurance Law. Article 2011 of the New Civil Code states: "The contract of insurance is governed by special laws. Matters not expressly provided for in such special laws shall be regulated by this Code." When not otherwise specifically provided for by the Insurance Law, the contract of life insurance is governed by the general rules of the civil law regulating contracts. And under Article 2012 of the same Code, "any person who is forbidden from receiving any donation under Article 739 cannot be named beneficiary of a life insurance policy by the person who cannot make a donation to him. Common-law spouses are, definitely, barred from receiving donations from each other. Article 739 of the new Civil Code provides:

The following donations shall be void: 1. Those made between persons who were guilty of adultery or

concubinage at the time of donation; 2. Those made between persons found guilty of the same criminal

offense, in consideration thereof; 3. Those made to a public officer or his wife, descendants or

ascendants by reason of his office. In the case referred to in No. 1, the action for declaration of nullity may be brought by the spouse of the donor or donee; and the guilt of the donee may be proved by preponderance of evidence in the same action. In essence, a l ife insurance policy is no different from a civil donation insofar as the beneficiary is concerned. Both are founded upon the same consideration: l iberality. A beneficiary is l ike a donee, because from the premiums of the policy which the insured pays out of l iberality, the beneficiary wil l receive the proceeds or profits of said insurance. As a consequence, the proscription in Article 739 of the new Civil Code should equally operate in l ife insurance contracts. The mandate of Article 2012 cannot be laid aside: any person who cannot receive a donation cannot be named as beneficiary in the life insurance policy of the person who cannot make the donation. Under American law, a policy of life insurance is considered as a testament and in construing it, the courts will, so far as possible treat it as a will and determine the effect of a clause designating the beneficiary by rules under which wins are interpreted.

Southern Luzon Employee’s Association v. Golpeo, G.R. No. L-6114, October 30, 1954

Facts: The plaintiff, Southern Luzon Employees' Association is composed of laborers and employees of Laguna tayabas Bus Co., and Batangas Transportation Company, and one of its purposes is mutual aid of its members and their defendants in case of death. Roman A. Concepcion was a mem

Page 15: Insurance Case Digests (1)

Insurance Case Digests Atty. Bathan-Basuel

2-S, 2013-2014 15

In the form required by the association to be accomplished by its members, with reference to the death benefit, Roman A. Concepcion listed as his beneficiaries Aquilina Maloles, Roman M. Concepcion, Jr., Estela M. Concepcion, Rolando M. Concepcion and Robin M. Concepcion.

After the death of Roman A. Concepcion, the association was able to collect voluntary contributions from its members amounting to P2,5055. Three sets of claimants presented themselves, namely, (1) Juanita Golpeo, legal wife of Roman A. Concepcion, and her children, Aquilina Maloles, common law wife of Roman, and her children, named beneficiaries by the deceased; and (3) Elsie Hicban, another common law wife of Roman A. Concepcion, and her child.

After hearing, the court rendered a decision, declaring the defendants Aquilina Maloles and her children the sole beneficiaries of the sum of P2,505.00, and ordering the plaintiff to deliver said amount to them.

Issue: whether the court erred in designating a common law wife of an insured as the beneficiary instead of the legal wife?

Held: no. judgement was affirmed!

The decision is based mainly on the theory that the contract between the plaintiff and the deceased Roman A. Concepcion partook of the nature of an insurance and that, therefore, the amount in question belonged exclusively to the beneficiaries, invoking the following pronouncements of this Court in the case of Del Val vs. Del Val, 29 Phil., 534:

With the finding of the trial court that the proceeds of the life-insurance policy belongs exclusively to the defendant as his individual and separate property, we agree. That the proceeds of an insurance policy belong exclusively to the beneficiary and not to the estate of the person whose life was insured, and that such proceeds are the separate and individual property of the beneficiary, and not of the heirs of the person whose life was insured, is the doctrine in America. We believe that the same doctrine obtains in these Islands by virtue of section 428 of the Code of Commerce, which reads:

"The amounts which the underwriter must deliver to the person insured, in fulfillment of the contract, shall be the property creditors of any kind whatsoever of the person who effected the insurance in favor of the formers."

Appellant also contend that the stipulation between the plaintiff and the deceased Roman A. Concepcion regarding the specification of the latter's beneficiaries, and the resolution of September 17, 1949, are void for the being contrary to law, moral or public policy. Specifically, the appellants cite article 2012 of the new Civil Code providing that "Any person who is forbidden from receiving any donation under article 739 cannot be named beneficiary of a life insurance policy and by the person who

cannot make any donation to him, according to said article." Inasmuch as, according to article 739 of the new Civil Code, a donation is valid when made "between persons who are guilty or adultery or concubinage at the time of the donation," it is alleged that the defendant-appellee Aquilina Maloles, cannot be named a beneficiary, every assuming that the insurance law is applicable. Without considering the intimation in the brief for the defendant appellees that appellant Juanita Golpeo, by her silence and actions, had acquiesced in the illicit relations between her husband and appellee Aquilina Maloles, appellant argument would certainly not apply to the children of Aquilina likewise named beneficiaries by the deceased Roman A. Concepcion. As a matter of a fact the new Civil Code recognized certain successional rights of illegitimate children. (Article 287.)

Gercio v. Sun Life Insurance of Canada, G.R. No. 23703 September 28, 1925

FACTS: On January 29, 1910, the defendant Sun Life Assurance Co. of Canada issued insurance policy No. 161481 on the life of Hilario Gercio. By its terms, the insurance company agreed to insure the life of Hilario Gercio for the sum of P2,000, to be paid him on February 1, 1930, or if the insured should die before said date, then to his wife, Mrs. Andrea Zialcita, should she survive him; otherwise to the executors, administrators, or assigns of the insured. The policy did not include any provision reserving to the insured the right to change the beneficiary.

On the date the policy was issued, Andrea Zialcita was the lawful wife of Hilario Gercio. Towards the end of the year 1919, she was convicted of the crime of adultery. On September 4, 1920, a decree of divorce was issued in civil case no. 17955, which had the effect of completely dissolving the bonds of matrimony contracted by Hilario Gercio and Andrea Zialcita.

On March 4, 1922, Hilario Gercio formally notified the Sun Life Assurance Co. of Canada that he had revoked his donation in favor of Andrea Zialcita, and that he had designated in her stead his present wife, Adela Garcia de Gercio, as the beneficiary of the policy. Gercio requested the insurance company to eliminate Andrea Zialcita as beneficiary, which the insurance company has refused and still refuses to do.

ISSUE: Whether the insured — plaintiff Gercio — has the power/right to change the beneficiary — his former wife — and to name instead his actual wife, where the insured and the beneficiary have been divorced and where the policy of insurance does not expressly reserve to the insured the right to change the beneficiary?

Held: NO, the wife has an insurable interest in the life of her husband. The beneficiary has an absolute vested interest in the policy from the date of its issuance and delivery. So when a policy of life insurance is taken out by the husband in which the wife is named as beneficiary, she has a subsisting interest in the policy. And this applies to a policy to which there are attached the incidents of a loan value, cash surrender value, an automatic extension by premiums paid, and to an endowment policy, as well as to an ordinary life insurance policy. If the husband wishes to retain to himself the control and

Page 16: Insurance Case Digests (1)

Insurance Case Digests Atty. Bathan-Basuel

2-S, 2013-2014 16

ownership of the policy he may so provide in the policy. But if the policy contains no provision authorizing a change of beneficiary without the beneficiary's consent, the insured cannot make such change. Accordingly, it is held that a life insurance policy of a husband made payable to the wife as beneficiary, is the separate property of the beneficiary and beyond the control of the husband.

As to the effect produced by the divorce, the Philippine Divorce Law, Act No. 2710, merely provides in section 9 that the decree of divorce shall dissolve the community property as soon as such decree becomes final. Unlike the statutes of a few jurisdictions, there is no provision in the Philippine Law permitting the beneficiary in a policy for the benefit of the wife of the husband to be changed after a divorce. It must follow, therefore, in the absence of a statute to the contrary, that if a policy is taken out upon a husband's life the wife is named as beneficiary therein, a subsequent divorce does not destroy her rights under the policy.

Since it is said that our Insurance Act is mostly taken from the statute of California, it should prove of interest to know the stand taken by the Supreme Court of that State. A California decision oft cited in the Cyclopedias isYore vs. Booth, in which we find the following:

. . . It seems to be the settled doctrine, with but slight dissent in the courts of this country, that a person who procures a policy upon his own life, payable to a designated beneficiary, although he pays the premiums himself, and keeps the policy in his exclusive possession, has no power to change the beneficiary, unless the policy itself, or the charter of the insurance company, so provides. In policy, although he has parted with nothing, and is simply the object of another's bounty, has acquired a vested and irrevocable interest in the policy, which he may keep alive for his own benefit by paying the premiums or assessments if the person who effected the insurance fails or refuses to do so.

Another controlling decision of the United States Supreme Court is that of the Central National Bank of Washington City vs. Hume ([1888], 128 U.S., 134). Therein, Mr. Chief Justice Fuller, as the organ of the court, announced the following doctrines:

We think it cannot be doubted that in the instance of contracts of insurance with a wife or children, or both, upon their insurable interest in the life of the husband or father, the latter, while they are living, can exercise no power of disposition over the same without their consent, nor has he any interest therein of which he can avail himself; nor upon his death have his personal representatives or his creditors any interest in the proceeds of such contracts, which belong to the beneficiaries to whom they are payable.

It is indeed the general rule that a policy, and the money to become due under it, belong, the moment it is issued, to the person or persons named in it as the beneficiary or beneficiaries, and that there is no power in the person procuring

the insurance, by any act of his, by deed or by will, to transfer to any other person the interest of the person named.

Vda. de Consuegra v. GSIS 37 SCRA 315 (1971) VDA. DE CONSUEGRA vs. GOVERNMENT SERVICE INSURANCE SYSTEM

FACTS:

Jose Consuegra, at the time of his death, was employed as a shop foreman of the office of the District Engineer in the province of Surigao del Norte. In his lifetime, Consuegra contracted two marriages, the first with herein respondent Rosario Diaz,; and the second, which was contracted in good faith while the first marriage was subsisting, with herein petitioner Basilia Berdin, out of which marriage were born seven children

Being a member of the Government Service Insurance System (GSIS, for short) when Consuegra died on September 26, 1965, the proceeds of his life insurance were paid by the GSIS to petitioner Basilia Berdin and her children who were the beneficiaries named in the policy. Having been in the service of the government for 22.5028 years, Consuegra was entitled to retirement insurance benefits in the sum of P6,304.47. Consuegra did not designate any beneficiary who would receive the retirement insurance benefits due to him.

Respondent Rosario Diaz, the widow by the first marriage, filed a claim with the GSIS asking that the retirement insurance benefits be paid to her as the only legal heir of Consuegra, considering that the deceased did not designate any beneficiary with respect to his retirement insurance benefits. Petitioner Basilia Berdin and her children, likewise, filed a similar claim with the GSIS, asserting that being the beneficiaries named in the life insurance policy of Consuegra, they are the only ones entitled to receive the retirement insurance benefits due the deceased Consuegra. Resolving the conflicting claims, the GSIS ruled that the legal heirs of the late Jose Consuegra were Rosario Diaz, his widow by his first marriage who is entitled to one-half, or 8/16, of the retirement insurance benefits, on the one hand; and Basilia Berdin, his widow by the second marriage and their seven children, on the other hand, who are entitled to the remaining one-half, or 8/16, each of them to receive an equal share of 1/16.

It is the submission of appellants that because the deceased Jose Consuegra failed to designate the beneficiaries in his retirement insurance, the appellants who were the beneficiaries named in the life insurance should automatically be considered the beneficiaries to receive the retirement insurance benefits, to the exclusion of respondent Rosario Diaz.

ISSUE:

Page 17: Insurance Case Digests (1)

Insurance Case Digests Atty. Bathan-Basuel

2-S, 2013-2014 17

To whom should the retirement insurance benefits of Jose Consuegra be paid, because he did not, or failed to, designate the beneficiary of his retirement insurance?

HELD:

When Consuegra designated his beneficiaries in his life insurance he could not have intended those beneficiaries of his life insurance as also the beneficiaries of his retirement insurance because the provisions on retirement insurance under the GSIS came about only when Com. Act 186 was amended by Rep. Act 660 on June 16, 1951. Hence, it cannot be said that because herein appellants were designated beneficiaries in Consuegra's life insurance they automatically became the beneficiaries also of his retirement insurance.

Section 11 of Commonwealth Act 186, as amended by Rep. Act 660, clearly indicate that there is need for the employee to file an application for retirement insurance benefits when he becomes a member of the GSIS, and he should state in his application the beneficiary of his retirement insurance. Hence, the beneficiary named in the life insurance does not automatically become the beneficiary in the retirement insurance unless the same beneficiary in the life insurance is so designated in the application for retirement insurance.

Thus, We see that the GSIS offers two separate and distinct systems of benefits to its members — one is the life insurance and the other is the retirement insurance. These two distinct systems of benefits are paid out from two distinct and separate funds that are maintained by the GSIS.

If the employee failed or overlooked to state the beneficiary of his retirement insurance, the retirement benefits will accrue to his estate and will be given to his legal heirs in accordance with law, as in the case of a life insurance if no beneficiary is named in the insurance policy.

It is Our view, therefore, that the respondent GSIS had correctly acted when it ruled that the proceeds of the retirement insurance of the late Jose Consuegra should be divided equally between his first living wife Rosario Diaz, on the one hand, and his second wife Basilia Berdin and his children by her, on the other.

Nario vs. Phil-Am Life Ins. Co. 20 SCRA 434 G.R. No. L-22796 June 26, 1967 DELFIN NARIO, and ALEJANDRA SANTOS-NARIO, plaintiffs-appellants, vs. THE PHILIPPINE AMERICAN LIFE INSURANCE COMPANY, defendant-appellee. REYES, J.B.L., J.:

FACTS: Mrs. Alejandra Santos-Nario was, upon application, issued, on June 12, 1959, by the Phil ippine American Life Insurance Co., a l ife insurance policy (No. 503617) under a 20-year endowment plan, with a face value of P5,000.00. She designated thereon her husband, Delfin Nario, and their unemancipated minor son, Ernesto Nario, as her irrevocable beneficiaries.

About the middle of June, 1963, Mrs. Nario applied for a loan on the above stated policy with the Insurance Company, which loan she, as policy-holder, has been entitled to avail of under one of the provisions of said policy after the same has been in force for three (3) years, for the purpose of using the proceeds thereof for the school expenses of her minor son, Ernesto Nario. Said application bore the written signature and consent of Delfin Nario in two capacities: first, as one of the irrevocable beneficiaries of the policy; and the other, as the father-guardian of said minor son and irrevocable beneficiary, Ernesto Nario, and as the legal administrator of the minor's properties, pursuant to Article 320 of the Civil Code of the Philippines.

The Insurance Company denied said application, manifesting to the policy holder that the written consent for the minor son must not only be given by his father as legal guardian but it must also be authorized by the court in a competent guardianship proceeding.

After the denial of said policy loan application, Mrs. Nario signified her decision to surrender her policy to the Insurance Company, which she was also entitled to avail of under one of the provisions of the same policy, and demanded its cash value which then amounted to P520.00.

The Insurance Company also denied the surrender of the policy, on the same ground as that given in disapproving the policy loan application; hence, on September 10, 1963, Mrs. Alejandra Santos-Nario and her husband, Delfin Nario, brought suit against the Phil ippine American Life Insurance Co. in the above mentioned court of first instance, seeking to compel Philam to grant their policy loan application and/or to accept the surrender of said policy in exchange for its cash value.

Philam answered the complaint, virtually admitting its material allegations, but it set up the affirmative defense that inasmuch as the policy loan application and the surrender of the policy involved acts of disposition and alienation of the property rights of the minor, said acts are not within the powers of the legal administrator, under article 320 in relation to article 326 of the Civil Code; hence, mere written consent given by the father-guardian, for and in behalf of the minor son, without any court authority therefor, was not a sufficient compliance of the law, and Philam was, therefore, justified in refusing to grant and in disapproving the proposed transactions in question.

The lower court found and opined that since the parties expressly stipulated in the endorsement attached to the policy and which formed part thereof that —

Page 18: Insurance Case Digests (1)

Insurance Case Digests Atty. Bathan-Basuel

2-S, 2013-2014 18

It is hereby understood and agreed that, notwithstanding the provisions of this Policy to the contrary, inasmuch as the designation of the beneficiaries have been made by the Insured without reserving the right to change said beneficiaries, the Insured may not designate a new beneficiary or assign, release or surrender this Policy to the Company and exercise any and all other rights and privileges hereunder or agree with the Company to any change in or amendment to this Policy, without the consent of the beneficiaries originally designated;

that under the above quoted provision, the minor son, as one of the designated irrevocable beneficiaries, "acquired a vested right to all benefits accruing to the policy, including that of obtaining a policy loan to the extent stated in the schedule of values attached to the policy; that the proposed transactions in question (policy loan and surrender of policy) involved acts of disposition or alienation of the minor's properties for which the consent given by the father-guardian for and in behalf of the minor son, must be with the requisite court authority; and in the case at bar, such consent was given by the father-guardian without any judicial authority; said court, agreeing with defendant's contention, sustained defendant's affirmative defense, and rendered, on January 28, 1964, its decision dismissing plaintiffs' complaint.

Unable to secure reconsideration of the trial Court's ruling, petitioner appealed directly to this Court.

ISSUE: Whether PhilAm Life is justified in denying the proposed transaction.

HELD: The appeal is unmeritorious. We agree with the lower court that the vested interest or right of the beneficiaries in the policy should be measured on its full face value and not on its cash surrender value, for in case of death of the insured, said beneficiaries are paid on the basis of its face value and in case the insured should discontinue paying premiums, the beneficiaries may continue paying it and are entitled to automatic extended term or paid-up insurance options, etc. and that said vested right under the policy cannot be divisible at any given time. The proposed transactions in question (policy loan and surrender of policy) constitute acts of disposition or alienation of property rights and not merely of management or administration because they involve the incurring or termination of contractual obligations.

As above noted, the full face value of the policy is P5,000.00 and the minor's vested interest therein, as one of the two (2) irrevocable beneficiaries, consists of one-half (½) of said amount or P2,500.00.

Article 320 of the Civil Code of the Philippines provides —

The father, or in his absence the mother, is the legal administrator of the property pertaining to the child under parental authority. If the property is

worth more than two thousand pesos, the father or mother shall give a bond subject to the approval of the Court of First Instance.

and article 326 of the same Code reads —

When the property of the child is worth more than two thousand pesos, the father or mother shall be considered a guardian of the child's property, subject to the duties and obligations of guardians under the Rules of Court.

It appearing that the minor beneficiary's vested interest or right on the policy exceeds two thousand pesos (P2,000.00); that plaintiffs did not file any guardianship bond to be approved by the court; and as later implemented in Section 7, Rule 93 of the Revised Rules of Court, plaintiffs should have, but, had not, filed a formal application or petition for guardianship, plaintiffs-parents cannot possibly exercise the powers vested on them, as legal administrators of their child's property, under articles 320 and 326 of the Civil Code. As there was no such petition and bond, the consent given by the father-guardian, for and in behalf of the minor son, without prior court authorization, to the policy loan application and the surrender of said policy, was insufficient and ineffective, and defendant-appellee was justified in disapproving the proposed transactions in question.

The result would be the same even if we regarded the interest of the ward to be worth less than P2,000.00. While the father or mother would in such event be exempt from the duty of filing a bond, and securing judicial appointment, still the parent's authority over the estate of the ward as a legal-guardian would not extend to acts of encumbrance or disposition, as distinguished from acts of management or administration. The distinction between one and the other kind of power is too basic in our law to be ignored. Thus, under Article 1877 of the Civil Code of the Philippines, an agency in general terms does not include power to encumber or dispose of the property of the principal; and the Code explicitly requires a special power or authority for the agent "to loan or borrow money, unless the latter act be urgent or indispensable for the preservation of the thing under administration" Similarly, special powers are required to required to effect novations, to waive any obligation gratuitously or obligate the principal as a guarantor or surety. By analogy, since the law merely constitutes the parent as legal administrator of the child's property (which is a general power), the parent requires special authority for the acts above specified, and this authority can be given only by a court. This restricted interpretation of the parent's authority becomes all the more necessary where as in the case before us, there is no bond to guarantee the ward against eventual losses.

Appellants seek to bolster their petition by invoking the parental power (patria potestas) under the Civil Code of 1889.The appeal profits them nothing. For the new Civil Code has not effected a restitutio in integrum of the Spanish patria potestas; the revival has been only in part.

Page 19: Insurance Case Digests (1)

Insurance Case Digests Atty. Bathan-Basuel

2-S, 2013-2014 19

Wherefore, the decision appealed from is affirmed. Costs against appellants Nario. So ordered.

Phil-Am Life Ins Co vs. Pineda 175 SCRA 416 FACTS: On January 15, 1968, private respondent procured an ordinary life insurance policy from the petitioner company and designated his wife and children as irrevocable beneficiaries of said policy. Under date February 22, 1980 private respondent filed a petition which was docketed as Civil Case No. 9210 of the then Court of First Instance of Rizal to amend the designation of the beneficiaries in his life policy from irrevocable to revocable. Petitioner then, on March 10, 1980 filed an Urgent Motion to Reset Hearing. Also on the same date, petitioner filed its Comment and/or Opposition to Petition. When the petition was called for hearing on March 19, 1980, the respondent Judge Gregorio G. Pineda, presiding Judge of the then Court of First Instance of Rizal, Pasig Branch XXI, denied petitioner's Urgent Motion, thus allowing the private respondent to adduce evidence, the consequence of which was the issuance of the questioned Order granting the petition. Petitioner promptly filed a Motion for Reconsideration but the same was denied in an Order June 10, 1980. Hence, this petition. ISSUE: 1. WHETHER THE DESIGNATION OF THE IRREVOCABLE BENEFICIARIES COULD BE CHANGED OR AMENDED WITHOUT THE CONSENT OF ALL THE IRREVOCABLE BENEFICIARIES. 2. WHETHER THE IRREVOCABLE BENEFICIARIES HEREIN, ONE OF WHOM IS ALREADY DECEASED WHILE THE OTHERS ARE ALL MINORS, COULD VALIDLY GIVE CONSENT TO THE CHANGE OR AMENDMENT IN THE DESIGNATION OF THE IRREVOCABLE BENEFICIARIES. HELD: 1. NO. Needless to say, the applicable law in the instant case is the Insurance Act, otherwise known as Act No. 2427 as amended, the policy having been procured in 1968. Under the said law, the beneficiary designated in a life insurance contract cannot be changed without the consent of the beneficiary because he has a vested interest in the policy (Gercio v. Sun Life Ins. Co. of Canada, 48 Phil. 53; Go v. Redfern and the International Assurance Co., Ltd., 72 Phil. 71). In this regard, it is worth noting that the Beneficiary Designation Indorsement in the policy which forms part of Policy Number 0794461 in the name of Rodolfo Cailles Dimayuga states that the designation of the beneficiaries is irrevocable (Annex "A" of Petition in Sp. Proc. No. 9210, Annex "C" of

the Petition for Review on Certiorari), to wit: It is hereby understood and agreed that, notwithstanding the provisions of this policy to the contrary, inasmuch as the designation of the primary/contingent beneficiary/beneficiaries in this Policy has been made without reserving the right to change said beneficiary/ beneficiaries, such designation may not be surrendered to the Company, released or assigned; and no right or privilege under the Policy may be exercised, or agreement made with the Company to any change in or amendment to the Policy, without the consent of the said beneficiary/beneficiaries. (Petitioner's Memorandum, p. 72, Rollo) 2. NO. The alleged acquiescence of the six (6) children beneficiaries of the policy (the beneficiary-wife predeceased the insured) cannot be considered an effective ratification to the change of the beneficiaries from irrevocable to revocable. Indubitable is the fact that all the six (6) children named as beneficiaries were minors at the time,** for which reason, they could not validly give their consent. Neither could they act through their father insured since their interests are quite divergent from one another. The parent-insured cannot exercise rights and/or privileges pertaining to the insurance contract, for otherwise, the vested rights of the irrevocable beneficiaries would be rendered inconsequential.

Sps. Cha vs. Court of Appeals, 277 SCRA 690 Cha vs Court of Appeals Padilla On October 5, 1988, petitioner spouses Nilo Cha and Stella Uy-Cha as lessees, entered into a lease contract with respondent CKS Development Corporation. The lease contract specifically provides that: “ the lessee shall not insure against fire the textiles, chattels, merchandise, goods and effects placed at any stall or store or space in the leased premises without first obtaining the written consent and approval of the lessor (CKS).” It also provides that “if the lessee should should obtain the insurance thereof without the consent of the lessor then the policy is deemed assigned and transferred to the lessor for its own benefit.” The Cha spouses insured the merchandise inside the leased premises for P500,000.00 with United Insurance Co., notwithstanding the said provision in the contract. On the day that the lease was about to expire, fire broke out in the leased premises. CKS then wrote United that based on the contract of lease, United should pay the proceeds of the insurance directly to it (CKS). United refused. CKS filed a complaint in the RTC of Manila which later rendered a decision in favor of CKS. On appeal, the CA affirmed the decision of the RTC with the modifications.

Page 20: Insurance Case Digests (1)

Insurance Case Digests Atty. Bathan-Basuel

2-S, 2013-2014 20

Hence this petition by the Cha spouses. In this petition, the Cha spouses argued that the stipulation in the lease contract that providing for the transfer of the proceeds of the insurance to CKS is null and void for being contrary to law, morals and public policy. Issue: whether the automatic transfer of the proceeds of the insurance policy entered into by CKS and the Cha spouses is valid. Held: No, the said stipulation is invalid for being contrary to law, morals and public policy. A non-life insurance policy such as the fire insurance policy taken by the Cha spouses over the merchandise is primarily a contract of indemnityInsurable interest in the property insured must exist at the time the insurance takes effect and at the time the loss occurs. The basis of such requirement of insurable interest in property insured is based on sound public policy: to prevent a person from taking out an insurance policy on property upon which he has no insurable interest and collecting the proceeds of said policy in case of loss of the property. In such a case, the contract of insurance is a mere wager which is void under Section 25 of the Insurance Code . CKS has no insurable interest in the goods and merchandise inside the leased premises under Section 17 of the Insurance Code. Therefore, CKS cannot under the Insurance Code – a special law – be validly be a beneficiary of the fire insurance policy taken by the petitioner spouses. The liability of the Cha spouses to CKS for violating their lease contract in that the Cha spouses obtained a fire insurance policy over their own merchandise, without the consent of CKS is a separate and distinct issue.

Ong Lim Sing vs. Feb Leasing & Finance Corporation G.R. No. 168115, June 8, 2007

Ong Lim Sing vs Feb Leasing

Facts:

- FEB Leasing and Finance Corporation (FEB) entered into a lease of equipment and motor vehicles with JVL Food Products (JVL). On the same date, Vicente Ong Lim Sing, Jr. (Lim) executed an Individual Guaranty Agreement with FEB to guarantee the prompt and faithful performance of the terms and conditions of the aforesaid lease agreement. Corresponding Lease Schedules with Delivery and Acceptance Certificates over the equipment and motor vehicles formed part of the agreement. Under the contract, JVL was obliged to pay FEB an aggregate gross monthly rental of P170,494.00.

- JVL defaulted in the payment of the monthly rentals. the amount in arrears, including penalty charges and insurance premiums, amounted to P3,414,468.75. Thereafter, FEB sent a letter to JVL demanding payment of the said amount. However, JVL failed to pay.

-FEB filed a Complaint with the Regional Trial Court of Manila for sum of money, damages, and replevin against JVL, Lim, and John Doe.

-In the Amended Answer, JVL and Lim admitted the existence of the lease agreement but asserted that it is in reality a sale of equipment on installment basis, with FEB acting as the financier. JVL and Lim claimed that this intention was apparent from the fact that they were made to believe that when full payment was effected, a Deed of Sale will be executed by FEB as vendor in favor of JVL and Lim as vendees. FEB purportedly assured them that documenting the transaction as a lease agreement is just an industry practice and that the proper documentation would be effected as soon as full payment for every item was made. They also contended that the lease agreement is a contract of adhesion and should, therefore, be construed against the party who prepared it, FEB.

- TC: The case was a sale on installment there is no chattel mortgage on the thing sold, but it appears amongst the Complaint’s prayer, that the plaintiff elected to exact fulfillment of the obligation.

For the vehicles returned, the plaintiff can only recover the unpaid balance of the price because of the previous payments made by the defendants for the reasonable use of the units, specially so, as it appears, these returned vehicles were sold at auction and that the plaintiff can apply the proceeds to the balance. However, with respect to the unreturned units and machineries still in the possession of the defendants, they are liable therefore and accordingly are ordered jointly and severally to pay the price thereof to the plaintiff together with attorney’s fee and the costs of suit in the sum of Php25,000.00.

- FEB filed its Notice of Appeal.

- CA: declared the transaction between the parties as a financial lease agreement under Republic Act (R.A.) No. 8556.

Issue: Whether or not the agreement was a contract of adhesion. Whether petitioner has insurable interest.

Held:

(1) The subject lease agreement is a contract of adhesion, such a contract is not void per se. It is as binding as any ordinary contract. A party who enters into an adhesion contract is free to reject the stipulations entirely. If the terms thereof are accepted without objection, then the contract serves as the law between the parties.

- In Section 23 of the lease contract, it was expressly stated that:

SECTION 23. ENTIRE AGREEMENT; SEVERABILITY CLAUSE

23.1. The LESSOR and the LESSEE agree this instrument constitute the entire agreement between them, and that no representations have been made other than as set forth herein. This Agreement shall not be amended or altered in any manner, unless

Page 21: Insurance Case Digests (1)

Insurance Case Digests Atty. Bathan-Basuel

2-S, 2013-2014 21

such amendment be made in writing and signed by the parties hereto.

-Petitioner’s claim that the real intention of the parties was a contract of sale of personal property on installment basis is more likely a mere afterthought in order to defeat the rights of the respondent.

- FEB leased the subject equipment and motor vehicles to JVL in consideration of a monthly periodic payment of P170,494.00. The periodic payment by petitioner is sufficient to amortize at least 70% of the purchase price or acquisition cost of the said movables in accordance with the Lease Schedules with Delivery and Acceptance Certificates.

-The allegation of petitioner that the rent for the use of each movable constitutes the value of the vehicle or equipment leased is of no moment. The law on financial lease does not prohibit such a circumstance and this alone does not make the transaction between the parties a sale of personal property on installment. In fact, the value of the lease, usually constituting the value or amount of the property involved, is a benefit allowed by law to the lessor for the use of the property by the lessee for the duration of the lease. It is recognized that the value of these movables depreciates through wear and tear upon use by the lessee.

- the validity of Lease between FEB and JVL should be upheld. JVL entered into the lease contract with full knowledge of its terms and conditions. The contract was in force for more than four years. Since its inception JVL and Lim never questioned its provisions. They only attacked the validity of the contract after they were judicially made to answer for their default in the payment of the agreed rentals.

(2) The stipulation in the lease contract, that the equipment shall be insured at the cost and expense of the lessee against loss, damage, or destruction from fire, theft, accident, or other insurable risk for the full term of the lease, is a binding and valid stipulation. Petitioner, as a lessee, has an insurable interest in the equipment and motor vehicles leased. Section 17 of the Insurance Code provides that the measure of an insurable interest in property is the extent to which the insured might be damnified by loss or injury thereof. It cannot be denied that JVL will be directly damnified in case of loss, damage, or destruction of any of the properties leased.

- In the financial lease agreement, FEB did not assume responsibility as to the quality, merchantability, or capacity of the equipment. This stipulation provides that, in case of defect of any kind that will be found by the lessee in any of the equipment, recourse should be made to the manufacturer. “The financial lessor, being a financing company, an extender of credit rather than an ordinary equipment rental company, does not extend a warranty of the fitness of the equipment for any particular use. Thus, the financial lessee was precisely in a position to enforce such warranty directly against the supplier of the equipment and not against the financial lessor. The court found nothing contra legem or contrary to public policy in such a contractual arrangement.

Bachrach v. British American Insurance Company 17 Phil 555 (1910)

Facts: E.M. Bachrach commenced an action against the defendant British American Assurance Company to recover the amount due, deducting the salvage, upon the following fire insurance policy issued by the defendant to the plaintiff:

The defendant alleged certain facts under which it claimed that it was released from all obligations whatever under said policy. These special facts are as follows:

-That the plaintiff maintained a paint and varnish shop in the said building where the goods which were insured were stored.

-That the plaintiff transferred his interest in and to the property covered by the policy to H. W. Peabody & Co. to secure certain indebtedness due and owing to said company, and also that the plaintiff had transferred his interest in certain of the goods covered by the said policy to one Macke, to secure certain obligations assumed by the said Macke for and on behalf of the insured. That the sanction of the said British American Insurance had not been obtained by the plaintiff, as required by the said policy.

- That the plaintiff, immediately preceding the outbreak of the alleged fire, willfully placed a gasoline can containing 10 gallons of gasoline in the upper story of said building in close proximity to a portion of said goods, wares, and merchandise, which can was so placed by the plaintiff as to permit the gasoline to run on the floor of said second story, and after so placing said gasoline, he, the plaintiff, placed in close proximity to said escaping gasoline a lighted lamp containing alcohol, thereby greatly increasing the risk of fire.

Bachrach, after denying nearly all of the facts set out in the special answer of the defendant, alleged:

- That he had been acquitted in a criminal action against him, after a trial duly and regularly had, upon a charge of arson, based upon the same alleged facts set out in the answer of the defendant.

After hearing the evidence adduced during the trial of the cause, the lower court found that the defendant was liable to the plaintiff and rendered a judgment against the defendant for the sum.

Issue: Whether British American may not be held liable because

1. The use of the building as a paint and varnish shop annulled the policy of insurance.

2. The execution of the chattel mortgages without the knowledge and consent of the insurance company annulled the policy of insurance.

Page 22: Insurance Case Digests (1)

Insurance Case Digests Atty. Bathan-Basuel

2-S, 2013-2014 22

3. E. M. Bachrach, the insured, willfully placed a gasoline can containing about 10 gallons of gasoline in the upper story of said building, in close proximity to a portion of the goods, wares, and merchandise stored therein, and that said can was so placed by said Bachrach as to permit the gasoline to run on the floor of said second story.

4. The court erred in refusing to deduct from the loss sustained by Bachrach the value of the automobile, which was saved without damage.

Ruling: No. British American is ordered to pay Bachrach.

1.) It is claimed that either gasoline or alcohol was kept in violation of the policy in the bodega containing the insured property. The property insured consisted mainly of household furniture kept for the purpose of sale. The preservation of the furniture in a salable condition by retouching or otherwise was incidental to the business. It is well settled that the keeping of inflammable oils on the premises, though prohibited by the policy, does not void it if such keeping is incidental to the business. It may be added that there was no provision in the policy prohibiting the keeping of paints and varnishes upon the premises where the insured property was stored

2.) There is no provision in said policy prohibiting the plaintiff from placing a mortgage upon the property insured, but, admitting that such a provision was intended, the interest in property insured does not pass by the mere execution of a chattel mortgage and that while a chattel mortgage is a conditional sale, there is no alienation within the meaning of the insurance law until the mortgage acquires a right to take possession by default under the terms of the mortgage. No such right is claimed to have accrued in the case at bar, and the alienation clause is therefore inapplicable.

3.) The record discloses that some time prior to the commencement of this present action, a criminal action was commenced against the plaintiff, in which he was charged with willfully and maliciously burning the property covered by the policy in the present case. The lower court, that the evidence was insufficient to show beyond peradventure of doubt that the defendant was guilty of the crime. While the evidence shows some very peculiar and suspicious circumstances concerning the burning of the goods covered by the said policy, yet, nevertheless, in view of the findings of the lower court and in view of the apparent conflict in the testimony, we can not find that there is a preponderance of evidence showing that the plaintiff did actually set fire or cause fire to be set to the goods in question.

4.) It does not positively appear of record that the automobile in question was not included in the other policies. It does appear that the automobile was saved and was considered as a part of the salvaged. It is alleged that the salvage amounted to P4,000, including the automobile. This amount (P4,000) was distributed among the different insurers and the amount of their responsibility was proportionately reduced. The defendant and appellant in the present case made no objection at any time in the lower court to that distribution of the salvage. The claim is now made for the first time.

San Miguel Brewery vs. Law Union & Rock Ins. Co., 41 Phil 674 (1920) Facts:

- On Jan. 12, 1918, Dunn mortgaged a parcel of land to SMB to secure a debt of 10T.

- Mortgage contract stated that Dunn was to have the property insured at his own expense, authorizing SMB to choose the insurers and to receive the proceeds thereof and retain so much of the proceeds as would cover the mortgage debt.

- Dunn likewise authorized SMB to take out the insurance policy for him. - Brias, SMB’s general manager, approached Law Union for insurance to the

extent of 15T upon the property. In the application, Brias stated that SMB’s interest in the property was merely that of a mortgagee.

- Law Union, not wanting to issue a policy for the entire amount, issued one for P7,500 and procured another policy of equal amount from Filipinas Cia de Seguros. Both policies were issued in the name of SMB only and contained no reference to any other interests in the propty. Both policies required assignments to be approved and noted on the policy.

- Premiums were paid by SMB and charged to Dunn. A year later, the policies were renewed.

- In 1917, Dunn sold the property to Harding, but no assignment of the policies was made to the latter.

- Property was destroyed by fire. SMB filed an action in court to recover on the policies. Harding was made a defendant because by virtue of the sale, he became the owner of the property, although the policies were issued in SMB’s name.

- SMB sought to recover the proceeds to the extent of its mortgage credit with the balance to go to Harding.

- Insurance Companies contended that they were not liable to Harding because their liability under the policies was limited to the insurable interests of SMB only.

- SMB eventually reached a settlement with the insurance companies and was paid the balance of it’s mortgage credit. Harding was left to fend for himself. Trial court ruled against Harding. Hence the appeal.

Issue: Whether or not the insurance companies are liable to Harding for the balance of the proceeds of the 2 policies. Held: NO. Under the Insurance Act, the measure of insurable interest in the property is the extent to which the insured might be daminified by the loss or injury thereof. Also it is provided in the IA that the insurance shall be applied exclusively to the proper interest of the person in whose name it is made. Undoubtedly, SMB as the mortgagee of the property, had an insurable interest therein; but it could NOT, an any event, recover upon the two policies an amount in excess of its mortgage credit. By virtue of the Insurance Act, neither Dunn nor Harding could have recovered from the two policies. With respect to Harding, when he acquired the property, no change or

Page 23: Insurance Case Digests (1)

Insurance Case Digests Atty. Bathan-Basuel

2-S, 2013-2014 23

assignment of the policies had been undertaken. The policies might have been worded differently so as to protect the owner, but this was not done. If the wording had been: “Payable to SMB, mortgagee, as its interests may appear, remainder to whomsoever, during the continuance of the risk, may become owner of the interest insured”, it would have proved an intention to insure the entire interest in the property, NOT merely SMB’s and would have shown to whom the money, in case of loss, should be paid. Unfortunately, this was not what was stated in the policies. If during the negotiation for the policies, the parties had agreed that even the owner’s interest would be covered by the policies, and the policies had inadvertently been written in the form in which they were eventually issued, the lower court would have been able to order that the contract be reformed to give effect to them in the sense that the parties intended to be bound. However, there is no clear and satisfactory proof that the policies failed to reflect the real agreement between the parties that would justify the reformation of these two contracts.

Heirs of Loreto C. Maramag represented by surviving spouse Vicente Pangilinan Maramag v. Eva Verna De Guzman Maramag, et al, G.R. No. 181132, June 5, 2009

HEIRS OF LORETO C. MARAMAG, represented by surviving spouse VICENTA PANGILINAN MARAMAG,Petitioners, vs. EVA VERNA DE GUZMAN MARAMAG, ODESSA DE GUZMAN MARAMAG, KARL BRIAN DE GUZMAN MARAMAG, TRISHA ANGELIE MARAMAG, THE INSULAR LIFE ASSURANCE COMPANY, LTD., and GREAT PACIFIC LIFE ASSURANCE CORPORATION, Respondents. The petition alleged that: (1) petitioners were the legitimate wife and children of Loreto Maramag (Loreto), while respondents were Loreto’s illegitimate family; (2) Eva de Guzman Maramag (Eva) was a concubine of Loreto and a suspect in the killing of the latter, thus, she is disqualified to receive any proceeds from his insurance policies from Insular Life Assurance Company, Ltd. (Insular)4 and Great Pacific Life Assurance Corporation (Grepalife);5 (3) the illegitimate children of Loreto—Odessa, Karl Brian, and Trisha Angelie—were entitled only to one-half of the legitime of the legitimate children, thus, the proceeds released to Odessa and those to be released to Karl Brian and Trisha Angelie were inofficious and should be reduced; and (4) petitioners could not be deprived of their legitimes, which should be satisfied first.

In answer,6 Insular admitted that Loreto misrepresented Eva as his legitimate wife and Odessa, Karl Brian, and Trisha Angelie as his legitimate children, and that they filed their claims for the insurance proceeds of the insurance policies; that when it ascertained that Eva was not the legal wife of Loreto, it disqualified her as a beneficiary and divided the proceeds among Odessa, Karl Brian, and Trisha Angelie, as the remaining designated beneficiaries; and that it released Odessa’s share as she was of age, but withheld the release of the shares of minors Karl Brian and Trisha Angelie pending submission of letters of guardianship. Insular alleged that the complaint or petition failed to state a

cause of action insofar as it sought to declare as void the designation of Eva as beneficiary, because Loreto revoked her designation as such in Policy No. A001544070 and it disqualified her in Policy No. A001693029; and insofar as it sought to declare as inofficious the shares of Odessa, Karl Brian, and Trisha Angelie, considering that no settlement of Loreto’s estate had been filed nor had the respective shares of the heirs been determined. Insular further claimed that it was bound to honor the insurance policies designating the children of Loreto with Eva as beneficiaries pursuant to Section 53 of the Insurance Code.

In its own answer7 with compulsory counterclaim, Grepalife alleged that Eva was not designated as an insurance policy beneficiary; that the claims filed by Odessa, Karl Brian, and Trisha Angelie were denied because Loreto was ineligible for insurance due to a misrepresentation in his application form that he was born on December 10, 1936 and, thus, not more than 65 years old when he signed it in September 2001; that the case was premature, there being no claim filed by the legitimate family of Loreto; and that the law on succession does not apply where the designation of insurance beneficiaries is clear.

The RTC initially ruled in favour of plaintiff-petitioners but upon a motion for reconsideration by the respondents, it reversed its own decision and dismissed the case. CA affirmed ISSUE: (A)re the members of the legitimate family entitled to the proceeds of the insurance for the concubine?

HELD: although petitioners are the legitimate heirs of Loreto, they were not named as beneficiaries in the insurance policies issued by Insular and Grepalife. The basis of petitioners’ claim is that Eva, being a concubine of Loreto and a suspect in his murder, is disqualified from being designated as beneficiary of the insurance policies, and that Eva’s children with Loreto, being illegitimate children, are entitled to a lesser share of the proceeds of the policies. They also argued that pursuant to Section 12 of the Insurance Code,19 Eva’s share in the proceeds should be forfeited in their favor, the former having brought about the death of Loreto. Thus, they prayed that the share of Eva and portions of the shares of Loreto’s illegitimate children should be awarded to them, being the legitimate heirs of Loreto entitled to their respective legitimes.

It is evident from the face of the complaint that petitioners are not entitled to a favorable judgment in light of Article 2011 of the Civil Code which expressly provides that insurance contracts shall be governed by special laws, i.e., the Insurance Code. Section 53 of the Insurance Code states—

SECTION 53. The insurance proceeds shall be applied exclusively to the proper interest of the person in whose name or for whose benefit it is made unless otherwise specified in the policy.

Page 24: Insurance Case Digests (1)

Insurance Case Digests Atty. Bathan-Basuel

2-S, 2013-2014 24

Pursuant thereto, it is obvious that the only persons entitled to claim the insurance proceeds are either the insured, if still alive; or the beneficiary, if the insured is already deceased, upon the maturation of the policy.20 The exception to this rule is a situation where the insurance contract was intended to benefit third persons who are not parties to the same in the form of favorable stipulations or indemnity. In such a case, third parties may directly sue and claim from the insurer.21

Petitioners are third parties to the insurance contracts with Insular and Grepalife and, thus, are not entitled to the proceeds thereof. Accordingly, respondents Insular and Grepalife have no legal obligation to turn over the insurance proceeds to petitioners. The revocation of Eva as a beneficiary in one policy and her disqualification as such in another are of no moment considering that the designation of the illegitimate children as beneficiaries in Loreto’s insurance policies remains valid. Because no legal proscription exists in naming as beneficiaries the children of illicit relationships by the insured,22the shares of Eva in the insurance proceeds, whether forfeited by the court in view of the prohibition on donations under Article 739 of the Civil Code or by the insurers themselves for reasons based on the insurance contracts, must be awarded to the said illegitimate children, the designated beneficiaries, to the exclusion of petitioners. It is only in cases where the insured has not designated any beneficiary,23 or when the designated beneficiary is disqualified by law to receive the proceeds,24 that the insurance policy proceeds shall redound to the benefit of the estate of the insured.

Violeta Lalican v. The Insular Life Assurance Company, Limited, G.R. No. 183526, August 25, 2009

LALICAN VS. INSULAR LIFE ASSURANCE COMPANY LIMITED G.R. No. 183526 August 25, 2009 Chico-Nazario, J.; FACTS: Violeta is the widow of the deceased Eulogio C. Lalican (Eulogio). During his lifetime, Eulogio applied for an insurance policy with Insular Life. On 24 April 1997, Insular Life, through Josephine Malaluan (Malaluan), its agent in Gapan City, issued in favor of Eulogio Policy No. 9011992, which contained a 20-Year Endowment Variable Income Package Flexi Plan worth P500,000.00, with two riders valued at P500,000.00 each. Thus, the value of the policy amounted to P1,500,000.00. Violeta was named as the primary beneficiary.

Under the terms of Policy No. 9011992, Eulogio was to pay the premiums on a quarterly basis in the amount of P8,062.00, payable every 24 April, 24 July, 24 October and 24 January of each year, until the end of the 20-year period of the policy. According to the Policy Contract, there was a grace period of 31 days for the payment of each premium subsequent to the first. If any premium was not paid on or before the due date, the policy would be in default, and if the premium remained unpaid until the end of the grace period, the policy would automatically lapse and become void.

Eulogio paid the premiums due on 24 July 1997 and 24 October 1997.

However, he failed to pay the premium due on 24 January 1998, even after the lapse of the grace period of 31 days. Policy No. 9011992, therefore, lapsed and became void.

Eulogio submitted to the Cabanatuan District Office of Insular Life, through Malaluan, on 26 May 1998, an Application for Reinstatement of Policy No. 9011992, together with the amount of P8,062.00 to pay for the premium due on 24 January 1998. In a letter dated 17 July 1998, Insular Life notified Eulogio that his Application for Reinstatement could not be fully processed because, although he already deposited P8,062.00 as payment for the 24 January 1998 premium, he left unpaid the overdue interest thereon amounting to P322.48. Thus, Insular Life instructed Eulogio to pay the amount of interest and to file another application for reinstatement. Eulogio was likewise advised by Malaluan to pay the premiums that subsequently became due on 24 April 1998 and 24 July 1998, plus interest.

On 17 September 1998, Eulogio went to Malaluan’s house and submitted a second Application for Reinstatement of Policy No. 9011992, including the amount of P17,500.00, representing payments for the overdue interest on the premium for 24 January 1998, and the premiums which became due on 24 April 1998 and 24 July 1998. As Malaluan was away on a business errand, her husband received Eulogio’s second Application for Reinstatement and issued a receipt for the amount Eulogio deposited.

A while later, on the same day, 17 September 1998, Eulogio died of cardio-respiratory arrest secondary to electrocution.

Without knowing of Eulogio’s death, Malaluan forwarded to the Insular Life Regional Office in the City of San Fernando, on 18 September 1998, Eulogio’s second Application for Reinstatement of Policy No. 9011992 and P17,500.00 deposit. However, Insular Life no longer acted upon Eulogio’s second Application for Reinstatement, as the former was informed on 21 September 1998 that Eulogio had already passed away.

On 28 September 1998, Violeta filed with Insular Life a claim for payment of the full proceeds of Policy No. 9011992.

Without waiting for the result of the re-evaluation by Insular Life, Violeta filed with the RTC, on 11 October 1999, a Complaint for Death Claim Benefit, which was docketed as Civil Case No. 2177. Violeta alleged that Insular Life engaged in unfair claim settlement practice and deliberately failed to act with reasonable promptness on her insurance claim. Violeta prayed that Insular Life be ordered to pay her death claim benefits on Policy No. 9011992, in the amount of P1,500,000.00, plus interests, attorney’s fees, and cost of suit. After trial, the RTC rendered, on 30 August 2007, a decision in favor of Insular Life.

Page 25: Insurance Case Digests (1)

Insurance Case Digests Atty. Bathan-Basuel

2-S, 2013-2014 25

Violeta then appealed directly to the Supreme Court alleging a question of law arising in this case involving the interpretation of the second sentence of Section 19 of the Insurance Code, which provides:

“Interest in the life or health of a person insured must exist when the insurance takes effect, but need not exist thereafter or when the loss occurs.”

ISSUE: Whether or not Eulogio still had insurable interest in his own life when he reinstated Policy No. 9011992 just before he passed away on September 17, 1998 HELD: An insurable interest is one of the most basic and essential requirements in an insurance contract. In general, an insurable interest is that interest which a person is deemed to have in the subject matter insured, where he has a relation or connection with or concern in it, such that the person will derive pecuniary benefit or advantage from the preservation of the subject matter insured and will suffer pecuniary loss or damage from its destruction, termination, or injury by the happening of the event insured against. The existence of an insurable interest gives a person the legal right to insure the subject matter of the policy of insurance. Section 10 of the Insurance Code indeed provides that every person has an insurable interest in his own life. Section 19 of the same code also states that an interest in the life or health of a person insured must exist when the insurance takes effect, but need not exist thereafter or when the loss occurs. Upon more extensive study of the Petition, it becomes evident that the matter of insurable interest is entirely irrelevant in the case at bar. It is actually beyond question that while Eulogio was still alive, he had an insurable interest in his own life, which he did insure under Policy No. 9011992.

Gaisano Cagayan, Inc. v. Insurance Company of North America, G.R. No. 147839, June 8, 2006

FACTS:

- Intercapitol Marketing Corporation (IMC) is the maker of Wrangler Blue Jeans. while Levi Strauss (Phils.) Inc. (LSPI) is the local distributor of products bearing trademarks owned by Levi Strauss & Co

- IMC and LSPI separately obtained from Insurance Company of North America fire insurance policies for their book debt endorsements related to their ready-made clothing materials which have been sold or delivered to various customers and dealers of the Insured anywhere in the Philippines which are unpaid 45 days after the time of the loss

- February 25, 1991: Gaisano Superstore Complex in Cagayan de Oro City, owned by Gaisano Cagayan, Inc., containing the ready-made clothing materials sold and delivered by IMC and LSPI was consumed by fire.

- February 4, 1992: Insurance Company of North America filed a complaint for damages against Gaisano Cagayan, Inc. alleges that IMC and LSPI filed their claims under their respective fire insurance policies which it paid thus it was

subrogated to their rights o Gaisano Cagayan, Inc: not be held liable because it was destroyed

due to fortuities event or force majeure - RTC: IMC and LSPI retained ownership of the delivered goods until fully paid, it

must bear the loss (res perit domino) - CA: Reversed - sales invoices is an exception under Article 1504 (1) of the

Civil Code to res perit domino ISSUE: W/N Insurance Company of North America can claim against Gaisano Cagayan for the debt that was isnured HELD: YES. petition is partly GRANTED. order to pay P535,613 is DELETED • insurance policy is clear that the subject of the insurance is the book debts

and NOT goods sold and delivered to the customers and dealers of the insured

• ART. 1504. Unless otherwise agreed, the goods remain at the seller's risk until the ownership therein is transferred to the buyer, but when the ownership therein is transferred to the buyer the goods are at the buyer's risk whether actual delivery has been made or not, except that:

• (1) Where delivery of the goods has been made to the buyer or to a bailee for the buyer, in pursuance of the contract and the ownership in the goods has been retained by the seller merely to secure performance by the buyer of his obligations under the contract, the goods are at the buyer's risk from the time of such delivery;

• IMC and LSPI did not lose complete interest over the goods. They have an insurable interest until full payment of the value of the delivered goods. Unlike the civil law concept of res perit domino, where ownership is the basis for consideration of who bears the risk of loss, in property insurance, one's interest is not determined by concept of title, but whether insured has substantial economic interest in the property

• Section 13 of our Insurance Code defines insurable interest as "every interest in property, whether real or personal, or any relation thereto, or liability in respect thereof, of such nature that a contemplated peril might directly damnify the insured." Parenthetically, under Section 14 of the same Code, an insurable interest in property may consist in: (a) an existing interest; (b) an inchoate interest founded on existing interest; or (c) an expectancy, coupled with an existing interest in that out of which the expectancy arises.

• Anyone has an insurable interest in property who derives a benefit from its existence or would suffer loss from its destruction. it is sufficient that the insured is so situated with reference to the

property that he would be liable to loss should it be injured or destroyed by the peril against which it is insured an insurable interest in property does not necessarily imply a property interest in, or a lien upon, or possession of, the subject matter of the insurance, and neither the title nor a beneficial interest is requisite to the existence of such an interest

- insurance in this case is not for loss of goods by fire but for

Page 26: Insurance Case Digests (1)

Insurance Case Digests Atty. Bathan-Basuel

2-S, 2013-2014 26

petitioner's accounts with IMC and LSPI that remained unpaid 45 days after the fire - obligation is pecuniary in nature obligor should be held exempt from liability when the loss occurs thru a fortuitous event only holds true when the obligation consists in the delivery of a determinate thing and there is no stipulation holding him liable even in case of fortuitous event

- Article 1263 of the Civil Code in an obligation to deliver a generic thing, the loss or destruction of anything of the same kind does not extinguish the obligation (Genus nunquan perit)

- The subrogation receipt, by itself, is sufficient to establish not only the relationship of respondent as insurer and IMC as the insured, but also the amount paid to settle the insurance claim

- Art. 2207. If the plaintiff's property has been insured, and he has received indemnity from the insurance company for the injury or loss arising out of the wrong or breach of contract complained of, the insurance company shall be subrogated to the rights of the insured against the wrongdoer or the person who has violated the contract.

o As to LSPI, no subrogation receipt was offered in evidence. o Failure to substantiate the claim of subrogation is fatal to petitioner's

case for recovery of the amount of P535,613

Garcia vs. The Hongkong Fire & Marine Insurance Co., Ltd, G.R. No. 20341, September 1, 1923

G.R. No. 20341; September 1, 1923 DOMINGO GARCIA and PNB, plaintiffs-appellees vs. THE HONGKONG FIRE & MARINE INSURANCE CO., LTD., defendant-appellant FACTS: In 1918, in Manila, Domingo Garcia, merchant-owner of a bazaar known as "Las Novedades" in Legaspi, Albay, entered into a contract with Hongkong Fire to insure his merchandise for P15,000 at a premium of P300 per annum. The latter issued fire insurance policy No. 1951, not on the merchandise but on the building which contained it. The policy does not contain the true agreement and intent of the parties. Garcia was not the owner of the building. In 1919, Garcia executed a mortgage to PNB on the merchandise to secure a loan. The bank advised Hongkong Fire of the same. In 1920, and while the policy was in force and effect, a fire took place which destroyed the merchandise valued at P20,000, together with the building itself. Garcia demanded P15,000, as provided, but payment was refused. The lower court decided in favor of Garcia. ISSUE: Whether Garcia is entitled to the insurance. HELD: YES. The policy was in English, of which Garcia is ignorant. He knew that P15,000 was correct, and never investigated. When Hongkong Fire surrendered the policy to PNB, the word "merchandise" was written in the letter. It is contended that when the letter was written, the Bank, which then had the possession of the policy, knew that it covered the building and did not insure the merchandise.

Garcia had his dealings with the officials of the branch Bank at Legaspi where he was doing business as a merchant, of which the officials of that Bank had knowledge. Under such facts, the presumption of knowledge, if any, on the part of the Bank would be that the policy was on the merchandise. Be that as it may, when the defendant received the letter from the Bank, it knew from its own records that the policy was issued on the building, and, as a matter of fair dealing, it should have notified the Bank that the policy was on the building. In the final analysis, Garcia wanted insurance upon a stock of goods, which he owned, and he received and paid for a policy on a building, which he did not own, and while the policy was in force and effect, both the building, which he did not own, and the stock of merchandise, which he did own, were completely destroyed by fire. The defense is purely technical, and is founded upon the contention that Garcia cannot recover, because the policy covers loss on a building, and does not cover loss of merchandise. It is very apparent that a mistake was made in the issuance of the policy. In its opinion the trial court says: Under these circumstances it seems clear and manifest that all have been in the belief that it was not the building but the merchandise that was insured, for the reason that none of them paid attention to the context of the policy. The opinion of the trial court further points out that, under the pleadings and proof, there is ground for the contention that the plaintiff would be entitled to recover on the policy for the loss of the building. Lower court decision is affirmed.

Rizal Commercial Banking Corporation (RCBC) vs. Court of Appeals GR 128833, 20 April 1998

Rizal Commercial Banking Corporation (RCBC) vs. Court of Appeals [GR 128833, 20 April 1998]; Facts:

Goyu & Sons, Inc. (Goyu) applied for credit facilities and accommodations with Rizal Commercial Banking Corporation (RCBC) at its Binondo Branch. After due evaluation, RCBC Binondo Branch, through its key officers, petitioners Uy Chun Bing and Eli D. Lao, recommended Goyu's application for approval by RCBC's executive committee. A credit facility in the amount of P30 million was initially granted. Upon Goyu's application and Uy's and Lao's recommendation, RCBC's executive committee increased Goyu's credit facility to P50 million, then to P90 million, and finally to P117 million.

As security for its credit facilities with RCBC, Goyu executed two real estate mortgages and two chattel mortgages in favor of RCBC, which were registered with the Registry of Deeds at Valenzuela, Metro Manila. Under each of these four mortgage contracts, Goyu committed itself to insure the mortgaged property with an insurance company approved by RCBC, and subsequently, to endorse and deliver the insurance policies to RCBC. Goyu obtained in its name a total of 10 insurance policies from MICO.

Page 27: Insurance Case Digests (1)

Insurance Case Digests Atty. Bathan-Basuel

2-S, 2013-2014 27

In February 1992, Alchester Insurance Agency, Inc., the insurance agent where Goyu obtained the Malayan insurance policies, issued 9 endorsements in favor of RCBC seemingly upon instructions of Goyu. On 27 April 1992, one of Goyu's factory buildings in Valenzuela was gutted by fire. Consequently, Goyu submitted its claim for indemnity on account of the loss insured against. MICO denied the claim on the ground that the insurance policies were either attached pursuant to writs of attachments/garnishments issued by various courts or that the insurance proceeds were also claimed by other creditors of Goyu alleging better rights to the proceeds than the insured.

Goyu filed a complaint for specific performance and damages which was docketed at the Regional Trial Court of the National Capital Judicial Region (Manila, Branch 3) as Civil Case 93-65442. RCBC, one of Goyu's creditors, also filed with MICO its formal claim over the proceeds of the insurance policies, but said claims were also denied for the same reasons that AGCO denied Goyu's claims. In an interlocutory order dated 12 October 1993, the Regional Trial Court of Manila (Branch 3), confirmed that Goyu's other creditors, namely, Urban Bank, Alfredo Sebastian, and Philippine Trust Company obtained their respective writs of attachments from various courts, covering an aggregate amount of P14,938,080.23, and ordered that the proceeds of the 10 insurance policies be deposited with the said court minus the aforementioned P14,938,080.23. Accordingly, on 7 January 1994, MICO deposited the amount of P50,505,594.60 with Branch 3 of the Manila RTC.

In the meantime, another notice of garnishment was handed down by another Manila RTC sala (Branch 28) for the amount of P8,696,838.75. After trial, Branch 3 of the Manila RTC rendered judgment in a favor of Goyu, ordering Malayan to pay Goyu its fire loss claims in the total amount of P74,040,518.58 less the amount of P50,000,000.00 which is deposited with the Court; damages by way of interest for the duration of the delay since 27 July 1992 (90 days after Malayan's receipt of the required proof of loss and notice of loss) at the rate of twice the ceiling prescribed by the Monetary Board, on the amounts of (1) P50,000,000.00 from 27 July 1992 up to the time said amount was deposited with the Court on 7 January 1994; and (2) P24,040,518.58 — from 17 July 1992 up to the time when the writs of attachments were received by Malayan. The court also ordered RCBC to pay Goyu actual and compensatory damages in the amount of P2,000,000.00, and both Malayan and RCBC to solidarily pay Goyu (1) P1,000,000.00 as exemplary damages; (2) P1,000,000.00 as, and for, attorneys fees; and (3) Costs of suit. The Court, on the Counterclaim of RCBC, ordered Goyu to pay its loan obligations with RCBC in the amount of P68,785,069.04, as of 27 April 1992, with interest thereon at the rate stipulated in the respective promissory notes (without surcharges and penalties).

From this judgment, all parties interposed their respective appeals. Issue: Whether RCBC, as mortgagee, has any right over the insurance policies taken by Goyu, the mortgagor, in case of the occurrence of loss Held: YES. It is settled that a mortgagor and a mortgagee have separate and distinct insurable interests in the same mortgaged property, such that each one of them may insure the same property for his own sole benefit. There is no question that Goyu could insure the mortgaged property for its own exclusive benefit. Herein, although it appears

that Goyu obtained the subject insurance policies naming itself as the sole payee, the intentions of the parties as shown by their contemporaneous acts, must be given due consideration in order to better serve the interest of justice and equity. It is to be noted that nine endorsement documents were prepared by Alchester in favor of RCBC. The Court is in a quandary how Alchester could arrive at the idea of endorsing any specific insurance policy in favor of any particular beneficiary or payee other than the insured had not such named payee or beneficiary been specifically disclosed by the insured itself.

It is also significant that Goyu voluntarily and purposely took the insurance policies from MICO, a sister company of RCBC, and not just from any other insurance company. Alchester would not have found out that the subject pieces of property were mortgaged to RCBC had not such information been voluntarily disclosed by Goyu itself. Had it not been for Goyu, Alchester would not have known of Goyu's intention of obtaining insurance coverage in compliance with its undertaking in the mortgage contracts with RCBC, and verify, Alchester would not have endorsed the policies to RCBC had it not been so directed by Goyu. On equitable principles, particularly on the ground of estoppel, the Court is constrained to rule in favor of mortgagor RCBC.

RCBC, in good faith, relied upon the endorsement documents sent to it as this was only pursuant to the stipulation in the mortgage contracts. Such reliance is justified under the circumstances of the case. Goyu failed to seasonably repudiate the authority of the person or persons who prepared such endorsements. Over and above this, Goyu continued, in the meantime, to enjoy the benefits of the credit facilities extended to it by RCBC. After the occurrence of the loss insured against, it was too late for Goyu to disown the endorsements for any imagined or contrived lack of authority of Alchester to prepare and issue said endorsements. If there had not been actually an implied ratification of said endorsements by virtue of Goyu's inaction in this case, Goyu is at the very least estopped from assailing their operative effects.

To permit Goyu to capitalize on its non- confirmation of these endorsements while it continued to enjoy the benefits of the credit facilities of RCBC which believed in good faith that there was due endorsement pursuant to their mortgage contracts, is to countenance grave contravention of public policy, fair dealing, good faith, and justice. Such an unjust situation, the Court cannot sanction. Under the peculiar circumstances, the Court is bound to recognize RCBC's right to the proceeds of the insurance policies if not for the actual endorsement of the policies, at least on the basis of the equitable principle of estoppel.

Filipino Merchants Insurance Co. Inc. vs. Court of Appeals GR 85141, 28, November

1989 Fil ipino Merchants Insurance Co. Inc. vs. Court of Appeals GR 85141, 28, November 1989 FACTS: Choa insured with Filipino Merchants the shipment of supposedly 600 metric tons of fishmeal from Bangkok, Thailand to Manila against all risks under warehouse to warehouse terms. However, what was imported was only 59.940 metric tons at

Page 28: Insurance Case Digests (1)

Insurance Case Digests Atty. Bathan-Basuel

2-S, 2013-2014 28

$395.42 per ton. The fishmeal in 666 new gunny bags arrived in Manila and were unloaded from the ship SS Bougainville. Upon receipt, the arrastre contractor E. Razon and Filipino Merchant's surveyor certified that 227 bags were in bad order condition amounting to 12,148 kilos. Choa made a formal claim against Filipino Merchant for P51,568.63 (amount of loss). However, Filipino Merchant refused to pay the claim. Consequently, Choa filed an action against Filipino Merchant seeking to recover P51,568.62 representing damages to said shipment which has been insured by the defendant insurance company. RTC Manila rendered judgment in favor of Choa. On appeal, CA affirmed the decision of RTC. ISSUE:

1. WON Filipino Merchant may be held liable under the “all risks” clause of the marine insurance policy in the absence of proof of some fortuitous event, casualty, or accidental cause to which the loss is attributable.

2. WON the marine insurance policy taken out by Choa is null and void considering that he had no insurable interest in the subject cargo.

HELD: 1. The SC ruled that Filipino Merchant is liable under the “all risks” clause of the marine insurance policy it had with Choa. Filipino Merchant contended that an "all risks" marine policy has a technical meaning in insurance in that before a claim can be compensable it is essential that there must be "some fortuity, " "casualty" or "accidental cause" to which the alleged loss is attributable and the failure of herein private respondent, upon whom lay the burden, to adduce evidence showing that the alleged loss to the cargo in question was due to a fortuitous event precludes his right to recover from the insurance policy. The SC ruled in the negative. It explained that an "all risks policy" should be read literally as meaning all risks whatsoever and covering all losses by an accidental cause of any kind. The terms "accident" and "accidental", as used in insurance contracts, have not acquired any technical meaning. They are construed by the courts in their ordinary and common acceptance. The term "all risks" cannot be given a strained technical meaning, the language of the clause under the Institute Cargo Clauses being unequivocal and clear, to the effect that it extends to all damages/losses suffered by the insured cargo except (a) loss or damage or expense proximately caused by delay, and (b) loss or damage or expense proximately caused by the inherent vice or nature of the subject matter insured. Generally, the burden of proof is upon the insured to show that a loss arose from a covered peril, but under an "all risks" policy the burden is not on the insured to prove the precise cause of loss or damage for which it seeks compensation. The insured under an "all risks insurance policy" has the initial burden of proving that the cargo was in good condition

when the policy attached and that the cargo was damaged when unloaded from the vessel; thereafter, the burden then shifts to the insurer to show the exception to the coverage. In the present case, there being no showing that the loss was caused by any of the excepted perils, the insurer is liable under the policy. There is no evidence presented to show that the condition of the gunny bags in which the fishmeal was packed was such that they could not hold their contents in the course of the necessary transit, much less any evidence that the bags of cargo had burst as the result of the weakness of the bags themselves. Had there been such a showing that spillage would have been a certainty, there may have been good reason to plead that there was no risk covered by the policy. 2. Choa as consignee of the goods in transit under an invoice containing the terms under "C & F Manila," has insurable interest in said goods. Choa as vendee/consignee of the goods in transit has such existing interest therein as may be the subject of a valid contract of insurance. His interest over the goods is based on the perfected contract of sale. The perfected contract of sale between him and the shipper of the goods operates to vest in him an equitable title even before delivery or before be performed the conditions of the sale. The contract of shipment, whether under F.O.B., C.I.F., or C. & F. as in this case, is immaterial in the determination of whether the vendee has an insurable interest or not in the goods in transit. The perfected contract of sale even without delivery vests in the vendee an equitable title, an existing interest over the goods sufficient to be the subject of insurance. Also pursuant to Article 1523 of the NCC, the delivery of the goods on board the carrying vessels partake of the nature of actual delivery since, from that time, the foreign buyers assumed the risks of loss of the goods and paid the insurance premium covering them. 4. Concealment and Representation

Ma. Lourdes S. Florendo v. Philam Plans, Inc., Perla Abcede and Ma. Celeste Abcede,

G.R. No. 186983, February 22, 2012 (1) G.R. No. 186983 February 22, 2012 MA. LOURDES S. FLORENDO, Petitioner, vs. PHILAM PLANS, INC., PERLA ABCEDE MA. CELESTE ABCEDE, Respondents. ABAD, J.:

FACTS: On October 23, 1997 Manuel Florendo filed an application for comprehensive pension plan with respondent Philam Plans, Inc. (Philam Plans) after some convincing by respondent Perla Abcede. The plan had a pre-need price of P997,050.00, payable in 10 years, and had a maturity value

Page 29: Insurance Case Digests (1)

Insurance Case Digests Atty. Bathan-Basuel

2-S, 2013-2014 29

of P2,890,000.00 after 20 years.1 Manuel signed the application and left to Perla the task of supplying the information needed in the application.2Respondent Ma. Celeste Abcede, Perla’s daughter, signed the application as sales counselor.3

The comprehensive pension plan also provided life insurance coverage to Florendo.4This was covered by a Group Master Policy that Philam Life issued to Philam Plans.5 Under the master policy, Philam Life was to automatically provide life insurance coverage, including accidental death, to all who signed up for Philam Plans’ comprehensive pension plan.6 If the plan holder died before the maturity of the plan, his beneficiary was to instead receive the proceeds of the life insurance, equivalent to the pre-need price. Further, the life insurance was to take care of any unpaid premium until the pension plan matured, entitling the beneficiary to the maturity value of the pension plan.7

On October 30, 1997 Philam Plans issued Pension Plan Agreement to Manuel, with petitioner Ma. Lourdes S. Florendo, his wife, as beneficiary. In time, Manuel paid his quarterly premiums.9

Eleven months later or on September 15, 1998, Manuel died of blood poisoning. Subsequently, Lourdes filed a claim with Philam Plans for the payment of the benefits under her husband’s plan.10 Because Manuel died before his pension plan matured and his wife was to get only the benefits of his life insurance, Philam Plans forwarded her claim to Philam Life.11

On May 3, 1999 Philam Plans wrote Lourdes declining her claim. Philam Life found that Manuel was on maintenance medicine for his heart and had an implanted pacemaker. Further, he suffered from diabetes mell itus and was taking insulin. Lourdes renewed her demand for payment under the plan13 but Philam Plans rejected it,14prompting her to file the present action against the pension plan company before the RTC of QC.15

On March 30, 2006 the RTC ruled that Manuel was not guilty of concealing the state of his health from his pension plan application; ordered Philam Plans, Perla and Ma. Celeste, solidari ly, to pay Lourdes all the benefits from her husband’s pension plan, namely: P997,050.00, the proceeds of his term insurance, and P2,890,000.00 lump sum pension benefit upon maturity of his plan; P100,000.00 as moral damages; and to pay the costs of the suit.

On December 18, 2007, CA reversed the RTC decision,17 holding that insurance policies are traditionally contracts uberrimae fidae or contracts of utmost good faith. It required Manuel to disclose conditions affecting the risk of which he was aware or material facts that he knew or ought to know.18

ISSUES: (1) Whether Manuel is guilty of concealing his illness when he kept blank and did not answer questions in his pension plan application.

(2) Whether Manuel was bound by the failure of Perla and Ma. Celeste to declare the condition of Manuel’s health. (3) Whether Philam Plans’ approval of Manuel’s pension plan application and acceptance of his premium payments precluded it from denying Lourdes’ claim.

HELD: (1) YES. Lourdes contends that Philam Plans should have returned the application to him for completion. Since it approved the application just as it was, it cannot cry concealment on Manuel’s part, that Philam Plans never queried Manuel directly regarding the state of his health.

Since Philam Plans waived medical examination for Manuel, it had to rely largely on his stating the truth regarding his health in his application. He knew more than anyone that he had been under treatment for heart condition and diabetes for more than five years preceding his application. But he kept those crucial facts from Philam Plans.

When Manuel signed the application, he adopted as his own the written representations and declarations embodied in it. It is clear from these representations that he concealed his chronic heart ailment and diabetes from Philam Plans. The pertinent portion of his representations and declarations read as follows:

(c) I have never been treated for heart condition, high blood pressure, cancer, diabetes, lung, kidney or stomach disorder or any other physical impairment in the last five years. (d) I am in good health and physical condition.

Manuel signed the application without filling in the details regarding his continuing treatments for heart condition and diabetes. The assumption is that he has never been treated for the said illnesses in the last five years preceding his application.

Lourdes insists that Perla, the soliciting agent, knew that Manuel had a pacemaker before he signed up for the pension plan.23 But by its tenor, the responsibil ity for preparing the application belonged to Manuel. Nothing in it implies that someone else may provide the information that Philam Plans needed. Manuel cannot sign the application and disown the responsibil ity for having it fi l led up. If he furnished Perla the needed information and delegated to her the filling up of the application, then she acted on his instruction, not on Philam Plans’ instruction. Manuel still had his pacemaker when he applied for a pension plan and it is an admission that he remained under treatment for irregular heartbeat within five years preceding that application.

Manuel had been taking medicine when he submitted his pension plan application. These clearly fell within the five-year period. It is not claimed that Perla was aware of his two other afflictions that needed medical treatments. Pursuant to Section 27 of IC, Manuel’s concealment entitles Philam Plans to rescind its contract of insurance with him.

Page 30: Insurance Case Digests (1)

Insurance Case Digests Atty. Bathan-Basuel

2-S, 2013-2014 30

(2) Lourdes contends that the mere fact that Manuel signed the application in blank and let Perla fill in the details did not make her his agent and bind him to her concealment of his true state of health. There is no evidence of collusion between them.

Manuel, in signing the pension plan application, he certified that he wrote all the information stated in it or had someone do it under his direction. Assuming that it was Perla who filled up the application form, Manuel is still bound by what it contains since he certified that he authorized her action. Philam Plans had every right to act on the faith of that certification. Manuel was made aware when he signed the pension plan application that, in granting the same, Philam Plans and Philam Life were acting on the truth of the representations contained in that application.

Manuel, a civil engineer and manager of a construction company, could be expected to know that one must read every document, especially if it creates rights and obligations affecting him, before signing the same. It could reasonably be expected that he would not trifle with something that would provide additional financial security to him and to his wife in his twilight years.

(3) Lourdes contends that any defect or insufficiency in the information provided by his pension plan application should be deemed waived after the same has been approved, the policy has been issued, and the premiums have been collected. 34

The Court cannot agree. The comprehensive pension plan that Philam Plans issued contains a one-year incontestabil ity period.

The incontestabil ity clause precludes the insurer from disowning liability under the policy it issued on the ground of concealment or misrepresentation regarding the health of the insured after a year of its issuance.

Since Manuel died on the eleventh month following the issuance of his plan,36 the one year incontestabil ity period has not yet set in.

CA decision AFFIRMED.

Vda. De Canilang v. CA, 223 SCRA 443 (1993) Facts:

- Canilang consulted Dr. Claudio and was diagnosed as suffering from "sinus tachycardia." Mr. Canilang consulted the same doctor again on 3 August 1982 and this time was found to have "acute bronchitis."

- On the next day, 4 August 1982, Canilang applied for a "non-medical" insurance policy with Grepalife naming his wife, as his beneficiary. Canilang was issued ordinary life insurance with the face value of P19,700.

- On 5 August 1983, Canilang died of "congestive heart failure," "anemia," and

"chronic anemia." The wife as beneficiary, filed a claim with Grepalife which the insurer denied on the ground that the insured had concealed material information from it.

- Vda Canilang filed a complaint with the Insurance Commissioner against Grepalife contending that as far as she knows her husband was not suffering from any disorder and that he died of kidney disorder.

- Grepalife was ordered to pay the widow by the Insurance Commissioner holding that there was no intentional concealment on the Part of Canilang and that Grepalife had waived its right to inquire into the health condition of the applicant by the issuance of the policy despite the lack of answers to "some of the pertinent questions" in the insurance application. CA reversed.

Issue: Whether or not Grepalife is liable. Held: SC took note of the fact that Canilang failed to disclose that hat he had twice consulted Dr. Wilfredo B. Claudio who had found him to be suffering from "sinus tachycardia" and "acute bronchitis. Under the relevant provisions of the Insurance Code, the information concealed must be information which the concealing party knew and "ought to [have] communicate[d]," that is to say, information which was "material to the contract. The information which Canilang failed to disclose was material to the ability of Grepalife to estimate the probable risk he presented as a subject of life insurance. Had Canilang disclosed his visits to his doctor, the diagnosis made and the medicines prescribed by such doctor, in the insurance application, it may be reasonably assumed that Grepalife would have made further inquiries and would have probably refused to issue a non-medical insurance policy or, at the very least, required a higher premium for the same coverage. The materiality of the information withheld by Canilang from Grepalife did not depend upon the state of mind of Jaime Canilang. A man's state of mind or subjective belief is not capable of proof in our judicial process, except through proof of external acts or failure to act from which inferences as to his subjective belief may be reasonably drawn. Neither does materiality depend upon the actual or physical events which ensue. Materiality relates rather to the "probable and reasonable influence of the facts" upon the party to whom the communication should have been made, in assessing the risk involved in making or omitting to make further inquiries and in accepting the application for insurance; that "probable and reasonable influence of the facts" concealed must, of course, be determined objectively, by the judge ultimately. SC found it difficult to take seriously the argument that Grepalife had waived inquiry into the concealment by issuing the insurance policy notwithstanding Canilang's failure to set out answers to some of the questions in the insurance application. Such failure precisely constituted concealment on the part of Canilang. Petitioner's argument, if accepted, would obviously erase Section 27 from the Insurance Code of 1978.

Ng Gan Zee v. Asian Crusader Life, 122 SCRA 461

Page 31: Insurance Case Digests (1)

Insurance Case Digests Atty. Bathan-Basuel

2-S, 2013-2014 31

NG GAN ZEE vs. ASIAN CRUSADER LIFE ASSURANCE CORPORATION

Facts:

- Kwong Nam applied for a 20-year endowment insurance on his life for the sum of P20,000.00, with his wife, Ng Gan Zee as beneficiary. On the same date, ACAC, upon receipt of the required premium from the insured, approved the application and issued the corresponding policy.

- Kwong Nam died of cancer of the liver with metastasis. All premiums had been religiously paid at the time of his death.

- Kwong Nam widow Ng Gan Zee presented a claim in due form to appellant for payment of the face value of the policy. She submitted the required proof of death of the insured. ACAC denied the claim on the ground that the answers given by the insured to the questions appealing in his application for life insurance were untrue.

- Ng Gan Zee brought the matter to the attention of the Insurance Commissioner, the Hon. Francisco Y. Mandamus, and the latter, after conducting an investigation, wrote the ACAC that he had found no material concealment on the part of the insured and that, therefore, Ng Gan Zee should be paid the full face value of the policy. This opinion of the Insurance Commissioner notwithstanding, ACAC refused to settle its obligation.

- ACAC alleged that the insured was guilty of misrepresentation when he answered "No" to the following question appearing in the application for life insurance:

Has any l ife insurance company ever refused your application for insurance or for reinstatement of a lapsed policy or offered you a policy different from that applied for? If, so, name company and date.

According to ACAC

"The insured applied for reinstatement of his lapsed life insurance policy with the Insular Life Insurance Co., Ltd, but this was declined by the insurance company, although later on approved for reinstatement with a very high premium as a result of his medical examination. Thus notwithstanding the said insured answered 'No' to the question propounded to him"

Issue: Whether Ng Gan Zee can collect the insurance claim.

CFI: ordered ACAC to pay the face value of the insurance policy

Held:

- There is no evidence that the Insular Life Assurance Co., Ltd. ever refused any application of Kwong Nam for insurance. Neither is there any evidence that any

other insurance company has refused any application of Kwong Nam for insurance. The evidence shows that the Insular Life Assurance Co., Ltd. approved Kwong Nam's request for reinstatement and amendment of his lapsed insurance policy. The Court notes from said application for reinstatement and amendment, that the amount applied for was P20,000.00 only and not for P50,000.00 as it was in the lapsed policy. The amount of the reinstated and amended policy was also for P20,000.00. It results, therefore, that when Kwong Nam answered 'No' to the question whether any life insurance company ever refused his application for reinstatement of a lapsed policy he did not misrepresent any fact.

- Sec. 27. Such party a contract of insurance must communicate to the other, in good faith, all facts within his knowledge which are material to the contract, and which the other has not the means of ascertaining, and as to which he makes no warranty.

- Thus, "concealment exists where the assured had knowledge of a fact material to the risk, and honesty, good faith, and fair dealing requires that he should communicate it to the assurer, but he designedly and intentionally withholds the same." It has also been held "that the concealment must, in the absence of inquiries, be not only material, but fraudulent, or the fact must have been intentionally withheld."

- Assuming that the aforesaid answer given by the insured is false, as claimed by the ACAC. Sec. 27 of the Insurance Law, nevertheless requires that fraudulent intent on the part of the insured be established to entitle the insurer to rescind the contract. And as correctly observed by the lower court, "misrepresentation as a defense of the insurer to avoid liability is an 'affirmative' defense. The duty to establish such a defense by satisfactory and convincing evidence rests upon the defendant. The evidence before the Court does not clearly and satisfactorily establish that defense."

- It bears emphasis that Kwong Nam had informed the ACAC's medical examiner that the tumor for which he was operated on was "associated with ulcer of the stomach." In the absence of evidence that the insured had sufficient medical knowledge as to enable him to distinguish between "peptic ulcer" and "a tumor", his statement that said tumor was "associated with ulcer of the stomach, " should be construed as an expression made in good faith of his belief as to the nature of his ailment and operation. Indeed, such statement must be presumed to have been made by him without knowledge of its incorrectness and without any deliberate intent on his part to mislead the appellant.

- While it may be conceded that, from the viewpoint of a medical expert, the information communicated was imperfect, the same was nevertheless sufficient to have induced appellant to make further inquiries about the ailment and operation of the insured.

- Section 32. The right to information of material facts maybe waived either by the terms of insurance or by neglect to make inquiries as to such facts where they are distinctly implied in other facts of which information is communicated

- It has been held that where, upon the face of the application, a question appears to be not answered at all or to be imperfectly answered, and the

Page 32: Insurance Case Digests (1)

Insurance Case Digests Atty. Bathan-Basuel

2-S, 2013-2014 32

insurers issue a policy without any further inquiry, they waive the imperfection of the answer and render the omission to answer more fully immaterial.

- As aptly noted by the lower court, "if the ailment and operation of Kwong Nam had such an important bearing on the question of whether the defendant would undertake the insurance or not, the court cannot understand why the defendant or its medical examiner did not make any further inquiries on such matters from the Chinese General Hospital or require copies of the hospital records from the appellant before acting on the application for insurance. The fact of the matter is that the defendant was too eager to accept the application and receive the insured's premium. It would be inequitable now to allow the defendant to avoid liability under the circumstances."

Henson v. Philamlife, 56 OG 7328 Facts:

- Celestino Henson was insured by Philamlife in 1954 upon his application or a 20-yr endowment life policy.

- In 1955, the policy lapsed due to non-payment of the premiums. - Upon payment of the premiums due, the policy was reinstated, but in the

application for reinstatement, Henson did not disclose the fact that he had been previously diagnosed for pyelonephritis, enlarged liver and hernia. He also did not disclose that he had been examined by a physician.

- In 1956, Henson died, and his beneficiaries’ claim was rejected by Philamlife on the ground of concealment.

- The company then filed for rescission. Beneficiaries’ contend that the intent to conceal must be proven to warrant rescission.

Issue: Whether or not there is need to prove intent to conceal to warrant rescission. Held: NO. Sec. 26 provides that “a concealment whether intentional or unintentional entitles the injured party to rescind the contract of insurance”. And aside from this, intent, being a state of the mind is hard to prove. According to Sec. 30 of the Insurance Code: Materiality is to be determined not by the event, but solely by the probable and reasonable influence of the facts upon the party to whom the communication is due, in forming his estimate of the disadvantages of the proposed contract, or in making his inquiries. In essence therefore, the insured need not have died of the very diseases he had failed to reveal to the insurance company. It is sufficient that his non-revelation had misled the insurer in forming its estimate of the disadvantages of the proposed policy reinstatement or in making its inquiries, in order to entitle the latter to rescind the contract.

Yu Pang Cheng v. CA, 105 PHIL 1930 YU PANG CHENG alias YU PANG CHING, petitioner, vs.

THE COURT OF APPEALS, ET AL., respondents. FACTS: Plaintiff brought this action to collect from defendant the sum of P10,000.00, value of an insurance policy taken upon the life of one Yu Pang Eng, plus interest thereon at the legal rate, the sum of P10,000.00 as moral damages the further sum of P3,000.00 as attorney's fees, and the costs of action. Defendant, in its answer, set up the defense that the insured was guilty of misrepresentation and concealment of material facts in that he gave false and untruthful answers to certain questions asked him in his application for insurance which were material to the risk insured against and have the effect of avoiding the insurance policy. Rtc ordered defendant to pay. On appeal, the Court of Appeals reversed the decision of the trial court, holding that the insured was guilty of concealment of material facts which relieves defendant from liability. The ISSUE to be determined is whether the insured is guilty of concealment of some facts material to the risk insured against which has the effect of avoiding the policy as found by respondent court. HELD: The insured, in his application for insurance, particularly in his declarations to the examining physician, stated the following in answering the questions propounded to him: 14. Have you ever had any of the following diseases or symptoms? Each question must be read and answered "Yes" or "No". x x x x x x x x x Gastritis, Ulcer of the Stomach or any disease of that organ? No. Vertigo, Dizziness, Fainting-spells or Unconscious? No. Cancer, Tumors or Ulcers of any kind? No. 15. Have you ever consulted any physician not included in any of the above answers? Give names and address or physicians list ailments or accidents and date. No. It appears that the insured entered the Chinese General Hospital for medical treatment on January 29, 1950 having stayed there up to February 11, 1950. Upon entering the hospital, he complained of dizziness, anemia, abdominal pains and tarry stools, and in the evening of his admission he had several abdominal pains and his discharges were with black tarry stools and felt dizzy and weak. The history of his illness shows that the same "started a year ago as frequent dizziness." An X-Ray picture of his stomach was taken and the diagnosis made of him by his doctors showed that his illness was "peptic ulcer, bleeding." It should be noted that the insured's confinement in the Chinese General Hospital took place from January 29, 1950 to February 11, 1950, whereas his application for insurance wherein he stated his answer to the questions propounded to him by the examining physician of defendant was submitted to defendant on September 5, 1950. It is apparent that when the insured gave his answers regarding his previous ailment, particularly with regard to "Gastritis, Ulcer of the Stomach or any disease of that organ" and "Vertigo, Dizziness, Fainting-spells or Unconsciousness", he concealed the ailment of which he was treated in the Chinese General, Hospital which precisely has direct connection with the subject of the questions propounded. The negative answers given

Page 33: Insurance Case Digests (1)

Insurance Case Digests Atty. Bathan-Basuel

2-S, 2013-2014 33

by the insured regarding his previous ailment, or his concealment of the fact that he was hospitalized and treated for sometime of peptic ulcer and had suffered from "dizziness, anemia, abdominal pains and tarry stools", deprived defendant of the opportunity to make the necessary inquiry as to the nature of his past illness so that as it may form its estimate relative to the approval of his application. Had defendant been given such opportunity, considering the previous illness of the insured as disclosed by the record of the Chinese General Hospital, defendant would probably had never consented to the issuance of the policy in question. In fact, according to the death certificate, the insured died of "infiltrating medullary carcinoma, Grade 4, advanced cardiac and of lesser curvature, stomach metastases spleen", which may have direct connection with his previous illness.

Argente v. West Coast Life Insurance Co., 51 PHIL 725 ARGENTE VS. WEST COAST LIFE INSURANCE CO. G.R. No. 28499 March 19, 1928 Malcom, J.; FACTS:

On February 9, 1925, Bernardo Argente signed an application for joint insurance with his wife in the sum of P2,000. The wife, Vicenta de Ocampo, signed a like application for the same policy. Both applications, with the exception of the names and the signatures of the applicants, were written by Jose Geronimo del Rosario, an agent for the West Coast Life Insurance Co. But all the information contained in the applications was furnished the agent by Bernardo Argente.

Pursuant to his application, Bernardo Argente was examined by Dr. Cesareo Sta. Ana, a medical examiner for the West Coast Life Insurance Co., on February 10, 1925, in the office of the Customs House. The result of such examination was recorded in the Medical Examiner's Report, and with the exception of the signature of Bernardo Argente, was in the hand-writing of Doctor Sta. Ana. But the information or answers to the questions contained on the face of the Medical Examiner's Report were furnished the doctor by the applicant, Bernardo Argente.

Pursuant to her application, Vicenta de Ocampo, wife of the plaintiff, was examined by Dr. Cesareo Sta. Ana on February 10, 1925, at her residence in Manila. The result of the medical examination, including among other things, the answers given by Vicenta de Ocampo to the questions propounded to her by the physician, appears in the Medical Examiner's Report.

On May 9, 1925, Bernardo Argente and his wife submitted to the West Coast Life Insurance Co. an amended application for insurance, increasing the amount thereof to P15,000, and asked that the policy be dated May 15, 1925.

A temporary policy for P15,000 was issued to Bernardo Argente and his wife as of May 15, but it was not delivered to Bernardo Argente until July 2, 1925, when the

first quarterly premium on the policy was paid. In view of the fact that more than thirty days had elapsed since the applicants were examined by the company's physician, each of them was required to file a certificate of health before the policy was delivered to them.

On November 18, 1925, Vicenta de Ocampo died of cerebral apoplexy. Thereafter Bernardo Argente presented a claim in due form to the West Coast Life Insurance Co. for the payment of the sum of P15,000 the amount of the joint life Insurance policy. Following investigation conducted by the Manager of the Manila office of the insurance company, it was apparently disclosed that the answers given by the insured in their medical examinations with regard to their health and previous illness and medical attendance were untrue. For that reason, the West Coast Life Insurance Co. refused to pay the claim of Bernardo Argente, and on May 25, 1926, wrote him to the effect that the claim was rejected because the insurance was obtained through fraud and misrepresentation.

It is admitted that it appears in the Medical Examiner's Report that Bernardo Argente, in response to the question asked by the medical examiner, "Have you ever consulted a physician for, or have you ever suffered from any ailment or disease of, the brain or nervous system?" answered "No." To the question, "Have you consulted a physician for any ailment or disease not included in your above answer," answered "Yes. Nature of Ailment, Disease or Injury. Scabies, Number of attacks 1, Date 1911. It is, however, not disputed that on January 10, 11, and 13, 1923, Bernardo Argente was confined in the Philippine General Hospital where he was treated by Dr. Agerico B. M. Sison for cerebral congestion and Bell's Palsy.

It is further admitted that it appears in the Medical Examiner's Report that Vicenta de Ocampo, in response to the question asked by the medical examiner, "How frequently, if at all, and in what quantity do you use beer, wine, spirits or other intoxicants?" answered "Beer only in small quantities occasionally." To the question, "Have you ever consulted a physician for or have you ever suffered from any ailment or disease of the brain or nervous system?" answered "No." To the question, "What physician or physicians, if any, not named above, have you consulted or been treated by, within the last five years and for what illness or ailment? (If none, so state)" answered "None." And to the question, "Are you in good health as far as you know and believe?" answered "Yes."It is, however, not disputed that Vicenta de Ocampo was taken by a patrolman, at the request of her husband, Bernardo Argente, on May 19, 1924, to the Meisic police station, and from there was transferred to the San Lazaro Hospital. In San Lazaro Hospital, her case was diagnosed by the admitting physician as "alcoholism," but later Doctor Domingo made a diagnosis of probable "manic-depressive psychosis," and still, later in Mary Chiles Hospital, made a final diagnosis of "phycho-neurosis."

The plaintiff, Bernardo Argente, while readily conceding most of the facts herein narrated, yet alleges that both he and his wife revealed to the company's physician. Doctor Sta. Ana, all the facts concerning the previous illnesses and medical

Page 34: Insurance Case Digests (1)

Insurance Case Digests Atty. Bathan-Basuel

2-S, 2013-2014 34

attendance, but that Doctor Sta. Ana, presumably acting in collusion, with the insurance agent, Jose Geronimo del Rosario, failed to record them in the medical reports. ISSUE: Whether or not the alleged concealment was immaterial and insufficient to avoid the policy HELD:

In an action on a life insurance policy where the evidence conclusively shows that the answers to questions concerning diseases were untrue, the truth of falsity of the answers become the determining factor. In the policy was procured by fraudulent representations, the contract of insurance apparently set forth therein was never legally existent. It can fairly be assumed that had the true facts been disclosed by the assured, the insurance would never have been granted.

It is also held that the concealment must, in the absence of inquiries, be not only material, but fraudulent, or the fact must have been intentionally withheld; so it is held under English law that if no inquiries are made and no fraud or design to conceal enters into the concealment the contract is not avoided.

But it would seem that if a material fact is actually known to the assured, its concealment must of itself necessarily be a fraud, and if the fact is one which the assured ought to know, or is presumed to know, the presumption of knowledge ought to place the assured in the same position as in the former case with relation to material facts; and if the jury in such cases find the fact material, and one tending to increase the risk, it is difficult to see how the inference of a fraudulent intent or intentional concealment can be avoided.

And if insured while being examined for life insurance and knowing that she had heart disease, falsely stated that she was in good health, and though she could not read the application, it was explained to her and the questions asked through an interpreter, and the application like the policy contained and provision that no liability should be incurred unless the policy was delivered while the insured was in good health, the court properly directed a verdict for the insurer, though a witness who was present at the examination testified that the insured was not asked whether she had heart disease.

The basis of the rule vitiating the contract in case of concealment is that it misleads or deceives the insurer into accepting the risk, or accepting it at the rate of premium agreed upon. If the assured has exclusive knowledge of material facts, he should fully and fairly disclose the same, whether he believes them material or not. The determination of the point whether there has or has not been a material concealment must rest largely in all cases upon the form of the questions propounded and the exact terms of the contract.

Saturnino v. Philamlife7 SCRA 316

Appellant: IGNACIO SATURNINO, in his own behalf and as the JUDICIAL GUARDIAN OF CARLOS SATURNINO Appellees: THE PHILIPPINE AMERICAN LIFE INSURANCE COMPANY, Ponente: MAKALINTAL, J. Facts: Ignacio Saturnino sought to recover the sum of P5,000.00, corresponding to the face value 20-year endowment non-medical insurance on the life of Estefania A. Saturnino. This type of policy dispenses with the medical examination of the applicant usually required in ordinary life policies but a detailed information is called for in the application concerning the applicant's health and medical history. The application was submitted by Saturnino to Philamlife on November 16, 1957, witnessed by appellee's agent Edward A. Santos.. On September 19, 1958 Saturnino died of pneumonia, secondary to influenza so her surviving husband and minor childdemanded payment of the face value of the policy. The claim was rejected on the ground that two months prior to the issuance of the policy, Saturnino was operated on for cancer, involving complete removal of the right breast. She was discharged and the surgeon said she could not be considered cured for her cancer was malignant type. And Estefania A. Saturnino did not make a disclosure thereof in her application for insurance. On the contrary, she stated therein that she did not have, nor had she ever had, among other ailments listed in the application, cancer or other tumors; that she had not consulted any physician, undergone any operation or suffered any injury within the preceding five years; and that she had never been treated for nor did she ever have any illness or disease peculiar to her sex, particularly of the breast, ovaries, uterus, and menstrual disorders. The application also recites that the foregoing declarations constituted "a further basis for the issuance of the policy." Issue: whether or not the insured made such false representations of material facts as to avoid the policy. Held: The Insurance Law (Section 30) provides that "materiality is to be determined not by the event, but solely by the probable and reasonable influence of the facts upon the party to whom the communication is due, in forming his estimate of the proposed contract, or in making his inquiries." the waiver of medical examination renders even more material the information required of the applicant concerning previous condition of health and diseases suffered, for such information necessarily constitutes an important factor which the insurer takes into consideration in deciding whether to issue the policy or not. It is logical to assume that if appellee had been properly apprised of the insured's medical history she would at least have been made to undergo medical examination in order to determine her insurability. There was no negligence on the part of the insurance company for it was precisely because the insured had given herself a clean bill of health that appellee no longer considered an actual medical checkup necessary.

Saturnino contend there was no fraudulent concealment of the truth inasmuch as the insured herself did not know, since her doctor never told her, that the disease for which she had been operated on was cancer. However, In this jurisdiction a concealment,

Page 35: Insurance Case Digests (1)

Insurance Case Digests Atty. Bathan-Basuel

2-S, 2013-2014 35

whether intentional or unintentional, entitles the insurer to rescind the contract of insurance, concealment being defined as "negligence to communicate that which a party knows and ought to communicate" (Sections 24 & 26, Act No. 2427). n the case of Argente v. West Coast Life Insurance Co., 51 Phil. 725, 732, this Court said, quoting from Joyce, The Law of Insurance, 2nd ed., Vol. 3:

"The basis of the rule vitiating the contract in cases of concealment is that it misleads or deceives the insurer into accepting the risk, or accepting it at the rate of premium agreed upon. The insurer, relying upon the belief that the assured will disclose every material fact within his actual or presumed knowledge, is misled into a belief that the circumstance withheld does not exist, and he is thereby induced to estimate the risk upon a false basis that it does not exist."

Judgment dismissing the complaint affirmed.

Edillon v. Manila Bankers Life Insurance Corp., G.R. No. 34200, September 30, 1982

Edil lon v Manila Bankers Life G.R. No. L-34200 September 30, 1982 J. Vasquez Facts: Carmen O, Lapuz applied with Manila Bankers for insurance coverage against accident and injuries. She gave the date of her birth as July 11, 1904. She paid the sum of P20.00 representing the premium for which she was issued the corresponding receipt. The policy was to be effective for 90 days. During the effectivity, Carmen O. Lapuz died in a vehicular accident in the North Diversion Road. Petitioner Regina L. Edillon, a sister of the insured and the beneficiary in the policy, filed her claim for the proceeds of the insurance. Her claim having been denied, Regina L. Edillon instituted this action in the trial court. The insurance corporation relies on a provision contained in the contract excluding its liability to pay claims under the policy in behalf of "persons who are under the age of sixteen (16) years of age or over the age of sixty (60) years" They pointed out that the insured was over sixty (60) years of age when she applied for the insurance coverage, hence the policy became void. The trial court dismissed the complaint and ordered Edillon to pay P1000. The reason was that a policy of insurance being a contract of adhesion, it was the duty of the insured to know the terms of the contract he or she is entering into. The insured could not have been qualified under the conditions stated in said contract and should have asked for a refund of the premium. Issue: Whether the acceptance by the insurance corporation of the premium and the issuance of the corresponding certificate of insurance should be deemed a waiver of the exclusionary condition of coverage stated in the policy.

Held: Yes. Petition granted. Ratio: The age of Lapuz was not concealed to the insurance company. Her application clearly indicated her age of the time of filing the same to be almost 65 years of age. Despite such information which could hardly be overlooked, the insurance corporation received her payment of premium and issued the corresponding certificate of insurance without question. There was sufficient time for the private respondent to process the application and to notice that the applicant was over 60 years of age and cancel the policy. Under the circumstances, the insurance corporation is already deemed in estoppel. It inaction to revoke the policy despite a departure from the exclusionary condition contained in the said policy constituted a waiver of such condition, similar to Que Chee Gan vs. Law Union Insurance. The insurance company was aware, even before the policies were issued, that in the premises insured there were only two fire hydrants contrary to the requirements of the warranty in question. It is usually held that where the insurer, at the time of the issuance of a policy of insurance, has knowledge of existing facts which, if insisted on, would invalidate the contract from its very inception, such knowledge constitutes a waiver of conditions in the contract inconsistent with the known facts, and the insurer is stopped thereafter from asserting the breach of such conditions. To allow a company to accept one's money for a policy of insurance which it then knows to be void and of no effect, though it knows as it must, that the assured believes it to be valid and binding, is so contrary to the dictates of honesty and fair dealing. Capital Insurance & Surety Co., Inc. vs. - involved a violation of the provision of the policy requiring the payment of premiums before the insurance shall become effective. The company issued the policy upon the execution of a promissory note for the payment of the premium. A check given subsequent by the insured as partial payment of the premium was dishonored for lack of funds. Despite such deviation from the terms of the policy, the insurer was held liable. “... is that although one of conditions of an insurance policy is that "it shall not be valid or binding until the first premium is paid", if it is silent as to the mode of payment, promissory notes received by the company must be deemed to have been accepted in payment of the premium. In other words, a requirement for the payment of the first or initial premium in advance or actual cash may be waived by acceptance of a promissory note...”

Sun Life v. CA, 245 SCRA 268 (1995) G.R. No. 105135; June 22, 1995 SUNLIFE ASSURANCE COMPANY OF CANADA, petitioner vs. CA and Spouses ROLANDO and BERNARDA BACANI, respondents QUIASON, J.: FACTS:

Page 36: Insurance Case Digests (1)

Insurance Case Digests Atty. Bathan-Basuel

2-S, 2013-2014 36

In 1986, Robert John Bacani procured a life insurance contract for himself from Sun Life Assurance Company of Canada. He was issued a policy valued at P100,000, with double indemnity in case of accidental death. The designated beneficiary was his mother, Bernarda Bacani. In 1987, Robert died in a plane crash. Bernarda filed a claim with Sun Life, seeking the benefits of the insurance policy taken by her son. Sun Life conducted an investigation which prompted it to reject the claim because Robert did not disclose material facts (consultation within the past 5 years and seeking medical advice) relevant to the issuance of the policy, thus rendering the contract of insurance voidable. A check representing the total premiums paid (P10,172) was given to Bernarda. Sun Life discovered that two weeks prior to Robert’s application for insurance, he was examined and confined at the Lung Center of the Philippines, where he was diagnosed for renal failure. In 1988, Bernarda and her husband Rolando filed an action for specific performance against Sun Life in RTC Valenzuela, which ruled in favor of the spouses concluding that the facts concealed by the insured were made in good faith and under a belief that they need not be disclosed. Moreover, it held that the health history of the insured was immaterial since the insurance policy was "non-medical". CA affirmed. MR denied. Hence, the appeal. ISSUE: Whether the concealment is enough to rescind the insurance contract. ISSUE: YES. Sec. 26 of The Insurance Code is explicit in requiring a party to a contract of insurance to communicate to the other, in good faith, all facts within his knowledge which are material to the contract and as to which he makes no warranty, and which the other has no means of ascertaining. Said Section provides: “A neglect to communicate that which a party knows and ought to communicate, is called concealment.” The terms of the contract are clear. The insured is specifically required to disclose to the insurer matters relating to his health. The information which the insured failed to disclose were material and relevant to the approval and issuance of the insurance policy. The matters concealed would have definitely affected petitioner's action on his application. Thus, "good faith" is no defense in concealment. The argument, that petitioner's waiver of the medical examination of the insured debunks the materiality of the facts concealed, is untenable. Such argument would make Sec. 27, which allows the injured party to rescind a contract of insurance where there is concealment, ineffective. Even if the facts concealed had no bearing to the cause of death of the insured, it is well settled that the insured need not die of the disease he had failed to disclose to the insurer. Petitioner properly exercised its right to rescind the contract of insurance by reason of the concealment employed by the insured. It must be emphasized that rescission was exercised within the two-year contestability period as recognized in Section 48 of The Insurance Code. Petition GRANTED.

Insular Life v. Feliciano 74 PHIL 468

Insular Life Assurance Co. v. Feliciano, 74 Phil. 468 (1943);

FACTS:

Evaristo, who died on September 29, 1935, was suffering with advanced pulmonary tuberculosis when he signed his application for insurance with the petitioner on October 12, 1934. On that same date Doctor Trepp, who had taken X-ray pictures of his lungs, informed the respondent Dr. Serafin , brother of Evaristo, that the latter "was already in a very serious and practically hopeless condition." Nevertheless the question contained in the application — "Have you ever suffered from any ailment or disease of the lungs, pleurisy, pneumonia or asthma?" — appears to have been answered, "No." And above the signature of the applicant, following the answers to the various questions propounded to him, is the following printed statement:

"I declare on behalf of myself and of any person who shall have or claim any interest in any policy issued hereunder, that each of the above answers is full, complete and true, and that to the best of my knowledge and belief I am a proper subject for life insurance." (Exhibit K.)

The false answer above referred to, as well as the others, was written by the Company's soliciting agent Romulo M. David, in collusion with the medical examiner Dr. Gregorio Valdez, for the purpose of securing the Company's approval of the application so that the policy to be issued thereon might be credited to said agent in connection with the inter-provincial contest which the Company was then holding among its soliciting agents to boost the sales of its policies. Agent David bribed Medical Examiner Valdez with money which the former borrowed from the applicant's mother by way of advanced payment on the premium, according to the finding of the Court of Appeals. Said court also found that before the insured signed the application he, as well as the members of his family, told the agent and the medical examiner that he had been sick and coughing for some time and that he had gone three times to the Santol Sanatorium and had X-ray pictures of his lungs taken; but that in spite of such information the agent and the medical examiner told them that the applicant was a fit subject for insurance.

In September 1935, he died. His heirs filed an insurance claim but Insular Life denied the application as it averred that Feliciano’s application was attended by fraud. It was later found in court that the insurance agent and the medical examiner of Insular Life who assisted Feliciano in signing the application knew that Feliciano was already suffering from tuberculosis; that they were aware of the true medical condition of Feliciano yet they still made it appear that he was healthy in the insurance application form; that Feliciano signed the application in blank and the agent filled the information for him. In Insular Life v. Feliciano 73 Phil 201, the Court held that Insular life cannot avoid the insurance policy. The weight of authority is that if an agent of the insurer, after obtaining from an applicant for insurance a correct and truthful answer to interrogatories contained in the

Page 37: Insurance Case Digests (1)

Insurance Case Digests Atty. Bathan-Basuel

2-S, 2013-2014 37

application for insurance, without knowledge of the applicant fills in false answers, either fraudulently or otherwise, the insurer cannot assert the falsity of such answers as a defense to liability on the policy, and this is true generally without regard to the subject matter of the answers or the nature of the agent’s duties or limitations on his authority, at least if not brought to the attention of the applicant. Thus this motion for reconsideration.

ISSUE: Whether or not Insular life is correct in contending that Feliciano is not entitled to the claim because the insurance policy is void ab initio; that he connived with the insurance agent and the medical examiner; and that at best, Feliciano is only entitled to refund or the reimbursement of what he has paid in premium

HELD: Yes. This time, the Supreme Court held that Insular Life’s contention is correct. When Evaristo Feliciano, the applicant for insurance, signed the application in blank and authorized the soliciting agent and/or medical examiner of Insular to write the answers for him, he made them his own agents for that purpose, and he was responsible for their acts in that connection. If they falsified the answers for him, he could not evade the responsibility for the falsification. He was not supposed to sign the application in blank. He knew that the answers to the questions therein contained would be “the basis of the policy,” and for that very reason he was required with his signature to vouch for truth thereof.

Although the agent and the medical examiner knew that statement to be false, no valid contract of insurance was entered into because there was no real meeting of the minds of the parties.

From all the facts and circumstances of the case, we are constrained to conclude that the insured was a coparticipant, and coresponsible with Agent David and Medical Examiner Valdez, in the fraudulent procurement of the policies in question and that by reason thereof said policies are void ab initio.

Insular Life v. Feliciano 73 Phil 201

Facts:

- Evaristo Feliciano filed an application with Insular Life upon the solicitation of one of its agents.

- It appears that during that time, Evaristo was already suffering from tuberculosis. Such fact appeared during the medical exam, but the examiner and the company’s agent ignored it.

- After that, Evaristo was made to sign an application form and thereafter the blank spaces were filled by the medical examiner and the agent making it appear that Evaristo was a fit subject of insurance. (Evaristo could not read and understand English)

- When Evaristo died, Insular life refused to pay the proceeds because of concealment.

Issue: Whether or not Insular Life was bound by their agent’s acts. Held: Yes. The insurance business has grown so vast and lucrative within the past century. Nowadays, even people of modest means enter into insurance contracts. Agents who solicit contracts are paid large commissions on the policies secured by them. They act as general representatives of insurance companies. IN the case at bar, the true state of health of the insured was concealed by the agents of the insurer. The insurer’s medical examiner approved the application knowing fully well that the applicant was sick. The situation is one in which of two innocent parties must bear a loss for his reliance upon a third person. In this case, it is the one who drafted and accepted the policy and consummated the contract. It seems reasonable that as between the two of them, the one who employed and gave character to the third person as its agent should be the one to bear the loss. Hence, Insular is liable to the beneficiaries.

Aranilla v. Insular Life, 69 OG No. 4 637 Facts:

- In 1959, Jose Aranilla applied for life insurance with Insular. In his application, these 2 questions appeared:

- WON he has suffered from any disease of the kidney and urinary tract, to which he answered NO.

- WON he has been confined in a hospital for consultation and treatment, to which he answered that in 1947, he was confined due to influenza.

- The truth however, was that a few months prior to his application, he was confined and treated for nephritis, a disease of the kidney and urinary tract, and he was accordingly informed of the cause.

- When Aranilla died of cirrhosis of the liver, Insular refused to pay the proceeds due to concealment.

Issue: Whether the contract can be rescinded. Held: Yes. If an answer given by the insured to a specific question asked by the insurer in an application for life insurance turns out to be false, it is a concealment of a material fact which entitles the insurer to rescind, even if the insured died of an ailment which has NO connection with the specific questions falsely answered by him. This is because materiality is to be determined NOT by the event but ONLY by the probable and reasonable influence of the facts upon the party to whom the communication is due, in forming his estimate of the disadvantages of the proposed contract or in making his inquiries.

Ng Zee v. Asian Crusader Life Assurance Corp, 122 SCRA 461 [1983] NG GAN ZEE vs. ASIAN CRUSADER LIFE ASSURANCE CORPORATION

Facts:

Page 38: Insurance Case Digests (1)

Insurance Case Digests Atty. Bathan-Basuel

2-S, 2013-2014 38

- Kwong Nam applied for a 20-year endowment insurance on his life for the sum of P20,000.00, with his wife, Ng Gan Zee as beneficiary. On the same date, ACAC, upon receipt of the required premium from the insured, approved the application and issued the corresponding policy.

- Kwong Nam died of cancer of the liver with metastasis. All premiums had been religiously paid at the time of his death.

- Kwong Nam widow Ng Gan Zee presented a claim in due form to appellant for payment of the face value of the policy. She submitted the required proof of death of the insured. ACAC denied the claim on the ground that the answers given by the insured to the questions appealing in his application for life insurance were untrue.

- Ng Gan Zee brought the matter to the attention of the Insurance Commissioner, the Hon. Francisco Y. Mandamus, and the latter, after conducting an investigation, wrote the ACAC that he had found no material concealment on the part of the insured and that, therefore, Ng Gan Zee should be paid the full face value of the policy. This opinion of the Insurance Commissioner notwithstanding, ACAC refused to settle its obligation.

- ACAC alleged that the insured was guilty of misrepresentation when he answered "No" to the following question appearing in the application for life insurance:

Has any l ife insurance company ever refused your application for insurance or for reinstatement of a lapsed policy or offered you a policy different from that applied for? If, so, name company and date.

According to ACAC

"The insured applied for reinstatement of his lapsed life insurance policy with the Insular Life Insurance Co., Ltd, but this was declined by the insurance company, although later on approved for reinstatement with a very high premium as a result of his medical examination. Thus notwithstanding the said insured answered 'No' to the question propounded to him"

Issue: Whether Ng Gan Zee can collect the insurance claim.

CFI: ordered ACAC to pay the face value of the insurance policy

Held:

- There is no evidence that the Insular Life Assurance Co., Ltd. ever refused any application of Kwong Nam for insurance. Neither is there any evidence that any other insurance company has refused any application of Kwong Nam for insurance. The evidence shows that the Insular Life Assurance Co., Ltd. approved Kwong Nam's request for reinstatement and amendment of his lapsed insurance policy. The Court notes from said application for reinstatement and amendment, that the amount applied for was P20,000.00 only and not for P50,000.00 as it was in the lapsed policy. The amount of the reinstated and amended policy was also for P20,000.00. It results, therefore,

that when Kwong Nam answered 'No' to the question whether any life insurance company ever refused his application for reinstatement of a lapsed policy he did not misrepresent any fact

- Sec. 27. Such party a contract of insurance must communicate to the other, in good faith, all facts within his knowledge which are material to the contract, and which the other has not the means of ascertaining, and as to which he makes no warranty.

- Thus, "concealment exists where the assured had knowledge of a fact material to the risk, and honesty, good faith, and fair dealing requires that he should communicate it to the assurer, but he designedly and intentionally withholds the same." It has also been held "that the concealment must, in the absence of inquiries, be not only material, but fraudulent, or the fact must have been intentionally withheld."

- Assuming that the aforesaid answer given by the insured is false, as claimed by the ACAC. Sec. 27 of the Insurance Law, nevertheless requires that fraudulent intent on the part of the insured be established to entitle the insurer to rescind the contract. And as correctly observed by the lower court, "misrepresentation as a defense of the insurer to avoid liability is an 'affirmative' defense. The duty to establish such a defense by satisfactory and convincing evidence rests upon the defendant. The evidence before the Court does not clearly and satisfactorily establish that defense."

- It bears emphasis that Kwong Nam had informed the ACAC's medical examiner that the tumor for which he was operated on was "associated with ulcer of the stomach." In the absence of evidence that the insured had sufficient medical knowledge as to enable him to distinguish between "peptic ulcer" and "a tumor", his statement that said tumor was "associated with ulcer of the stomach, " should be construed as an expression made in good faith of his belief as to the nature of his ailment and operation. Indeed, such statement must be presumed to have been made by him without knowledge of its incorrectness and without any deliberate intent on his part to mislead the appellant.

- While it may be conceded that, from the viewpoint of a medical expert, the information communicated was imperfect, the same was nevertheless sufficient to have induced appellant to make further inquiries about the ailment and operation of the insured.

- Section 32. The right to information of material facts maybe waived either by the terms of insurance or by neglect to make inquiries as to such facts where they are distinctly implied in other facts of which information is communicated

- It has been held that where, upon the face of the application, a question appears to be not answered at all or to be imperfectly answered, and the insurers issue a policy without any further inquiry, they waive the imperfection of the answer and render the omission to answer more fully immaterial.

- As aptly noted by the lower court, "if the ailment and operation of Kwong Nam had such an important bearing on the question of whether the defendant would undertake the insurance or not, the court cannot understand why the defendant or its medical examiner did not make any further inquiries on such matters from the Chinese General Hospital or require copies of the hospital

Page 39: Insurance Case Digests (1)

Insurance Case Digests Atty. Bathan-Basuel

2-S, 2013-2014 39

records from the appellant before acting on the application for insurance. The fact of the matter is that the defendant was too eager to accept the application and receive the insured's premium. It would be inequitable now to allow the defendant to avoid liability under the circumstances."

5. Incontestabil ity

Philamcare Health Care System v. CA GR No. 125678, March 18, 2002

FACTS: Ernani Trinos, deceased husband of respondent Julita Trinos, applied for a health care coverage with petitioner Philamcare Health Systems, Inc. In the standard application form, he answered no to the following question:

Have you or any of your family members ever consulted or been treated for high blood pressure, heart trouble, diabetes, cancer, liver disease, asthma or peptic ulcer? (If Yes, give details).

The application was approved for a period of one year. Under the agreement, respondent’s husband was entitled to avail of hospitalization benefits, whether ordinary or emergency, listed therein. He was also entitled to avail of "out-patient benefits" such as annual physical examinations, preventive health care and other out-patient services. Upon the termination of the agreement, the same was extended for another year. The amount of coverage was increased to a maximum sum of P75,000.00 per disability. During the period of his coverage, Ernani suffered a heart attack and was confined at the Manila Medical Center (MMC) for one month beginning March 9, 1990. While her husband was in the hospital, respondent tried to claim the benefits under the health care agreement. However, petitioner denied her claim saying that the Health Care Agreement was void. According to petitioner, there was a concealment regarding Ernani’s medical history. Doctors at the MMC allegedly discovered at the time of Ernani’s confinement that he was hypertensive, diabetic and asthmatic, contrary to his answer in the application form. Thus, respondent paid the hospitalization expenses herself, amounting to about P76,000.00. In the morning of April 13, 1990, Ernani had fever and was feeling very weak. Respondent was constrained to bring him back to the Chinese General Hospital where he died on the same day. Respondent instituted with the Regional Trial Court of Manila, Branch 44, an action for damages against petitioner and its president, Dr. Benito Reverente. She asked for reimbursement of her expenses plus moral damages and attorney’s fees. After trial, the lower court ruled against petitioners. On appeal, the Court of Appeals affirmed the decision of the trial court but deleted all awards for damages and absolved petitioner Reverente. Petitioner’s motion for reconsideration was denied. Hence, petitioner brought the instant petition for review, raising the primary argument that a health care

agreement is not an insurance contract; hence the "incontestability clause" under the Insurance Code does not apply. PETITIONER ARGUES: The incontestability clause does not apply, as the same requires an effectivity period of at least two years. ISSUES: a.) Whether there has been a concealment. b.) Whether the incontestability clause apply in the case at bar. HELD: a.) Petitioner argues that respondent’s husband concealed a material fact in his application. It appears that in the application for health coverage, petitioners required respondent’s husband to sign an express authorization for any person, organization or entity that has any record or knowledge of his health to furnish any and all information relative to any hospitalization, consultation, treatment or any other medical advice or examination. Specifically, the Health Care Agreement signed by respondent’s husband states:

We hereby declare and agree that all statement and answers contained herein and in any addendum annexed to this application are full, complete and true and bind all parties in interest under the Agreement herein applied for, that there shall be no contract of health care coverage unless and until an Agreement is issued on this application and the full Membership Fee according to the mode of payment applied for is actually paid during the lifetime and good health of proposed Members; that no information acquired by any Representative of PhilamCare shall be binding upon PhilamCare unless set out in writing in the application; that any physician is, by these presents, expressly authorized to disclose or give testimony at anytime relative to any information acquired by him in his professional capacity upon any question affecting the eligibil ity for health care coverage of the Proposed Members and that the acceptance of any Agreement issued on this application shall be a ratification of any correction in or addition to this application as stated in the space for Home Office Endorsement.

In addition to the above condition, petitioner additionally required the applicant for authorization to inquire about the applicant’s medical history, thus:

hereby authorize any person, organization, or entity that has any record or knowledge of my health and/or that of __________ to give to the PhilamCare Health Systems, Inc. any and all information relative to any hospitalization, consultation, treatment or any other medical advice or examination. This authorization is in connection with the application for health care coverage only. A photographic copy of this authorization shall be as valid as the original.

Page 40: Insurance Case Digests (1)

Insurance Case Digests Atty. Bathan-Basuel

2-S, 2013-2014 40

Petitioner cannot rely on the stipulation regarding "Invalidation of agreement" which reads:

Failure to disclose or misrepresentation of any material information by the member in the application or medical examination, whether intentional or unintentional, shall automatically invalidate the Agreement from the very beginning and liability of Philamcare shall be limited to return of all Membership Fees paid. An undisclosed or misrepresented information is deemed material if its revelation would have resulted in the declination of the applicant by Philamcare or the assessment of a higher Membership Fee for the benefit or benefits applied for.

The answer assailed by petitioner was in response to the question relating to the medical history of the applicant. This largely depends on opinion rather than fact, especially coming from respondent’s husband who was not a medical doctor. Where matters of opinion or judgment are called for, answers made in good faith and without intent to deceive will not avoid a policy even though they are untrue. The fraudulent intent on the part of the insured must be established to warrant rescission of the insurance contract. Concealment as a defense for the health care provider or insurer to avoid liability is an affirmative defense and the duty to establish such defense by satisfactory and convincing evidence rests upon the provider or insurer. In any case, with or without the authority to investigate, petitioner is liable for claims made under the contract. Having assumed a responsibility under the agreement, petitioner is bound to answer the same to the extent agreed upon. In the end, the liability of the health care provider attaches once the member is hospitalized for the disease or injury covered by the agreement or whenever he avails of the covered benefits which he has prepaid. Under Section 27 of the Insurance Code, "a concealment entitles the injured party to rescind a contract of insurance." The right to rescind should be exercised previous to the commencement of an action on the contract. In this case, no rescission was made. Besides, the cancellation of health care agreements as in insurance policies require the concurrence of the following conditions: 1. Prior notice of cancellation to insured; 2. Notice must be based on the occurrence after effective date of the policy of one or more of the grounds mentioned; 3. Must be in writing, mailed or delivered to the insured at the address shown in the policy; 4. Must state the grounds relied upon provided in Section 64 of the Insurance Code and upon request of insured, to furnish facts on which cancellation is based. None of the above pre-conditions was fulfilled in this case. b.) Anent the incontestability of the membership of respondent’s husband, we quote with approval the following findings of the trial court:

(U)nder the title Claim procedures of expenses, the defendant Philamcare Health Systems Inc. had twelve months from the date of issuance of the Agreement within which to contest the membership of the patient if he had previous ailment of asthma, and six months from the issuance of the agreement if the patient was sick of diabetes or hypertension. The periods having expired, the defense of concealment or misrepresentation no longer lie.

Tan Chay Heng v. West Coast Life, 51 Phil 80 [1927]

Facts:

- In 1926, Tan Chay Heng sued West Coast on the policy allegedly issued to his “uncle”, Tan Caeng who died in 1925. He was the sole beneficiary thereof.

- West Coast refused on the ground that the policy was obtained by Tan Caeng with the help of agents Go Chuilian, Francisco Sanchez and Dr. Locsin of West Coast.

- West Coast said that it was made to appear that Tan Caeng was single, a merchant, health and not a drug user, when in fact he was married, a laborer, suffering form tuberculosis and addicted to drugs.

- West Coast now denies liability based on these misrepresentations. - Tan Chay contends that West Coast may not rescind the contract because an

action for performance has already been filed. - Trial court found for Tan Chay holding that an insurer cannot avoid a policy

which has been procured by fraud unless he brings an action to rescind it before he is sued thereon.

Issue: Whether or not West Coast’s action for rescission is therefore barred by the collection suit filed by Tan Chay. Held: NO. Precisely, the defense of West Cast was that through fraud in its execution, the policy is void ab initio, and therefore, no valid contract was ever made. Its action then cannot be fore rescission because an action to rescind is founded upon and presupposes the existence of the contract. Hence, West Coast’s defense is not barred by Sec. 47. In the instant case, it will be noted that even in its prayer, the defendant does not seek to have the alleged insurance contract rescinded. It denies that it ever made any contract of insurance on the life of Tan Caeng, or that any such a contract ever existed, and that is the question which it seeks to have litigated by its special defense. In the very nature of things, if the defendant never made or entered into the contract in question, there is no contract to rescind, and, hence, section 47 upon which the lower court based its decision in sustaining the demurrer does not apply. As stated, an action to rescind a contract is founded upon and presupposes the existence of the contract which is sought to be rescinded. If all of the material matters set forth and alleged in the defendant's special plea are true, there was no valid contract of insurance, for the simple reason that the minds of the parties never met

Page 41: Insurance Case Digests (1)

Insurance Case Digests Atty. Bathan-Basuel

2-S, 2013-2014 41

and never agreed upon the terms and conditions of the contract. We are clearly of the opinion that, if such matters are known to exist by a preponderance of the evidence, they would constitute a valid defense to plaintiff's cause of action. Upon the question as to whether or not they are or are not true, we do not at this time have or express any opinion, but we are clear that section 47 does not apply to the allegations made in the answer, and that the trial court erred in sustaining the demurrer.

Tan v. Court of Appeals, 174 SCRA 403 [1989] Facts:

- Tan Lee Siong was issued a policy by Philamlife on Nov. 6, 1973. - On Aprl 26, 1975, Tan died of hepatoma. His beneficiaries then filed a claim

with Philamlife for the proceeds of the insurance. - Philamlife wrote the beneficiaries in Sep. 1975 denying their claim and

rescinding the contract on the ground of misrepresentation. The beneficiaries contend that Philamlife can no longer rescind the contract on the ground of misrepresentation as rescission must allegedly be done “during the lifetime of the insured” within two years and prior to the commencement of the action following the wording of Sec. 48, par. 2.

Issue: Whether or not Philamlife can rescind the contract. Held: YES. The phrase “during the lifetime” found in Sec. 48 simply means that the policy is no longer in force after the insured has died. The key phrase in the second paragraph is “for a period of two years”.

Soliman v. US Life, 104 Phil 1046 [1958] Facts:

- US Life issued a 20 yr endowment life policy on the joint lives of Patricio Soliman and his wife Rosario, each of them being the beneficiary of the other.

- In Mar. 1949, the spouses were informed that the premium for Jan 1949 was still unpaid notwithstanding that the 31-day grace period has already expired, and they were furnished at the same time long-form health certificates for the reinstatement of the policies.

- In Apr 1949, they submitted the certificates and paid the premiums. - In Jan. 1950, Rosario died of acute dilation of the heart, and thereafter,

Patricio filed a claim for the proceeds of the insurance. - US life denied the claim and filed for the rescission of the contract on the

ground that the certificates failed to disclose that Rosario had been suffering from bronchial asthma for 3 years prior to their submission.

Issue: Whether or not the contract can still be rescinded. Held: Yes. The insurer is once again given two years from the date of reinstatement to investigate

into the veracity of the facts represented by the insured in the application for reinstatement. When US life sought to rescind the contract on the ground of concealment/misrepresentation, two years had not yet elapsed. Hence, the contract can still be rescinded.

Gonzalez Lao v. Yek Tong Lin Fire & Marine Insurance, 55 PHIL 386 EMILIO GONZALES LAO, plaintiff-appellee, vs. THE YEK TONG LIN FIRE AND MARINE INSURANCE CO., LTD., defendant-appellant VILLAMOR, J.: FACTS: Plaintiff Gonzales Lao was issued 2(two) fire insurance policies by Yek for P 100,000 covering his leaf tobacco products.These products were stored in Gonzales’ building on No. 188 Soler St. Plaintiff filed action to recover of the defendant the Yek Tong Lin Fire & Marine Insurance Co., Ltd., the amount of two insurance policies. It was alleged that on January 11, 1928, the leaf tobacco products was damaged by the fire that destroyed the building where said tobacco was stored, The defendant filed a general and specific denial of each and every allegation of the complaint, set up three special defenses, and prayed to be absolved from the complaint with costs against the plaintiff. In Art. 3 of the Insurance policies, it was provided that: “Any insurance in force upon all or part of the things unsured must be declared in writing by the insured and he (insured) should cause the company to insert or mention it in the policy. Without such requisite, such policy will be regarded as null and void and the insured will be deprived of all rights of indemnity in case of loss." Notwithstanding said provision, Gonzales entered into other insurance contracts. When he sought to claim from Yek after the fire, the latter denied any liability on the ground of violation of Art. 3 of the said policies. Plaintiff Gonzales however proved that the insurer knew of the other insurance policies obtained by him long efore the fire, and the insurer did NOT rescind the insurance polices in question but demanded and collected from the insured the premiums. ISSUE: Whether defendant Yek is still entitled to annul the contract HELD : NO. The defendant - appellant cannot invoke the violation of article 3 of the conditions of the insurance policies for the first time on appeal, having failed to do so in its answer; besides, as the appellee correctly contends in his brief, Guillermo Cu Unjieng, who was then president and majority shareholder of the appellant company, the Yek Tong Lin Fire & Marine Insurance Co., knew that there were other insurances, at least from the attempt to raise the insurance premium on the warehouse and the appellee's tobacco deposited therein to 1 per centum, and it was later reduced upon petition of the appellant itself and other assurance companies to 0.75 per centum presented to the association of assurance companies in the year 1927, and notwithstanding this, said appellant did not rescind the insurance policies in question, but demanded and collected from the appellee the increased premium.

Page 42: Insurance Case Digests (1)

Insurance Case Digests Atty. Bathan-Basuel

2-S, 2013-2014 42

That the defendant had knowledge of the existence of other policies obtained by the plaintiff from other insurance companies, is specifically shown by the defendant's answer wherein it alleges, by way of special defense, the fact that there exist other policies issued by the companies mentioned therein. If, with the knowledge of existence of other insurances which the defendant deemed violations of the contract, it has preferred to continue the policy, its action amounts to a waiver of the annulment of the contract.

Pacific Bank Corporation v. CA, 163 SCRA 1 Pacific Banking Corporation v. CA G.R. No. L-41014 November 28, 1988 Paras, J. Facts: On October 21,1963, Fire Policy No. F-3770, an open policy, was issued to the Paramount Shirt Manufacturing Co. by which Oriental Assurance Corporation bound itself to indemnify the insured for any loss or damage, not exceeding P61,000.00, caused by fire to its property consisting of stocks, materials and supplies usual to a shirt factory, including furniture, fixtures, machinery and equipment while contained in the ground, second and third floors of the building situated at number 256 Jaboneros St., San Nicolas, Manila, for a period of one year commencing from that date to October 21, 1964. Paramount was at the time of the issuance of the policy and is up to this time, a debtor of Pacific Banking Corp (Pacific) in the amount of not less P800,000.00 and the goods described in the policy were held in trust by the insured for the PBC under thrust receipts. Said policy was duly endorsed to Pacific as mortgagee/ trustor of the properties insured, with the knowledge and consent of Oriental to the effect that "loss if any under this policy is payable to the Pacific. While the aforesaid policy was in full force and effect, a fire broke out on the subject premises destroying the goods contained in its ground and second floors. The counsel for the Pacific sent a letter of demand to Oriental for indemnity due to the loss of property by fire under the endorsement of said policy However, Oriental informed counsel for the Pacific that it was not yet ready to accede to the latter's demand as the former is awaiting the final report of the insurance adjuster, H.H. Bayne Adjustment Company. Later, the said insurance adjuster notified counsel for the Pacific that the insured under the policy had not filed any claim with it, nor submitted proof of loss which is a clear violation of Policy Condition No.11, and for which reason, determination of the liability of Oriental could not be had. Pacific's counsel replied to aforesaid letter asking the insurance adjuster to verify from the records of the Bureau of Customs the entries of merchandise taken into the customs bonded warehouse razed by fire as a reliable proof of loss. For failure of the insurance company to pay the loss as demanded, Pacific filed in the court a quo an action for a sum of money against the private respondent, Oriental Assurance Corporation, in the principal sum of P61,000.00 issued in favor of Paramount Shirt Manufacturing Co. ).

At the trial, Pacific presented in evidence Exhibit "H", which is a communication dated December 22, 1965 of the insurance adjuster, H.H. Bayne Adjustment Co. to Asian Surety Insurance Co., Inc., revealing undeclared co-insurances with the following: P30,000.00 with Wellington Insurance; P25,000. 00 with Empire Surety and P250,000.00 with Asian Surety; undertaken by insured Paramount on the same property covered by its policy with private respondent whereas the only co-insurances declared in the subject policy are those of P30,000.00 with Malayan P50,000.00 with South Sea and P25.000.00 with Victory Issue: 1. WON Pacific unrevealed co-insurances which amounted to violation of policy conditions No. 3 and 2. WON Pacific failed to file the required proof of loss prior to court action. Policy Condition No. 3 explicitly provides:

3. The Insured shall give notice to the Company of any insurance already effected, or which may subsequently be effected, covering any of the property hereby insured, and unless such notice be given and the particulars of such insurance or insurances be stated in or endorsed on this Policy by or on behalf of the Company before the occurrence of any loss or damage, all benefit under this policy shall be forfeited. (Record on Appeal, p. 12)

Held: 1. Yes, It is not disputed that the insured failed to reveal before the loss three other insurances. As found by the Court of Appeals, by reason of said unrevealed insurances, the insured had been guilty of a false declaration; a clear misrepresentation and a vital one because where the insured had been asked to reveal but did not, that was deception. Otherwise stated, had the insurer known that there were many co-insurances, it could have hesitated or plainly desisted from entering into such contract. Hence, the insured was guilty of clear fraud Petitioner's contention that the allegation of fraud is but a mere inference or suspicion is untenable. In fact, concrete evidence of fraud or false declaration by the insured was furnished by the petitioner itself when the facts alleged in the policy under clauses "Co-Insurances Declared" and "Other Insurance Clause" are materially different from the actual number of co-insurances taken over the subject property. Consequently, "the whole foundation of the contract fails, the risk does not attach and the policy never becomes a contract between the parties. Representations of facts are the foundation of the contract and if the foundation does not exist, the superstructure does not arise. Falsehood in such representations is not shown to vary or add to the contract, or to terminate a contract which has once been made, but to show that no contract has ever existed. Undoubtedly, it is but fair and just that where the insured who is primarily entitled to receive the proceeds of the policy has by its fraud and/or misrepresentation,

Page 43: Insurance Case Digests (1)

Insurance Case Digests Atty. Bathan-Basuel

2-S, 2013-2014 43

forfeited said right, with more reason petitioner which is merely claiming as indorsee of said insured, cannot be entitled to such proceeds. 2. Yes. As the insurance policy against fire expressly required that notice should be given by the insured of other insurance upon the same property, the total absence of such notice nullifies the policy. The argument that notice of co-insurances may be made orally is preposterous and negates policy condition No. 20 which requires every notice and other communications to the insurer to be written or printed. The evidence adduced shows that twenty-four (24) days after the fire, petitioner merely wrote letters to private respondent to serve as a notice of loss, thereafter, the former did not furnish the latter whatever pertinent documents were necessary to prove and estimate its loss. Instead, petitioner shifted upon private respondent the burden of fishing out the necessary information to ascertain the particular account of the articles destroyed by fire as well as the amount of loss. It is noteworthy that private respondent and its adjuster notified petitioner that insured had not yet filed a written claim nor submitted the supporting documents in compliance with the requirements set forth in the policy. Despite the notice, the latter remained unheedful. Since the required claim by insured, together with the preliminary submittal of relevant documents had not been complied with, it follows that private respondent could not be deemed to have finally rejected petitioner's claim and therefore the latter's cause of action had not yet arisen. Compliance with condition No. 11 is a requirement sine qua non to the right to maintain an action as prior thereto no violation of petitioner's right can be attributable to private respondent. This is so, as before such final rejection, there was no real necessity for bringing suit. Petitioner should have endeavored to file the formal claim and procure all the documents, papers, inventory needed by private respondent or its adjuster to ascertain the amount of loss and after compliance await the final rejection of its claim. Indeed, the law does not encourage unnecessary litigation.

Verily, petitioner prematurely filed Civil Case No. 56889 and dismissal thereof was warranted under the circumstances. While it is a cardinal principle of insurance law that a policy or contract of insurance is to be construed liberally in favor of the insured and strictly as against the insurer company yet, contracts of insurance, like other contracts, are to be construed according to the sense and meaning of the terms which the parties themselves have used.

It appearing that insured has violated or failed to perform the conditions under No. 3 and 11 of the contract, and such violation or want of performance has not been waived by the insurer, the insured cannot recover, much less the herein petitioner. Courts are not permitted to make contracts for the parties; the function and duty of the courts is simply to enforce and carry out the contracts actually made.

6. The Policy

Perez v. CA, 323 SCRA 613 (2000)

VIRGINIA A. PEREZ, petitioner, vs. COURT OF APPEALS and BF LIFEMAN INSURANCE CORPORATION, respondents.

G.R. No. 112329. January 28, 2000]

FACTS:

Primitivo B. Perez had been insured with the BF Lifeman Insurance Corporation since 1980 for P20,000.00. Sometime in October 1987, an agent of the insurance corporation, Rodolfo Lalog, visited Perez in Guinayangan, Quezon and convinced him to apply for additional insurance coverage of P50,000.00, to avail of the ongoing promotional discount of P400.00 if the premium were paid annually. On October 20, 1987, Primitivo B. Perez accomplished an application form for the additional insurance coverage of P50,000.00. On the same day, petitioner Virginia A. Perez, Primitivo’s wife, paid P2,075.00 to Lalog. The receipt issued by Lalog indicated the amount received was a "deposit." Unfortunately, Lalog lost the application form accomplished by Perez and so he asked the latter to fill up another application form. On November 1, 1987, Perez was made to undergo the required medical examination, which he passed.

Pursuant to the established procedure of the company, Lalog forwarded the application for additional insurance of Perez, together with all its supporting papers, to the office of BF Lifeman Insurance Corporation at Gumaca, Quezon which office was supposed to forward the papers to the Manila office. On November 25, 1987, Perez died in an accident. He was riding in a banca which capsized during a storm. At the time of his death, his application papers for the additional insurance of P50,000.00 were still with the Gumaca office. Lalog testified that when he went to follow up the papers, he found them still in the Gumaca office and so he personally brought the papers to the Manila office of BF Lifeman Insurance Corporation. It was only on November 27, 1987 that said papers were received in Manila. Without knowing that Perez died on November 25, 1987, BF Lifeman Insurance Corporation approved the application and issued the corresponding policy for the P50,000.00 on December 2, 1987. Petitioner Virginia Perez went to Manila to claim the benefits under the insurance policies of the deceased. She was paid P40,000.00 under the first insurance policy for P20,000.00 (double indemnity in case of accident) but the insurance company refused to pay the claim under the additional policy coverage of P50,000.00, the proceeds of which amount to P150,000.00 in view of a triple indemnity rider on the insurance policy. In its letter of January 29, 1988 to Virginia A. Perez, the insurance company maintained that the insurance for P50,000.00 had not been perfected at the time of the death of Primitivo Perez. Consequently, the insurance company refunded the amount of P2,075.00 which Virginia Perez had paid.

Page 44: Insurance Case Digests (1)

Insurance Case Digests Atty. Bathan-Basuel

2-S, 2013-2014 44

Then, private respondent BF Lifeman Insurance Corporation filed a complaint against Virginia A. Perez seeking the rescission and declaration of nullity of the insurance contract in question.

Petitioner Virginia A. Perez, on the other hand, averred that the deceased had fulfilled all his prestations under the contract and all the elements of a valid contract are present. She then filed a counterclaim against private respondent for the collection of P150,000.00 as actual damages, P100,000.00 as exemplary damages, P30,000.00 as attorney’s fees and P10,000.00 as expenses for litigation.

The RTC of Manila ruled in favor of Virginia. The premium for the additional insurance of P50,000.00 had been fully paid and even if the sum of P2,075.00 were to be considered merely as partial payment, the same does not affect the validity of the policy. The trial court further stated that the deceased had fully complied with the requirements of the insurance company. He paid, signed the application form and passed the medical examination. He should not be made to suffer the subsequent delay in the transmittal of his application form to private respondent’s head office since these were no longer within his control.

The CA reversed the decision of the trial court and held that the insurance contract for P50,000.00 could not have been perfected since at the time that the policy was issued, Primitivo was already dead. The Court of Appeals held that the contract of insurance had to be assented to by both parties and so long as the application for insurance has not been either accepted or rejected, it is merely an offer or proposal to make a contract.

ISSUES:

- Whether, as between Primitivo and BF Lifeman Insurance Corp, there was a perfected contract of insurance.

- Whether the condition that the policy issued by the corporation be delivered and received by the applicant in good health, is potestative, being dependent upon the will of the insurance company, and is therefore null and void.

HELD:

1. None. When Primitivo filed an application for insurance, paid P2,075.00 and submitted the results of his medical examination, his application was subject to the acceptance of private respondent BF Lifeman Insurance Corporation. The perfection of the contract of insurance between the deceased and respondent corporation was further conditioned upon compliance with the following requisites stated in the application form:

"there shall be no contract of insurance unless and until a policy is issued on this application and that the said policy shall not take effect until the premium has been paid and the policy delivered to and accepted by me/us in person while I/We, am/are in good health."

The assent of private respondent BF Lifeman Insurance Corporation therefore was not given when it merely received the application form and all the requisite supporting papers of the applicant. Its assent was given when it issues a corresponding policy to the applicant. Also, it when Primitivo died on November 25, 1987, his application papers for additional insurance coverage were still with the branch office of respondent corporation in Gumaca and it was only two days later when Lalog personally delivered the application papers to the head office in Manila. Consequently, there was absolutely no way the acceptance of the application could have been communicated to the applicant for the latter to accept inasmuch as the applicant at the time was already dead.

2. The SC did not agree. In the case at bar, the following conditions were imposed by the respondent company for the perfection of the contract of insurance:

1. a policy must have been issued;

2. the premiums paid; and

3. the policy must have been delivered to and accepted by the applicant while he is in good health.

The condition imposed by the corporation that the policy must have been delivered to and accepted by the applicant while he is in good health can hardly be considered as a potestative or facultative condition. On the contrary, the health of the applicant at the time of the delivery of the policy is beyond the control or will of the insurance company. Rather, the condition is a suspensive one whereby the acquisition of rights depends upon the happening of an event which constitutes the condition. In this case, the suspensive condition was the policy must have been delivered and accepted by the applicant while he is in good health. There was non-fulfillment of the condition, however, inasmuch as the applicant was already dead at the time the policy was issued. Hence, the non-fulfillment of the condition resulted in the non-perfection of the contract.

As stated above, a contract of insurance, like other contracts, must be assented to by both parties either in person or by their agents. So long as an application for insurance has not been either accepted or rejected, it is merely an offer or proposal to make a contract. The contract, to be binding from the date of application, must have been a

Page 45: Insurance Case Digests (1)

Insurance Case Digests Atty. Bathan-Basuel

2-S, 2013-2014 45

completed contract, one that leaves nothing to be done, nothing to be completed, nothing to be passed upon, or determined, before it shall take effect. There can be no contract of insurance unless the minds of the parties have met in agreement.

Commissioner of Internal Revenue v. Lincoln Philippine Life Insurance Co., 379 SCRA423

COMMISSIONER OF INTERNAL REVENUE v. LINCOLN PHILIPPINE LIEF INSURANCE COMPANY INC. (now JARDINE-CMA LIFE INSURANCE COMPANY, INC.) Lincoln Philippine Life Insurance Co., Inc., (now Jardine-CMA Life Insurance Company, Inc.) is a domestic corporation engaged in life insurance business. In the years prior to 1984, it issued a special kind of life insurance policy known as the “Junior Estate Builder Policy,” the distinguishing feature of which is a clause providing for an automatic increase in the amount of life insurance coverage upon attainment of a certain age by the insured without the need of issuing a new policy. The clause was to take effect in the year 1984. That year, the insurance company also issued shares of stock dividends. Documentary stamp taxes were paid based only on the par value. Subsequently, the Commissioner on Internal Revenue issued deficiency documentary stamps tax assessment for the year 1984 in the amounts corresponding to the amount of automatic increase of the sum assured on the policy issued by respondent, and another corresponding to the book value in excess of the par value of the stock dividends. The insurance company questioned the deficiency assessments before the Court of Tax Appeals. The CTA found no valid basis for the deficiency tax assessment on the stock dividends, as well as on the insurance policy. The CIR appealed the decision to the Court of Appeals, which affirmed the CTA decision insofar as it nullified the deficiency assessment on the insurance policy, but reversing the same with regard to the deficiency assessment on the stock dividends. Both parties appealed. The insurance company questions the validity of the deficiency tax assessment on stock dividends. The CIR, meanwhile, questioned the decision invalidating the deficiency assessment on the insurance policy. The CIR claims that the automatic increase clause in the subject insurance policy is separate and distinct from the main agreement and involves another transaction, and that, while no new policy was issued, the original policy was essentially re-issued when the additional obligation was assumed upon the effectivity of this “automatic increase clause” in 1984. Hence, a deficiency assessment based on the additional insurance not covered in the main policy is in order.

The Court of Appeals sustained the CTA’s ruling that there was only one transaction involved in the issuance of the insurance policy and that the “automatic increase clause” is an integral part of that policy. ISSUE Whether or not there was a separate transaction RULING The subject insurance policy at the time it was issued contained an “automatic increase clause.” Although the clause was to take effect only in 1984, it was written into the policy at the time of its issuance. The distinctive feature of the “junior estate builder policy” called the “automatic increase clause” already formed part and parcel of the insurance contract, hence, there was no need for an execution of a separate agreement for the increase in the coverage that took effect in 1984 when the assured reached a certain age. It is clear from Section 173 of the NIRC that the payment of documentary stamp taxes is done at the time the act is done or transaction had and the tax base for the computation of documentary stamp taxes on life insurance policies under Section 183 is the amount fixed in policy, unless the interest of a person insured is susceptible of exact pecuniary measurement. What then is the amount fixed in the policy? Logically, we believe that the amount fixed in the policy is the figure written on its face and whatever increases will take effect in the future by reason of the “automatic increase clause” embodied in the policy without the need of another contract. Here, although the automatic increase in the amount of life insurance coverage was to take effect later on, the date of its effectivity, as well as the amount of the increase, was already definite at the time of the issuance of the policy. Thus, the amount insured by the policy at the time of its issuance necessarily included the additional sum covered by the automatic increase clause because it was already determinable at the time the transaction was entered into and formed part of the policy. The “automatic increase clause” in the policy is in the nature of a conditional obligation under Article 1181 by which the increase of the insurance coverage shall depend upon the happening of the event which constitutes the obligation. In the instant case, the additional insurance that took effect in 1984 was an obligation subject to a suspensive obligation, but still a part of the insurance sold to which private respondent was liable for the payment of the documentary stamp tax. The deficiency of documentary stamp tax imposed on private respondent is definitely not on the amount of the original insurance coverage, but on the increase of the amount insured upon the effectivity of the “Junior Estate Builder Policy.” Finally, it should be emphasized that while tax avoidance schemes and arrangements are not prohibited,10 tax laws cannot be circumvented in order to evade the payment of just taxes. In the case at bar, to claim that the increase in the amount insured (by virtue of the automatic increase clause incorporated into the policy at the time of

Page 46: Insurance Case Digests (1)

Insurance Case Digests Atty. Bathan-Basuel

2-S, 2013-2014 46

issuance) should not be included in the computation of the documentary stamp taxes due on the policy would be a clear evasion of the law requiring that the tax be computed on the basis of the amount insured by the policy.WHEREFORE, the petition is hereby given DUE COURSE.

Lim v. Sun Life Assurance Co of Canada 41 Phil 265 Facts:

- On July 6, 1917, Luis Lim Y Garcia of Zamboanga applied for a policy of life insurance with Sunlife in the amount of 5T.

- He designated his wife Pilar Lim as the beneficiary. The first premium of P433 was paid by Lim and company issued a “provisional policy”

- Such policy contained the following provisions “xx the abovementioned life is to be assured in accordance with the terms and conditions contained or inserted by the Company in the policy which may be granted by it in this particular case for 4 months only from the date of the application, PROVIDED that the company shall confirm this agreement by issuing a policy on said application xxx. Should the company NOT issue such a policy, then this agreement shall be null and void ab initio and the Company shall be held not to have been on the risk at all, but in such case, the amount herein shall be returned.

- Lim died on Aug. 23, 1917 after the issuance of the provisional policy but before the approval of the application by the home office of the insurance company.

- The instant action is brought by the beneficiary to recover from Sun Life the sum of 5T.

Issue: Whether or not the beneficiary can collect the 5T. Held: NO. The contract of insurance was not consummated by the parties. The above quoted agreement clearly stated that the agreement should NOT go into effect until the home office of the Company shall confirm it by issuing a policy. It was nothing but an acknowledgment by the Company that it has received a sum of money agreed upon as the first year’s premium upon a policy to be issued upon the application if it is accepted by the Company. When an agreement is made between the applicant and the agent whether by signing an application containing such condition or otherwise, that no liability shall attach until the principal approves the risk and a receipt is given by the agent, such acceptance is merely conditional and is subordinated to the company’s act in approving or rejecting; so in life insurance a “binding slip or receipt” does not insure itself.

Enriquez v Sun Life Assurance Co 41 Phil 269 RAFAEL ENRIQUEZ v. SUN LIFE ASSURANCE COMPANY OF CANADA

- Rafael Enriquez, administrator of the estate of the late Joaquin Herrera made

application to the Sun Life Assurance Company of Canada for a life annuity. The application was immediately forwarded to the head office of the company in Canada, which gave notice of acceptance by cable to Manila. The policy was issued at Montreal on December 4, 1917. However, on December 18, Herrera informed the Manila office that he desired to withdraw his application. The local office replied to Torres stating that the policy had been issued. This letter was received by the attorney on the morning of December 21. Mr. Herrera died on December 20, 1917.

- According to the SC, the letter of November 26, 1917, notifying Mr. Herrera that his application had been accepted, was prepared and signed in the local office of the insurance company, was placed in the ordinary channels for transmission, but as far as the SC knows was never actually mailed and thus was never received by the applicant. RULING

- The law applicable to the case is found to be the second paragraph of article 1262 of the Civil Code providing that an acceptance made by letter shall not bind the person making the offer except from the time it came to his knowledge.

- The pertinent fact is, that according to the provisional receipt, three things had to be accomplished by the insurance company before there was a contract: (1) There had to be a medical examination of the applicant; (2) there had to be approval of the application by the head office of the company; and (3) this approval had in some way to be communicated by the company to the applicant.

- The further admitted facts are that the head office in Montreal did accept the application, did cable the Manila office to that effect, did actually issue the policy and did, through its agent in Manila, actually write the letter of notification and place it in the usual channels for transmission to the addressee.

- The fact as to the letter of notification thus fails to concur with the essential elements of the general rule pertaining to the mailing and delivery of mail matter as announced by the American courts, namely, when a letter or other mail matter is addressed and mailed with postage prepaid there is a rebuttable presumption of fact that it was received by the addressee as soon as it could have been transmitted to him in the ordinary course of the mails. But if any one of these elemental facts fails to appear, it is fatal to the presumption. For instance, a letter will not be presumed to have been received by the addressee unless it is shown that it was deposited in the post-office, properly addressed and stamped.

- We hold that the contract for a life annuity in the case at bar was not perfected because it has not been proved satisfactorily that the acceptance of the application ever came to the knowledge of the applicant.

Development Bank of the Phils v CA 231 SCRA 370

Page 47: Insurance Case Digests (1)

Insurance Case Digests Atty. Bathan-Basuel

2-S, 2013-2014 47

Development Bank of the Phil ippines v CA - March 21, 1994

Facts:

Juan B. Dans, together with his family applied for a loan of P500,000 with DBP. As principal mortgagor, Dans, then 76 years of age was advised by DBP to obtain a mortgage redemption insurance (MRI) with DBP MRI pool. A loan in the reduced amount was approved and released by DBP. From the proceeds of the loan, DBP deducted the payment for the MRI premium. The MRI premium of Dans, less the DBP service fee of 10%, was credited by DBP to the savings account of DBP MRI-Pool. Accordingly, the DBP MRI Pool was advised of the credit.

Dans died of cardiac arrest. DBP MRI Pool notified DBP that Dans was not eligible for MRI coverage, being over the acceptance age limit of 60 years at the time of application. DBP apprised Candida Dans of the disapproval of her late husband’s MRI application. DBP offered to refund the premium which the deceased had paid, but Candida Dans refused to accept the same demanding payment of the face value of the MRI or an amount equivalent of the loan. She, likewise, refused to accept an ex gratia settlement which DBP later offered. Hence the case at bar.

Issue:

Whether or not the DBP MRI Pool should be held liable on the ground that the contract was already perfected?

Held:

No, it is not liable. The power to approve MRI application is lodged with the DBP MRI Pool. The pool, however, did not approve the application. There is also no showing that it accepted the sum which DBP credited to its account with full knowledge that it was payment for the premium. There was as a result no perfected contract of insurance’ hence the DBP MRI Pool cannot be held liable on a contract that does not exist

In dealing with Dans, DBP was wearing 2 legal hats: the first as a lender and the second as an insurance agent. As an insurance agent, DBP made Dans go through the motion of applying for said insurance, thereby leading him and his family to believe that they had already fulfilled all the requirements for the MRI and that the issuance of their policy was forthcoming. DBP had full knowledge that the application was never going to be approved. The DBP is not authorized to accept applications for MRI when its clients are more than 60 years of age. Knowing all the while that Dans was ineligible, DBP exceeded the scope of its authority when it accepted the application for MRI by collecting the insurance premium and deducting its agent’s commission and service fee. Since the

third person dealing with an agent is unaware of the limits of the authority conferred by the principal on the agent and he has been deceived by the non-disclosure thereof by the agent, then the latter is liable for damages to him.

Ang Giok Chip v. Springfield 56 Phil 375.

FACTS: Ang Giok Chip doing business under the name and style of Hua Bee Kong Si was formerly the owner of a warehouse in Manila.The contents of the warehouse were insured with three insurance companies for the total sum of P60,000. One insurance policy, in the amount of P10,000, was taken out with the Springfield Fire & Marine Insurance Company. The warehouse was destroyed by fire on January 11, 1928, while the policy issued by the latter company was in force. Ang Giok instituted an action to recover the insurance proceeds from Springfield. Springfield contended that Ang GIok violated warranty F in the insurance policy because the warehouse contained 39% hazardous material when said warranty only allows 3% hazardous material of the total value of the goods stored in the warehouse. "WARRANTY F

"It is hereby declared and agreed that during the currency of this policy no hazardous goods be stored in the Building to which this insurance applies or in any building communicating therewith, provided, always, however, that the Insured be permitted to store a small quantity of the hazardous goods specified below, but not exceeding in all 3 per cent of the total value of the whole of the goods or merchandise contained in said warehouse, viz;. . . "

Ang Giok contends that Warranty F is in the nature of a rider included in an insurance policy which is null and void because the same does not comply with Section 65 of the Insurance Act which provides that, "Every express warranty, made at or before the execution of a policy, must be contained in the policy itself, or in another instrument signed by the insured and referred to in the policy, as making a part of it."

ISSUE: Whether a warranty referred to in the policy as forming part of the contract of insurance and in the form of a rider to the insurance policy, is null and void?

HELD: NO. Based on prevailing doctrines, it is well settled that a rider attached to a policy is a part of the contract, to the same extent and with like effect as if actually embodied therein. It is equally well settled that an express warranty must appear upon the face of the policy, or be clearly incorporated therein and made a part thereof by explicit reference, or by words clearly evidencing such intention. In the case at bar, the rider, warranty F, is contained in the policy itself, because by the contract of insurance agreed to by the parties it is made to form a part of the same, but is not another instrument signed by the insured and referred to in the policy as forming a part of it.

Page 48: Insurance Case Digests (1)

Insurance Case Digests Atty. Bathan-Basuel

2-S, 2013-2014 48

Moreover, it is admitted that the policy before us was accepted by the plaintiff. The receipt of this policy by the insured without objection binds both the acceptor and the insured to the terms thereof. The insured may not thereafter be heard to say that he did not read the policy or know its terms, since it is his duty to read his policy and it will be assumed that he did so.

We have here a standard insurance policy and the issuance of the policy in this case with its attached rider conforms to well established practice in the Philippines and elsewhere. To nullify such policies would place an unnecessary hindrance in the transactions of insurance business in the Philippines. These are matters of public policy.

Sindayen v Insular Life 62 Phil 9 Facts:

- Arturo Sindayen was a linotype operator in the Bureau of Printing. He and his wife Fortunat went to Camiling to spend Christmas with his aunt Felicidad Estrada.

- On Dec. 26, 1932, while still in Camiling, he made a written application to Insular Life, through its agent, Cristobal Hendoza, for a policy of insurance on his life in the sum of 1,000.

- He paid the agent P15 as part of the first premium. It was agreed that the policy, when and if issued, should be delivered to Felicidad with whom Sindayen left the sum P25.06 to complete the payment of the first annual premium of P40.06.

- On Jan 1, 1933, Sindayen was examined by Insular’s doctor who made a favorable report to Insular.

- The next day, Sindayen returned to Manila and resumed his work. On Jan. 11, 1933, Insular accepted the risk and issued a policy, and mailed the same to its agent for delivery to the insured.

- On Jan. 12, 1933, Sindayen complained of a severe headache. ON Jan. 15, 1933, he called a physician who found that Sindayen was suffering from acute nephritis and uremia. His illness did not yield to treatment and on Jan. 19, 1933, he died.

- The policy which the company issued and mailed in manila on Jan. 11 1933 was received by its agent in Camilin on Jan. 16, 1933. On Jan 18, 1933, the agent, in accordance with his agreement with the insured delivered the policy to Felicided upon her payment of the balance of the 1st year’s premium.

- The agent asked Felicidad if her nephew was in good health and she replied that she believed so because she had no information that he was sick, and thereupon , the policy was handed to her by the agent.

- On Jan. 20, 1933, the agent learned of the death of Sindayen, afterwhich he called upon Felicidad and asked her to return the policy. Felicidad did so.

- On Feb. 4, 1933, the company obtained from Sindayen’s widow Fortunata (also the beneficiary), her signature on a legal document whereby in consideration of the sum 40.06 representing the amount of premium paid,

Fortunata thereby releases forever and discharges Insular from any and all claims and obligations she may have against the latter.

- A check for the above-mentioned amount was drawn in the name of Fortunata, but the same was never encashed.

- Instead, it was returned to Insular and this complaint to enforce payment under the policy was instituted.

- The application which Sindayen signed in Camiling contained the following provisions:

- “xxx - (3) That the said policy shall not take effect until the first premium has been

paid and the policy has been delivered to and accepted by me, while I am in good health.”

o The main defense of the company is the policy never took effect

because of par. 3 of the application, since at the time of the delivery of the agent, the insured was not in good health.

Issue: Whether or not the policy took effect. Held: YES. There is one line of American cases which holds that the stipulation contained par. 3 is in the nature of a condition precedent, that is to say, that there can be no valid delivery to the insured unless he is in good health at that time; that this condition precedent goes to the very essence of the contract and cannot be waived by the agent making delivery of the policy; HOWEVER, there is also a number of American decision which state the contrary. These decisions say that an agent to whom a life insurance policy (similar to the one at bar) was sent with instruction to deliver it to the insured, has authority to bind the company by making such delivery, ALTHOUGH the insured was NOT in good health at the time of delivery, on the theory that the delivery of the policy being the final act to the consummation of the contract, the condition as to the insured’s good health was WAIVED by the company. These same cases further hold that the delivery of the policy by the agent to the insured consummates the contract even though the agent knew that the insured was NOT in good health at the time, the theory being, that his knowledge is the company’s knowledge; and his delivery is the company’s delivery; that when the delivery is made notwithstanding this knowledge of the defect, the company is deemed to have WAIVED such defect. The agent, Mendoza was duly licensed by the Insurance Commission to act for Insular Life. He had the authority given by him by the company to withhold the delivery of the policy to the insured until the first premium has been paid and the policy has been delivered to and accepted by the insured while he is in good health. Whether that condition had been met or not plainly calls for the exercise of discretion. Mendoza’s decision that the condition had been met by the insured and that it was proper to make

Page 49: Insurance Case Digests (1)

Insurance Case Digests Atty. Bathan-Basuel

2-S, 2013-2014 49

delivery of the policy to him is just as binding on the company as if the decision had been made by its Board of Directors. Admittedly, Mendoza made a mistake of judgment because he acted on insufficient evidence as to the state of health of the insured, and this mistake cannot be said to be induced by any misconduct on the part of the insured. It is in the interest of not only of the applicant but of all insurance companies as well that there should be some act which gives the applicant the definite assurance that the contract has been consummated. This sense of security and of piece of mind that one’s dependents are provided for without risk of either loss or of litigation is the bedrock of life insurance. A cloud will be thrown over the entire insurance business if the condition of health of the insured at the time of the delivery of the policy may be inquired into years afterwards with the view of avoiding the policy on the ground that it never took effect because of an alleged lack of good health at the time of delivery. It is therefore in the public interest that we are constrained to hold, as we do, that the delivery of the policy to the insured by an agent of the company who is authorized to make delivery or withhold delivery is the final act which binds the company and the insured, in the absence of fraud or other legal grounds for rescission. The fact that the agent to whom it has entrusted this duty is derelict or negligent or even dishonest in the performance of the duty which has been entrusted to him would create an obligation based upon the authorized acts of the agent toward a third party who was not in collusion with the agent.

Pacific Timber Export Corporation v. CA 112 SCRA 199 Facts:

- On March 13, 1963, Pacific secured temporary insurance from the Workemen’s Insurance Co. for its exportation of logs to Japan. Workmen issued on said date Cover Note 1010 insuring said cargo.

- The regular marine policies were issued by the company in favor of Pacific on Apr 2, 1963. The 2 marine policies bore the number 53H01032 and 53H01033.

- After the issuance of the cover note but BEFORE the issuance of the 2 policies, some of the logs intended to be exported were lost due to a typhoon.

- Pacific filed its claim with the company, but the latter refused, contending that said loss may not be considered as covered under the cover note because such became null and void by virtue of the issuance of the marine policies.

Issue: Whether or not the cover not was without consideration, thus null and void. Held: It was with consideration. SC upheld Pacific’s contention that said cover not was with consideration. The fact that no separate premium was paid on the cover note before the loss was insured

against occurred does not militate against the validity of Pacific’s contention, for no such premium could have been paid, since by the nature of the cover note, it did not contain, as all cover notes do not contain, particulars of the shipment that would serve as basis for the computation of the premiums. As a logical consequence, no separate premiums are required to be paid on a cover note. If the note is to be treated as a separate policy instead of integrating it to the regular policies subsequently issued, its purpose would be meaningless for it is in a real sense a contract, not a mere application.

Gloria v. Philamlife Insurance Co., 73 OG 8660 GLORIA V. PHILAMLIFE INSURANCE CO. 73 OG 8660 Facts:

- In 1966, Roberto Narito applied for a 100T life insurance policy with Philamlife Insurance Company. Narito was examined by Dra. Vergel de dios, the insurer’s medical examiner.

- She opined that Narito was insurable. Her opinion was confirmed by Dr. Orobia, the Associate Medical Director of the insurer.

- On Oct. 31, 1966, an agent of the insured prepared an application for the life insurance whose annual premium was P1,178. On the same date, the application was signed by Narito.

- Narito paid the first annual premium on the policy applied for. The insurer’s application form contained a so-called “Binding Receipt” which was detachable.

- It is not sure whether or not Narito was given the Binding Receipt upon his payment of the first premium, but what is certain is that he was handed a Cashier’s Receipt.

- From the time the insured received the application from its agent on Nov. 5, 1966, up to Dec. 6, 1966, it did not take any action with regard to the controverted insurance coverage.

- On Dec. 6, 1966, Narito was shot and killed. The beneficiaries submitted a claim to the insurer. After an underwriting analysis conducted by the insurer, it found out that Narito was unacceptable as an insurance risk. The claim was denied.

Issue: Whether or not the beneficiaries can claim. Held: YES. The application for insurance signed by the deceased contained the following stipulation: “The binding receipt must NOT be issued unless a binding deposit is paid which must be at least equal to the first full premium.” The preponderance of evidence is to the effect that the binding receipt was not issued to the deceased when he paid the company’s agent, the first annual premium of P1,178. Hence the rights of the beneficiaries and the obligation of the company have to be determined solely in the application for insurance and in the Cashier’s receipt.

Page 50: Insurance Case Digests (1)

Insurance Case Digests Atty. Bathan-Basuel

2-S, 2013-2014 50

The application for insurance contained the following clause:“There shall be no contract of insurance unless a policy is issued on this application and the full first premium thereon actually paid.” It should be conceded that there shall be a contract of insurance once the first premium is paid and a policy is issued. There is no question that the first premium was paid. The problem is to resolve whether or not it can be said that the policy has been issued. IN this connection, what may be noted is that, in contrast to the requirement of actual payment of the premium, it was NOT required that the policy be actually issued. And assuming that no policy had indeed been issued, it should still be held that the application for insurance was approved by the company, with the actual issuance of the policy being a mere technicality. When an insurer accepts and retains the first premium for an unreasonable length of time, it should be presumed that the insurer had assumed the risk. It should therefore be liable for loss before the application is subsequently rejected. In the case at bar, the company did NOT act on the application for insurance, one way or the other, from Nov. 2 to Dec. 5, 1966, and no justification for the delay had been proven. Hence, it should be held that the application for insurance of the deceased had been approved prior to his death, although the policy had not actually been issued, for which reason, the company should be liable to the beneficiaries.

Del Val v Del Val FRANCISCO DEL VAL, ET AL. vs. ANDRES DEL VAL GR NO. 9374. FEBRUARY 16, 1915 MORELAND FACTS: Plaintiffs and defendants are brothers and sisters, the only heirs at law and next of kin of Gregorio Nacianceno Del Val who died in Manila on August 4, 1910 intestate. An administrator was appointed for the estate of the decease, and, after a partial administration, it was closed and the administrator discharged by order of CFI Manila dated December 9, 1911. During the lifetime of the deceased, he took out insurance on his life for P40,000 and made it payable to the defendant as sole beneficiary. After his death, the defendant collected the face of the policy. Of said policy, he paid P18,365.20 to redeem certain real estate which the decedent had sold to third persons with a right to repurchase. The redemption of said premises was made by the attorney of the defendant in the name of the plaintiffs and the defendant as heirs of the deceased vendor. The redemption in the name of the plaintiffs was, so defendant declares, without his knowledge or consent. Since the redemption of said premises, they have been in the possession of the plaintiffs who have had the use and benefit thereof. During that time, plaintiffs paid no taxes and made no repairs.

Further, the defendant, on the death of the deceased, took possession of most of his personal property, which he still has in his possession, and that he has also the balance on said insurance policy amounting to P21,634.80. Plaintiffs contend that the amount of the insurance policy belonged to the estate of the deceased and not to the defendant personally, therefore, they are entitled to a partition not only of the real and personal property, but also of the P40,000 life insurance. The complaint prays a partition of all the property, both real and personal, left by the deceased, that the defendant account for P21,634.80 and that the sum be divided equally among the plaintiffs and defendant along with the other property of the deceased. The defendant denies the material allegations of the complaint and sets up as special defense and counterclaim that the redemption of the real estate sold by his father was made in the name of the plaintiffs and himself instead of in his name alone without his knowledge or consent and that it was not his intention to use the proceeds of the insurance policy for the benefit of any person but himself alleging that he was and is the sole owner thereof and it is his individual property. He asks that he be declared the owner of the real estate redeemed by the payment of the P18,365.20, the owner of the remaining P21,634.80, the balance of the insurance policy, and that the plaintiffs account for the use and occupation of the premises so redeemed since the date of the redemption. CFI Manila refused to give relief to either party and dismissed the action due to the complaint’s failure to comply with Code. Civ. Pro. Sec. 183 because it does not contain an adequate description of the real property of which partition is demanded; and since it is an exclusive real property action, the institution thereof gives the court no jurisdiction over chattels but no relief could possibly be granted in this action as to any property except the last (real estate), for the law contemplated that all the personal property of an estate be distributed before the administration is closed. Indeed, it is only in exceptional cases that the partition of the real estate is provided for, and this too is evidently intended to be effected as a part of the administration, but here the complaint alleges that the estate was finally closed on December 9, 1911, and we find upon referring to the record in that case that subsequent motions to reopen the same were denied; so that the matter of the distribution of the personal property at least must be considered res judicata (for the final judgment in the administration proceedings must be treated as concluding not merely what was adjudicated, but what might have been). So far, therefore, as the personal property at least is concerned, plaintiffs' only remedy was an appeal from said order. NOTE: Even though the complaint is defective to the extent of failing in allegations necessary to constitute a cause of action, if, on the trial of the cause, evidence is offered which establishes the cause of action which the complaint intended to allege, and such evidence is received without objection, the defect is thereby cured and cannot be made the ground of a subsequent objection. If, therefore, evidence was introduced on the trial in this case definitely and clearly describing the real estate sought to be

Page 51: Insurance Case Digests (1)

Insurance Case Digests Atty. Bathan-Basuel

2-S, 2013-2014 51

partitioned, the defect in the complaint was cured in that regard and should not have been used to dismiss the action. Courts do not stop to inquire whether such evidence was or was not introduced on the trial, inasmuch as this case must be returned for a new trial with opportunity to both parties to present such evidence as is necessary to establish their respective claims. The courts of the Islands have jurisdiction to divide personal property between the common owners thereof and that power is as full and complete as is the power to partition real property. If an actual partition of personal property cannot be made it will be sold under the direction of the court and the proceeds divided among the owners after the necessary expenses have been deducted. The administration of the estate of the decedent consisted simply in the payment of the debts. No division of the property, either real or personal, seems to have been made. On the contrary, the property appears to have been turned over to the heirs in bulk. The failure to partition the real property may have been due either to the lack of request to the court by one or more of the heirs to do so, as the court has no authority to make a partition of the real estate without such request; or it may have been due to the fact that all the real property of decedent had been sold under pacto de retro and that, therefore, he was not the owner of any real estate at the time of his death. As to the personal property, it does not appear that it was disposed of in the manner provided by law. (Sec. 753, Code of Civil Procedure.) So far as this action is concerned, however, it is sufficient that none of the property was actually divided among the heirs in the administration proceedings and that they remain co-owners and tenants-in-common thereof at the present time. To maintain an action to partition real or personal property it is necessary to show only that it is owned in common. The order finally closing the administration and discharging the administrator, referred to in the opinion of the trial court, has nothing to do with the division of either the real or the personal property. The heirs have the right to ask the probate court to turn over to them both the real and personal property without division; and where that request is unanimous it is the duty of the court to comply with it, and there is nothing in section 753 of the Code of Civil Procedure which prohibits it. In such case an order finally settling the estate and discharging the administrator would not bar a subsequent action to require a division of either the real or personal property. If, on the other hand, an order had been made in the administration proceedings dividing the personal or the real property, or both, among the heirs, then it is quite possible that, to a subsequent action brought by one of the heirs for a partition of the real or personal property, or both, there could have been interposed a plea of res judicata based on such order. As the matter now stands, however, there is no ground on which to base such a plea. Moreover, no such plea has been made and no evidence offered to support it. ISSUE: Whether the proceeds of the life- insurance policy belong exclusively to the defendant as his individual and separate property. The attorney for the plaintiffs claims that Sec. 428 of the Code of Commerce just q is subordinated to the provisions of the Civil Code as found in Art. 1035, which reads:

“An heir by force of law surviving with others of the same character to a succession must bring into the hereditary estate the property or securities he may have received from the deceased during the life of the same, by way of dowry, gift, or for any good consideration, in order to compute it in fixing the legal portions and in the account of the division." Counsel also claims that the proceeds of the insurance policy were a donation or gift made by the father during his lifetime to the defendant and that, as such, its ultimate destination is determined by those provisions of the Civil Code which relate to donations, especially Art. 819 which provides that "gifts made to children which are not betterments shall be considered as part of their legal portion”. HELD: YES. The proceeds of the l ife- insurance policy belong exclusively to the defendant as his individual and separate property. That the proceeds of an insurance policy belong exclusively to the beneficiary and not to the estate of the person whose life was insured, and that such proceeds are the separate and individual property of the beneficiary, and not of the heirs of the person whose life was insured, is the doctrine in America. The same doctrine obtains in the Islands by virtue of Section 428 of the Code of Commerce, which reads: "The amounts which the underwriter must deliver to the person insured, in fulfillment of the contract, shall be the property of the latter, even against the claims of the legitimate heirs or creditors of any kind whatsoever of the person who effected the insurance in favor of the former." The contract of l ife insurance is a special contract and the destination of the proceeds thereof is determined by special laws which deal exclusively with that subject. The Civil Code has no provisions which relate directly and specifically to l ife-insurance contracts or to the destination of l ife insurance proceeds. That subject is regulated exclusively by the Code of Commerce which provides for the terms of the contract, the relations of the parties and the destination of the proceeds of the policy. The proceeds of the life-insurance policy being the exclusive property of the defendant and he having used a portion thereof in the repurchase of the real estate sold by the decedent prior to his death with right to repurchase, and such repurchase having been made and the conveyance taken in the names of all of the heirs instead of the defendant alone, plaintiffs claim that the property belongs to the heirs in common and not to the defendant alone. This cannot hold true unless the fact appear or be shown that the defendant acted as he did with the intention that the other heirs should enjoy with him the ownership of the estate—in other words, that he proposed, in effect, to make a gift of the real estate to the other heirs. If it is established by the evidence that that was his intention and that the real estate was delivered to the plaintiffs with that understanding, then it

Page 52: Insurance Case Digests (1)

Insurance Case Digests Atty. Bathan-Basuel

2-S, 2013-2014 52

is probable that their contention is correct and that they are entitled to share equally with the defendant therein. If, however, it appears from the evidence in the case that the conveyances were taken in the name of the plaintiffs without his knowledge or consent, or that it was not his intention to make a gift to them of the real estate, then it belongs to him. If the f acts are as stated, he has two remedies. The one is to compel the plaintiffs to reconvey to him and the other is to let the title stand with them and to recover from them the sum he paid on their behalf. The case was REMANDED to the trial court with instructions to permit the parties to frame such issues as will permit the settlement of all the questions involved and to introduce such evidence as may be necessary for the full determination of the issues framed. Upon such issues and evidence taken thereunder the court will decide the questions involved according to the evidence, subordinating his conclusions of law to the rules laid down.

Bonifacio Bros. v. Mora, 20 SCRA 262 BONIFACIO BROS., INC., ET AL vs. ENRIQUE MORA, ET AL. Enrique Mora, owner of an Oldsmobile sedan model 1956 mortgaged the same to the H.S. Reyes, Inc., with the condition that the former would insure the automobile with the latter as beneficiary. The automobile was thereafter insured with the State Bonding & Insurance Co, Inc., and a motor car insurance was to Mora. The insurer obligated itself to indemnify the Insured against loss of or damages to the Motor Vehicle and its accessories and spare parts whilst thereon; (a) by accidental collision or overturning or collision or overturning consequent upon mechanical breakdown or consequent upon wear and tear. The Company may pay in cash the the amount the loss or damage or may repair, reinstate, or replace the Motor Vehicle or any part thereof or its accessories or spare parts. The liability of the Company shall not exceed the value of the parts whichever is the less. The Insured's estimate of value stated in the schedule will be the maximum amount payable by the Company in respect of any claim for loss or damage. The Insured may authorize the repair of the Motor Vehicle necessitated by damage for which the Company may be liable under this Policy provided that:—(a) The estimated cost of such repair does not exceed the Authorized Repair Limit, (b) A detailed estimate of the cost is forwarded to the Company without delay, subject to the condition that 'Loss, if any, is payable to H.S. Reyes, Inc..', by virtue of the fact that said Oldsmobile sedan was mortgaged in favor of the said H.S. Reyes, Inc. and that under a clause in said insurance policy, any loss was made payable to the H.S, Reyes, Inc. as Mortgagee; During the effectivity of the insurance contract, the car met with an accident. The insurance company then assigned the accident to the H.H. Bayne Adjustment Co. for investigation and appraisal of the damage. Enrique Mora, without the knowledge and consent of the H.S. Reyes, Inc., authorized the Bonifacio Bros. Inc. to furnish the labor and materials, some of which were supplied by the Ayala Auto Parts Co. For the cost of

labor and materials, Enrique Mora was billed at P2,102.73 through the H.H. Bayne Adjustment Co. The insurance company, after claiming a franchise in the amount of ?100, drew a check in the amount of P2,002.73, as proceeds of the insurance policy, payable to the order of Enrique Mora or H.S. Reyes, Inc., and entrusted the check to the H.H. Bayne Adjustment Co. for disposition and delivery to the proper party. In the meantime, the car was delivered to Enrique Mora without the consent of the H.S, Reyes, Inc., and without payment to the Bonifacio Bros. Inc. and the Ayala Auto Parts Co. of the cost of repairs and materials, Upon the theory that the insurance proceeds should be paid directly to them, the Bonifacio Bros. Inc. and the Ayala Auto Parts Co. a complaint with the against Enrique Mora and the State Bonding & Insurance Co., Inc. for the collection of the sum of P2,002.73. The insurance company filed its answer with a counterclaim for interpleader, requiring the Bonifacio Bros. Inc. and the H.S. Reyes, Inc. to interplead in order to determine who has better right to the insurance proceeds in question. Enrique Mora was declared in default for failure to appear at the hearing, and evidence against him was received ex parte. However, the counsel for the Bonifacio Bros. Inc., Ayala Auto Parts Co. and State Bonding & Insurance Co. Inc. submitted a stipulation of facts, on the basis of which the Municipal Court rendered a decision declaring the H.S. Reyes, Inc. as having a better right to the disputed amount, and ordering the State Bonding & Insurance Co. Inc. to pay to the H.S. Reyes, Inc. the said sum of P2,002.73. From this decision, the herein appellants elevated the case to the Court of First Instance of Manila before which the stipulation of facts was reproduced. The latter court rendered a decision, affirming the decision of the Municipal Court. The Bonifacio Bros. Inc. and the Ayala Auto Parts Co. moved for reconsideration of the decision, but the trial court denied the motion. Hence, this appeal. ISSUE Whether or not there is a privity of contract between Bonifacio Bros and Ayala Auto Parts on one hand and the insurance company on the other RULING The appellants argue that the insurance company and Enrique Mora are parties to the repair of the car as well as the towage thereof performed. The authority for this assertion is to be found, it is alleged, in paragraph 4 of the insurance contract which provides that "the insured may authorize the repair of the Motor Vehicle necessitated by damage for which the company may be liable under the policy provided that (a) the estimated cost of such repair does not exceed the Authorized Repair Limit, and (b) a detailed estimate of the cost is forwarded to the company without delay." It is stressed that the H.H. Bayne Adjustment Company's recommendation of payment of the appellants' bill for materials and repairs for which the latter drew a check for P2,002.73 indicates that Mora and the H.H. Bayne Adjustment Co. acted for and in representation of the insurance company. This argument is, in our view, beside the point, because from the undisputed facts and from the pleadings it will be seen that the appellants' alleged cause of action rests exclusively upon the terms of the insurance contract. The appellants seek to recover

Page 53: Insurance Case Digests (1)

Insurance Case Digests Atty. Bathan-Basuel

2-S, 2013-2014 53

the insurance proceeds, and for this purpose, they rely upon paragraph 4 of the insurance contract document executed by and between the State Bonding & Insurance Company, Inc. and Enrique Mora. The appellants are not mentioned in the contract as parties thereto; nor is there any clause or provision thereof from which we can infer that there is an obligation on the part of the insurance company to pay the cost of repairs directly to them. It is fundamental' that contracts take effect only between the parties thereto, except in some specific instances provided by law where the contract contains some stipulation in favor of a third person. Such stipulation is known as stipulation pour autrui or a provision in favor of a third person not a party to the contract. Under this doctrine, a third person is allowed to avail himself of a benefit granted to him by the terms of the contract, provided that the contracting parties have clearly and deliberately conferred a favor upon such person. Consequently, a third person not a party to the contract has no action against the parties thereto, and cannot generally demand the enforcement of the same. The question of whether a third person has an enforcible interest in a contract must be settled by determining whether the contracting parties intended to tender him such an interest by deliberately inserting terms in their agreement with the avowed purpose of conferring a favor upon such third person. In this connection, this Court has laid down the rule that the fairest test to determine whether the interest of a third person in a contract is a stipulation pour autrui or merely an incidental interest, is to rely upon the intention of the parties as disclosed by their contract. In the instant case the insurance contract does not contain any words or clauses to disclose an intent to give any benefit to any repairmen or materialmen in case of repair of the car in question. The parties to the insurance contract omitted such stipulation, which is a circumstance that supports the said conclusion. On the other hand, the "loss payable" clause of the insurance policy stipulates that "Loss, if any, is payable to H.S. Reyes, Inc." indicating that it was only the H.S. Reyes, Inc. which they intended to benefit. We likewise observe from the brief of the State Bonding & Insurance Company that it has vehemently opposed the assertion or pretension of the appellants that they are privy to the contract. If it were the intention of the insurance company to make itself liable to the repair shop or materialmen, it could have easily inserted in the contract a stipulation to that effect. To hold now that the original parties to the insurance contract intended to confer upon the appellants the benefit claimed by them would require us to ignore the indispensable requisite that a stipulation pour autrui must be clearly expressed by the parties, which we cannot do. As regards paragraph 4 of the insurance contract, a perusal thereof would show that instead of establishing privity between the appellants and the insurance company, such stipulation merely establishes the procedure that the insured has to follow in order to be entitled to indemnity for repair. This paragraph therefore should not be construed as bringing into existence in favor of the appellants a right of action against the insurance company as such intention can never be inferred therefrom. Another cogent reason for not recognizing a right of action by the appellants against the insurance company is that "a policy of insurance is a distinct and independent contract between the insured and insurer, and third persons have no right either in a

court of equity, or in a court of law, to the proceeds of it, unless there be some contract or trust, expressed or implied between the insured and third person." In this case, no contract of trust, expressed or implied exists. We, therefore, agree with the trial court that no cause of action exists in favor of the appellants in so far as the proceeds of insurance are concerned. The appellants' claim, if at all, is merely equitable in nature and must be made effective through Enrique Mora who entered into a contract with the Bonifacio Bros. Inc. This conclusion is deducible not only from the principle governing the operation and effect of insurance contracts in general, but is clearly covered by the express provisions of section 50 of the Insurance Act which read: "The insurance shall be applied exclusively to the proper interests of the person in whose name it is made unless otherwise specified in the policy." The policy in question has been so framed that "Loss, if any, is payable to H.S. Reyes, Inc.," which unmistakably shows the intention of the parties. The final contention of the appellants is that the right of the H.S. Reyes, Inc. to the insurance proceeds arises only if there was loss and not where there is mere damage as in the instant case. Suffice it to say that any attempt to draw a distinction between "loss" and "damage" is uncalled for, because the word "loss" in insurance law embraces injury or damage.

Coquia v. Fieldmen’s Insurance, 26 SCRA 172 FACTS: 1. Fieldmen's Insurance Company, Inc. issued, in favor of the Manila Yellow Taxicab

Co., Inc. a common carrier accident insurance policy, for a period of one year. It was stipulated in said policy that:

The Company will, subject to the Limits of Liability and under the Terms of this Policy, indemnify the Insured in the event of accident caused by or arising out of the use of Motor Vehicle against all sums which the Insured will become legally liable to pay in respect of: Death or bodily injury to any fare-paying passenger including the Driver, Conductor and/or Inspector who is riding in the Motor Vehicle insured at the time of accident or injury.

2. Thereafter, a taxicab of the Insured, driven by Carlito Coquia, met a vehicular accident at Mangaldan, Pangasinan, in consequence of which Carlito died.

3. Thus, The Manila Yellow Taxicab filed therefor a claim for P5,000.00 to which the Fieldmen’s Insurance replied with an offer to pay P2,000.00, by way of compromise. The Insured rejected the same and made a counter-offer for P4,000.00, but the Fieldman did not accept it.

Page 54: Insurance Case Digests (1)

Insurance Case Digests Atty. Bathan-Basuel

2-S, 2013-2014 54

4. Hence,Yellow Taxicab and Carlito's parents (Melecio Coquia and Maria Espanueva) filed a complaint against the Fieldmen’s Insurance to collect the proceeds of the policy.

5. ANSWER OF Fieldmen’s Insurance: admitted the existence of the policy, but pleaded lack of cause of action on the part of the plaintiffs (parents).

Decision of TC: sentencing the Fieldmen’s Insurance to pay to the plaintiffs the sum of P4,000.00 and the costs.

Contentions of Fieldmen’s Insurance before C.A - contends that plaintiffs (parents) have no cause of action because: 1) the Coquias have no contractual relation with them and 2) the Insured has not complied with the provisions of the policy concerning arbitration.

ISSUES: Whether the plaintiffs (parents) have a cause of action against Fielmen’s Insurance.

Whether the insured has complied with the provisions of the policy concerning arbitration.

HELD:

(1)YES.

It should be noted that, although, in general, only parties to a contract may bring an action based thereon, this rule is subject to exceptions, one of which is found in the second paragraph of Article 1311 of the Civil Code:

If a contract should contain some stipulation in favor of a third person, he may demand its fulfillment provided he communicated his acceptance to the obligor before its revocation. A mere incidental benefit or interest of a person is not sufficient. The contracting parties must have clearly and deliberately conferred a favor upon a third person.

This is but the restatement of a well-known principle concerning contracts pour autrui, the enforcement of which may be demanded by a third party for whose benefit it was made, although not a party to the contract, before the stipulation in his favor has been revoked by the contracting parties.

Pursuant to these stipulations, the Company "will indemnify any authorized Driver who is driving the Motor Vehicle" of the Insured and, in the event of death of said driver, the Company shall, likewise, "indemnify his personal representatives." In fact, the Company "may, at its option, make indemnity payable directly to the claimants or heirs of claimants ... it being the true intention of this Policy to protect ... the liabilities of the

Insured towards the passengers of the Motor Vehicle and the Public" — in other words, third parties.

Thus, the policy under consideration is typical of contracts pour autrui, this character being made more manifest by the fact that the deceased driver paid fifty percent of the corresponding premiums, which were deducted from his weekly commissions. Under these conditions, it is clear that the Coquias — who, admittedly, are the sole heirs of the deceased — have a direct cause of action against the Company, and, since they could have maintained this action by themselves, without the assistance of the Insured, it goes without saying that they could and did properly join the latter in filing the complaint herein.

(2) BOTH PARTIES WAIVED SUCH PROVISION.

The second defense set up by the Company is based upon Section 17 of the policy reading:

If any difference or dispute shall arise with respect to the amount of the Company's liability under this Policy, the same shall be referred to the decision of a single arbitrator to be agreed upon by both parties or failing such agreement of a single arbitrator, to the decision of two arbitrators, one to be appointed in writing by each of the parties within one calendar month after having been required in writing so to do by either of the parties and in case of disagreement between the arbitrators, to the decision of an umpire who shall have been appointed in writing by the arbitrators before entering on the reference and the costs of and incident to the reference shall be dealt with in the Award. And it is hereby expressly stipulated and declared that it shall be a condition precedent to any right of action or suit upon this Policy that the award by such arbitrator, arbitrators or umpire of the amount of the Company's liability hereunder if disputed shall be first obtained.

The record shows, however, that none of the parties to the contract invoked this section, or made any reference to arbitration, during the negotiations preceding the institution of the present case. In fact, counsel for both parties stipulated, in the trial court, that none of them had, at any time during said negotiations, even suggested the settlement of the issue between them by arbitration, as provided in said section. Their aforementioned acts or omissions had the effect of a waiver of their respective right to demand an arbitration.

The test for determining whether there has been a waiver in a particular case is stated by the author of an exhaustive annotation in 117 A.L.R. p. 304, as follows: "Any conduct of the parties inconsistent with the notion that they treated the arbitration provision as in effect, or any conduct which might be reasonably construed as showing

Page 55: Insurance Case Digests (1)

Insurance Case Digests Atty. Bathan-Basuel

2-S, 2013-2014 55

that they did not intend to avail themselves of such provision, may amount to a waiver thereof and estop the party charged with such conduct from claiming its benefits".

The decisive facts here are that both parties from the inception of their dispute proceeded in entire disregard of the provisions of the contract relating to arbitration and that neither at any stage of such dispute, either before or after commencement of the action, demanded arbitration, either by oral or written demand, pleading, or otherwise. Their conduct was as effective a rejection of the right to arbitrate as if, in the best Coolidge tradition, they had said, "We do not choose to arbitrate". As arbitration under the express provisions of article 40 was "at the choice of either party," and was chosen by neither, a waiver by both of the right to arbitration followed as a matter of law.

Guingon v. Del Monte 20 SCRA 1043

GUINGON v DEL MONTE

FACTS: Julio Aguilar owned and operated several jeepneys in the City of Manila. He entered into a contract with the Capital Insurance & Surety Co., Inc. insuring the operation of his jeepneys against accidents with third-party liability. The pertinent provisions of which in so far as this case is concerned contains the following:

Section II —LIABILITY TO THE PUBLIC

1. The Company, will, subject to the limits of liability, indemnify the Insured in the event of accident caused by or arising out of the use of the Motor Vehicle/s or in connection with the loading or unloading of the Motor Vehicle/s, against all sums including claimant's costs and expenses which the Insured shall become legally liable to pay in respect of:

a. death of or bodily injury to any person

b. damage to property

During the effectivity of such insurance policy on February 20, 1961 Iluminado del Monte, one of the drivers of the jeepneys operated by Aguilar, while driving along the intersection of Juan Luna and Moro streets, City of Manila, bumped with the jeepney one Gervacio Guingon who had just alighted from another jeepney and as a consequence the latter died some days thereafter.

A corresponding information for homicide thru reckless imprudence was filed against Iluminado del Monte, who pleaded guilty.

Heirs of Gervacio Guingon filed an action for damages praying that the sum of P82,771.80 be paid to them jointly and severally by Iluminado del Monte, owner and operator Julio Aguilar, and the Capital Insurance & Surety Co.. Capital Insurance & Surety Co., Inc. answered, alleging that the plaintiff has no cause of action against it.

the Court of First Instance of Manila rendered its judgment sentencing defendants liable.

The case was appealed to the Court of Appeals which appellate court certified the case to the SC because the appeal raises purely questions of law.

ISSUE: Plaintiffs not being parties to the insurance contract, do they have a cause of action against the company;

HELD: YES.

The policy contains a clause stating:

E. Action Against Company

No action shall lie against the Company unless, as a condition precedent thereto, the Insured shall have fully complied with all of the terms of this Policy, nor until the amount of the Insured's obligation to pay shall have been finally determined either by judgment against the Insured after actual trial or by written agreement of the Insured, the claimant, and the Company.

The policy in the present case, as aforequoted, is one whereby the insurer agreed to indemnify the insured "against all sums . . . which the Insured shall become legally liable to pay in respect of: a. death of or bodily injury to any person . . . ." Clearly, therefore, it is one for indemnity against liability;1 from the fact then that the insured is liable to the third person, such third person is entitled to sue the insurer.1äwphï1.ñët

the test applied has been this: Where the contract provides for indemnity against liability to third persons, then third persons to whom the insured is liable, can sue the insurer. Where the contract is for indemnity against actual loss or payment, then third persons cannot proceed against the insurer, the contract being solely to reimburse the insured for liability actually discharged by him thru payment to third persons, said third persons' recourse being thus limited to the insured alone.2

The "no action" clause in the policy of insurance cannot prevail over the Rules of Court provision aimed at avoiding multiplicity of suits.

RCBC v. CA, 289 SCRA 292 (1998)

Page 56: Insurance Case Digests (1)

Insurance Case Digests Atty. Bathan-Basuel

2-S, 2013-2014 56

Rizal Commercial Banking Corporation (RCBC) vs. Court of Appeals [GR 128833, 20 April 1998]; Facts:

Goyu & Sons, Inc. (Goyu) applied for credit facilities and accommodations with Rizal Commercial Banking Corporation (RCBC) at its Binondo Branch. After due evaluation, RCBC Binondo Branch, through its key officers, petitioners Uy Chun Bing and Eli D. Lao, recommended Goyu's application for approval by RCBC's executive committee. A credit facility in the amount of P30 million was initially granted. Upon Goyu's application and Uy's and Lao's recommendation, RCBC's executive committee increased Goyu's credit facility to P50 million, then to P90 million, and finally to P117 million.

As security for its credit facilities with RCBC, Goyu executed two real estate mortgages and two chattel mortgages in favor of RCBC, which were registered with the Registry of Deeds at Valenzuela, Metro Manila. Under each of these four mortgage contracts, Goyu committed itself to insure the mortgaged property with an insurance company approved by RCBC, and subsequently, to endorse and deliver the insurance policies to RCBC. Goyu obtained in its name a total of 10 insurance policies from MICO.

In February 1992, Alchester Insurance Agency, Inc., the insurance agent where Goyu obtained the Malayan insurance policies, issued 9 endorsements in favor of RCBC seemingly upon instructions of Goyu. On 27 April 1992, one of Goyu's factory buildings in Valenzuela was gutted by fire. Consequently, Goyu submitted its claim for indemnity on account of the loss insured against. MICO denied the claim on the ground that the insurance policies were either attached pursuant to writs of attachments/garnishments issued by various courts or that the insurance proceeds were also claimed by other creditors of Goyu alleging better rights to the proceeds than the insured.

Goyu filed a complaint for specific performance and damages which was docketed at the Regional Trial Court of the National Capital Judicial Region (Manila, Branch 3) as Civil Case 93-65442. RCBC, one of Goyu's creditors, also filed with MICO its formal claim over the proceeds of the insurance policies, but said claims were also denied for the same reasons that AGCO denied Goyu's claims. In an interlocutory order dated 12 October 1993, the Regional Trial Court of Manila (Branch 3), confirmed that Goyu's other creditors, namely, Urban Bank, Alfredo Sebastian, and Philippine Trust Company obtained their respective writs of attachments from various courts, covering an aggregate amount of P14,938,080.23, and ordered that the proceeds of the 10 insurance policies be deposited with the said court minus the aforementioned P14,938,080.23. Accordingly, on 7 January 1994, MICO deposited the amount of P50,505,594.60 with Branch 3 of the Manila RTC.

In the meantime, another notice of garnishment was handed down by another Manila RTC sala (Branch 28) for the amount of P8,696,838.75. After trial, Branch 3 of the Manila RTC rendered judgment in a favor of Goyu, ordering Malayan to pay Goyu its fire loss claims in the total amount of P74,040,518.58 less the amount of P50,000,000.00 which is deposited with the Court; damages by way of interest for the duration of the delay since 27 July 1992 (90 days after Malayan's receipt of the required proof of loss and notice of loss) at the rate of twice the ceiling prescribed by the Monetary Board, on the amounts of (1) P50,000,000.00 from 27 July 1992 up to the time said amount was deposited with the Court on 7 January 1994; and (2)

P24,040,518.58 — from 17 July 1992 up to the time when the writs of attachments were received by Malayan. The court also ordered RCBC to pay Goyu actual and compensatory damages in the amount of P2,000,000.00, and both Malayan and RCBC to solidarily pay Goyu (1) P1,000,000.00 as exemplary damages; (2) P1,000,000.00 as, and for, attorneys fees; and (3) Costs of suit. The Court, on the Counterclaim of RCBC, ordered Goyu to pay its loan obligations with RCBC in the amount of P68,785,069.04, as of 27 April 1992, with interest thereon at the rate stipulated in the respective promissory notes (without surcharges and penalties).

From this judgment, all parties interposed their respective appeals.

Issue: Whether RCBC, as mortgagee, has any right over the insurance policies taken by Goyu, the mortgagor, in case of the occurrence of loss Held: YES. It is settled that a mortgagor and a mortgagee have separate and distinct insurable interests in the same mortgaged property, such that each one of them may insure the same property for his own sole benefit. There is no question that Goyu could insure the mortgaged property for its own exclusive benefit. Herein, although it appears that Goyu obtained the subject insurance policies naming itself as the sole payee, the intentions of the parties as shown by their contemporaneous acts, must be given due consideration in order to better serve the interest of justice and equity. It is to be noted that nine endorsement documents were prepared by Alchester in favor of RCBC. The Court is in a quandary how Alchester could arrive at the idea of endorsing any specific insurance policy in favor of any particular beneficiary or payee other than the insured had not such named payee or beneficiary been specifically disclosed by the insured itself.

It is also significant that Goyu voluntarily and purposely took the insurance policies from MICO, a sister company of RCBC, and not just from any other insurance company. Alchester would not have found out that the subject pieces of property were mortgaged to RCBC had not such information been voluntarily disclosed by Goyu itself. Had it not been for Goyu, Alchester would not have known of Goyu's intention of obtaining insurance coverage in compliance with its undertaking in the mortgage contracts with RCBC, and verify, Alchester would not have endorsed the policies to RCBC had it not been so directed by Goyu. On equitable principles, particularly on the ground of estoppel, the Court is constrained to rule in favor of mortgagor RCBC.

RCBC, in good faith, relied upon the endorsement documents sent to it as this was only pursuant to the stipulation in the mortgage contracts. Such reliance is justified under the circumstances of the case. Goyu failed to seasonably repudiate the authority of the person or persons who prepared such endorsements. Over and above this, Goyu continued, in the meantime, to enjoy the benefits of the credit facilities extended to it by RCBC. After the occurrence of the loss insured against, it was too late for Goyu to disown the endorsements for any imagined or contrived lack of authority of Alchester to prepare and issue said endorsements. If there had not been actually an implied ratification of said endorsements by virtue of Goyu's inaction in this case, Goyu is at the very least estopped from assailing their operative effects.

To permit Goyu to capitalize on its non- confirmation of these endorsements while it continued to enjoy the benefits of the credit facilities of RCBC which believed in good faith that there was due endorsement pursuant to their mortgage contracts, is to

Page 57: Insurance Case Digests (1)

Insurance Case Digests Atty. Bathan-Basuel

2-S, 2013-2014 57

countenance grave contravention of public policy, fair dealing, good faith, and justice. Such an unjust situation, the Court cannot sanction. Under the peculiar circumstances, the Court is bound to recognize RCBC's right to the proceeds of the insurance policies if not for the actual endorsement of the policies, at least on the basis of the equitable principle of estoppel.

Ang v. Fulton Fire Ins Co 2 SCRA 945

FACTS: On September 9, 1953, defendant Fulton Fire Insurance Company issued a policy No. F-4730340, in favor of P. & S Department Store (Sally C. Ang) over stocks of general merchandise, consisting principally of dry goods, contained in a building occupied by the plaintiffs at Laoag, Ilocos Norte.

The premium is P500.00 annually. The insurance was issued for one year, but the same was renewed for another year on September 31, 1954. On December 17, 1954, the store containing the goods insured was destroyed by fire. On December 30, following, plaintiffs executed the first claim form. The claim together with all the necessary papers relating thereto, were forwarded to the Manila Adjustment Company, the defendants' adjusters and received by the latter on Jane 8, 1955.

On April 6, 1956, the Fulton Fire Insurance Company wrote the plaintiffs that their claim was denied. This denial of the claim was received by the plaintiffs on April 19, 1956.

On January 13, 1955, plaintiff Paulo Ang and ten others were charged for arson but they were subsequently acquitted.

The present action was instituted on May 5, 1958.

The action was originally instituted against both the Fulton Fire Insurance Company and the Paramount Surety and Insurance Company, Inc., but on June 16, 1958, upon motion of the Paramount Surety, the latter was dropped from the complaint.

On May 26, 1958, the defendant Fulton Fire Insurance Company filed an answer to the complaint, admitting the existence of the contract of insurance, its renewal and the loss by fire of the department store and the merchandise contained therein, but denying that the loss by the fire was accidental, alleging that it was occasioned by the willful act of the plaintiff Paulo Ang himself.

It claims that under paragraph 13 of the policy, if the loss or damage is occasioned by the willful act of the insured, or if the claim is made and rejected but no action is commenced within 12 months after such rejection, all benefits under the policy would be forfeited, and that since the claim of the plaintiffs was denied and plaintiffs received notice of denial on April 18, 1956, and they brought the action only on May 5, 1958, all the benefits under the policy have been forfeited.

On February 12, 1959, plaintiffs filed a reply to the above answer of the Fulton Fire Insurance, alleging that on May 11, 1956, plaintiffs had instituted Civil Case No. 2949 in the Court of First Instance of Manila, to assert the claim; that this case was dismissed without prejudice on September 3, 1957 and that deducting the period within which said action was pending, the present action was still within the 12 month period from April 12, 1956.

The trial court held that the bringing of the action tolled the prescription ruling:

The complaint, Exh. 'C', was dismissed by the Court without prejudice (Exh. 'H-1') on September 3, 1957, and motion for reconsideration dated September 21, 1957. The instant complaint was filed on May 8, 1958. As correctly pointed out by the plaintiffs' counsel, by simple mathematical computation, the present action was filed leas thin nine (9) months after the notice of rejection received by plaintiffs on April 19, 1956, because the filing of the original complaint stopped the running of the period."

The clause subject of the issue is paragraph 13 of the policy, which reads as follows:

13. If the claim be in any respect fraudulent, or if any false declaration is made or used in support thereof, or if any fraudulent means or devices are used by the Insured or any one acting on his behalf to obtain any benefit under this Policy, or, if the loss or damage be occasioned by the willful act or with connivance of the Insured, or, if the claim be made and rejected and an action or suit be not commenced within twelve months after such rejection or (in case of arbitration place in pursuance of the 18th condition of this Policy) within twelve months after the arbitrator or arbitrators or umpire shall have made their award, all benefits under this Policy shall be forfeited. (Emphasis supplied). (Decision. p. 10, R.O.A.).

HELD:

The basic error committed by the trial court is its view that the filing of the action against the agent of the defendant company was "merely a procedural mistake of no significance or consequence, which may be overlooked." The condition contained in the insurance policy that claims must be presented within one year after rejection is not merely a procedural requirement. The condition is an important matter, essential to a prompt settlement of claims against insurance companies, as it demands that insurance suits be brought by the insured while the evidence as to the origin and cause of destruction have not yet disappeared. It is in the nature of a condition precedent to the liability of the insurer, or in other terms, a resolutory cause, the purpose of which is to terminate all liabilities in case the action is not filed by the insured within the period stipulated.

Page 58: Insurance Case Digests (1)

Insurance Case Digests Atty. Bathan-Basuel

2-S, 2013-2014 58

The bringing of the action against the Paramount Surety & Insurance Company, the agent of the defendant Company cannot have any legal effect except that of notifying the agent of the claim. Beyond such notification, the filing of the action can serve no other purpose. There is no law giving any effect to such action upon the principal. Besides, there is no condition in the policy that the action must be filed against the agent, and this Court can not by interpretation, extend the clear scope of the agreement beyond what is agreed upon by the parties.

Eagle Star Ins Co Ltd v Chia Yu 96 Phil 966 EAGLE STAR INSURANCE CO., LTD. VS. CHIA YU G.R. No. L-5915 March 31, 1955 REYES, A., J.: FACTS:

On January 15, 1946, Atkins, Kroll & Co., loaded on the S. S. Roeph Silverlight owned and operated by Leigh Hoegh & Co., A/S, of San Francisco, California, 14 bales of assorted underwear valued at P8,085.23 consigned to Chia Yu in the City of Manila. The shipment was insured against all risks by Eagle Star Ins. Co. of San Francisco, California, under a policy issued to the shipper and by the latter assigned to the consignee. The vessel arrived in Manila on February 10, 1946, and on March 4 started discharging its cargo into the custody of the Manila Terminal Co., Inc., which was then operating the arrastre service for the Bureau of Customs. But of the 14 bales consigned to Chia Yu only 10 were delivered to him as the remaining 4 could not be found. Three of those delivered were also found damaged to the extent of 50 per cent.

Chia Yu claimed indemnity for the missing and damaged bales. But the claim was declined, first, by the carrier and afterward by the insurer, whereupon Chia Yu brought the present action against both, including their respective agents in the Philippines. Commenced in the Court of First Instance of Manila on November 16, 1948, or more than two years after delivery of the damaged bales and the date when the missing bales should have been delivered, the action was resisted by the defendants principally on the ground of prescription. But the trial court found for plaintiff and rendered judgment in his favor for the sum claimed plus legal interest and costs. The judgment was affirmed by the Court of Appeals.

Under our law the time limit for bringing a civil action upon a written contract

is ten years after the right of action accrues. But counsel for the insurer claim that this statutory limitation must yield to the following stipulation in the policy:

"No suit or action on this Policy, for the recovery of any claim, shall be sustainable in any Court of law or equity unless the insured shall have fully complied with all the terms and conditions of this Policy nor unless commenced within twelve (12) months next after the happening of the loss * * *"

ISSUE:

Whether or not the action to claim the proceeds from the insurance policy issued by Eagle Star Insurance Company has already prescribed. HELD:

The case for the insurer stands on a different footing from that of the shipping company, for its claim of prescription is founded upon the terms of the policy and not upon the bill of lading.

In the case of E. Macias & Co. vs. China Fire Insurance & Co., Ltd., et al., 46 Phil. 345, relied upon by the insurer, this Court held that a clause in an insurance policy providing that an action upon the policy by the insured must be brought within a certain time is, if reasonable, valid and will prevail over statutory limitations of the action. That decision, however, was rendered before the passage of Act 4101, which amended the Insurance Act by inserting the following section in chapter one thereof:

"SEC. 61-A. Any condition, stipulation or agreement in any policy of insurance, limiting the time for commencing' an action thereunder to a period of less than one year from the time when the cause of action accrues, is void."

Examining the policy sued upon in the present case, we find that its

prescriptive clause, if given effect in accordance with the terms of the policy, would reduce the period allowed the insured for bringing his action to less than one year. This is so because the said clause makes the prescriptive period begin from the happening of the loss and at the same time provides that no suit on the policy shall be sustainable in any court unless the insured shall have first fully complied with all the terms and conditions of the policy, among them that which requires that, as soon as the loss is determined, written claim therefor be filed with the carrier and that the letter to the carrier and the latter's reply should be attached to the claim papers to be sent to the insurer. It is obvious that compliance with this condition precedent will necessarily consume time and thus shorten the period for bringing suit to less than one year if the period is to begin, as stated in the policy, from "the happening of the loss" Being contrary to the law of the forum, such stipulation cannot be given effect.

In concluding, we may state that contractual limitations contained in insurance policies are regarded with extreme jealousy by courts and will be strictly construed against the insurer and should not be permitted to prevent a recovery when their just and honest application would not produce that result.

Sun Life Office Ltd v CA 195 SCRA 193 SUN INSURANCE OFFICE LTD. V CA (TAN) 195 SCRA 193 PARAS; March 13, 1991 NATURE Petition for certiorari to review the decision of the CA

Page 59: Insurance Case Digests (1)

Insurance Case Digests Atty. Bathan-Basuel

2-S, 2013-2014 59

FACTS - Private respondent Emilio Tan took from the petitioner a Peso 300,000 property insurance policy to cover his interest in the electrical insurance store of his brother housed in a building in Iloilo City on August 15, 1983. Four days after the issuance of the policy, the building including the insured store burned. - On August 20, 1983, Tan filed his claim for fire loss. Sun Insurance, on February 29, 1984, wrote the private respondent denying the claim. On April 3, 1984, private respondent wrote another letter to the insurance company requesting reconsideration of the denial. Tan’s lawyer wrote another letter to the insurance company inquiring about the April 3 letter which sought for a reconsideration of the denial. In its reply to the lawyer’s letter, Sun Insurance reiterated its denial of the claim and enclosed therein copies of the two previous denials dated February 29, 1984 and May 17, 1985. - On November 20, 1985, Tan filed a civil case with the RTC. Petition filed a motion to dismiss on the alleged ground that the action has already prescribed based on Condition 27 of the Insurance Policy which stated that the window to file the appropriate action with either the Insurance Commission or in any court of competent jurisdiction is twelve months from the rejection of the claim. RTC denied the motion and the subsequent motion for reconsideration. The CA likewise denied the petition of Sun Insurance. ISSUE 1. WON the court the filing of a motion for reconsideration interrupts the 12 months prescription period to contest the denial of the insurance claim 2. WON the rejection of the claim shall be deemed final only if it contains words to the effect that the denial is final HELD 1. NO - The SC held that Condition 27 of the Insurance policy is very clear and free from any doubt or ambiguity. It has to be taken in its plain, ordinary, and popular sense. The rejection letter of February 29, 1984 was clear and plain. The Court noted that the one year period is likewise in accord with Section 23 of the Insurance Code which states that any condition which limits the time for commencing an action to a period of less than one year when the cause of action accrues is void. The right of action, according to the SC, accrues at the time that the claim is rejected at the first instance. A request for reconsideration of the denial cannot suspend the running of the prescriptive period. The Court noted that the rationale for the one year period is to ensure that the evidence as to the origin and cause of the destruction have not yet disappeared. 2. NO - The Court clarified its ruling in Eagle Star Insurance Co. vs Chia Yu where it ruled that “the cause of action in an insurance contract does not accrue until the Insured’s claim is finally rejected by the Insurer” by stating the use of the word “finally” cannot be construed to mean the rejection of a petition for reconsideration. What the court referred to in effect is the rejection in the first instance as claimed by Sun Insurance Disposition The decision of the CA is reversed and set aside. The case is dismissed

Filipino Merchants Insurance Co Inc V CA 179 SCRA 638

G.R. No. 85141 November 28, 1989

FILIPINO MERCHANTS INSURANCE CO., INC., petitioner, vs. COURT OF APPEALS and CHOA TIEK SENG, respondents.

On the issue of misrepresentation of lack of insurable interest. FACTS:

§ Choa Tiek Seng, consignee of the shipment of fishmeal loaded, insured in "all risks policy" 600 metric tons of fishmeal in new gunny bags of 90 kilos each from Bangkok, Thailand to Manila against all risks under warehouse to warehouse terms but only 59.940 metric tons was imported

§ When it was unloaded unto the arrastre contractor E. Razon, Inc. and Filipino Merchants' surveyor ascertained and certified that in such discharge 105 bags were in bad order condition which was reflected in the survey report of Bad Order cargoes

§ Before delivery to Choa, E. Razon's Bad Order Certificate showed that a total of 227 bags in bad order condition

§ Choa brought an action against Filipino Merchants Insurance Co. who brought a third party complaint against Compagnie Maritime Des Chargeurs Reunis and/or E. Razon, Inc.

§ RTC: Ordered Filipino Merchants to pay Choa and reimburse from Compagnie Maritime Des Chargeurs Reunis and third party defendant E. Razon, Inc.

§ CA: Affirmed but modified by adjudicating the third party complaint § Filipino Merchants contended that Chao has no insurable interest and therefore

the policy should be void and that it was fraud that it did not disclose of such fact

ISSUE: W/N Choa Tiek Seng as consignee of the shipment has insurable interest HELD: YES. CA affirmed.

§ Filipino contends that Chao does not have insurable interest, being only a consignee of the goods.

§ Anent the issue of insurable interest, SC upheld the ruling of the CA that Chao, as consignee of the goods in transit under an invoice containing the terms under "C & F Manila," has insurable interest in said goods.

§ Section 13 of the Insurance Code defines insurable interest in property as every interest in property, whether real or personal, or any relation thereto, or liability in

Page 60: Insurance Case Digests (1)

Insurance Case Digests Atty. Bathan-Basuel

2-S, 2013-2014 60

respect thereof, of such nature that a contemplated peril might directly damnify the insured. In principle, anyone has an insurable interest in property who derives a benefit from its existence or would suffer loss from its destruction whether he has or has not any title in, or lien upon or possession of the property. Insurable interest in property may consist in (a) an existing interest; (b) an inchoate interest founded on an existing interest; or (c) an expectancy, coupled with an existing interest in that out of which the expectancy arises.

§ Chao, as vendee/consignee of the goods in transit has such existing interest

therein as may be the subject of a valid contract of insurance. His interest over the goods is based on the perfected contract of sale. The perfected contract of sale between him and the shipper of the goods operates to vest in him an equitable title even before delivery or before he performed the conditions of the sale. The contract of shipment, whether under F.O.B., C.I.F., or C. & F. as in this case, is immaterial in the determination of whether the vendee has an insurable interest or not in the goods in transit. The perfected contract of sale even without delivery vests in the vendee an equitable title, an existing interest over the goods sufficient to be the subject of insurance

§ Article 1523 of the Civil Code provides that where, in pursuance of a contract of

sale, the seller is authorized or required to send the goods to the buyer, delivery of the goods to a carrier, whether named by the buyer or not, for, the purpose of transmission to the buyer is deemed to be a delivery of the goods to the buyer, the exceptions to said rule not obtaining in the present case. The Court has heretofore ruled that the delivery of the goods on board the carrying vessels partake of the nature of actual delivery since, from that time, the foreign buyers assumed the risks of loss of the goods and paid the insurance premium covering them

§ C & F contracts are shipment contracts. The term means that the price fixed

includes in a lump sum the cost of the goods and freight to the named destination. It simply means that the seller must pay the costs and freight necessary to bring the goods to the named destination but the risk of loss or damage to the goods is transferred from the seller to the buyer when the goods pass the ship's rail in the port of shipment.

§ Moreover, the issue of lack of insurable interest was not among the defenses

averred in petitioners answer.

Mayer Steel Pipe Corporation V CA 274 SCRA 432 G.R. No. 124050; June 19, 1997 MAYER STEEL PIPE CORPORATION and HONGKONG GOVERNMENT SUPPLIES DEPARTMENT, petitioners vs. CA, SOUTH SEA SURETY AND INSURANCE CO., INC. and the CHARTER INSURANCE CORPORATION, respondents PUNO, J.:

FACTS: In 1983, Hongkong contracted Mayer to manufacture and supply various steel pipes and fittings. From August to October, 1983, Mayer shipped the same to Hongkong as evidenced by Invoices. Prior to the shipping, Mayer insured the pipes and fittings against all risks with South Sea and Charter. Mayer and Hongkong jointly appointed Industrial Inspection (International) Inc. as third-party inspector, which certified all the pipes and fittings to be in good order condition before they were loaded in the vessel. Nonetheless, when the goods reached Hongkong, it was discovered that a substantial portion was damaged. A claim for indemnity was filed under the insurance contract. Charter paid Hongkong HK$64,904.75. The latter and Mayer demanded payment of the balance of HK$299,345.30 but was refused because the insurance surveyor's report allegedly showed that the damage is a factory defect. In 1986, an action to recover the sum of HK$299,345.30 was filed. The trial court ruled in favor of Mayer and Hongkong. CA affirmed that the damage is not due to factory defect and that it was covered by the "all risks" insurance policies. However, it set aside the decision of the trial court and dismissed the complaint on the ground of prescription. It held that the action is barred under Section 3(6) of the Carriage of Goods by Sea Act since it was filed only on April 17, 1986, more than two years from the time the goods were unloaded from the vessel. Section 3(6) of the Carriage of Goods by Sea Act provides that "the carrier and the ship shall be discharged from all liability in respect of loss or damage unless suit is brought within one year after delivery of the goods or the date when the goods should have been delivered." Respondent court ruled that this provision applies not only to the carrier but also to the insurer, citing Filipino Merchants Insurance Co., Inc. vs. Alejandro. Hence, this petition. ISSUE: Whether the CA erred in holding that the cause of action had already prescribed on the mistaken application of the Carriage of Goods by Sea Act and thus dismissing the complaint. HELD: YES. CA erred in applying Section 3(6) of the Carriage of Goods by Sea Act. Under this provision, only the carrier's liability is extinguished if no suit is brought within one year. But the liability of the insurer is not extinguished because the insurer's liability is based not on the contract of carriage but on the contract of insurance. The Carriage of Goods by Sea Act governs the relationship between the carrier on the one hand and the shipper, the consignee and/or the insurer on the other hand. It defines the obligations of the carrier under the contract of carriage. It does not, however, affect the relationship between the shipper and the insurer. The latter case is governed by the Insurance Code. The Filipino Merchants case is different from the case at bar. In Filipino Merchants, it was the insurer which filed a claim against the carrier for reimbursement of the amount

Page 61: Insurance Case Digests (1)

Insurance Case Digests Atty. Bathan-Basuel

2-S, 2013-2014 61

it paid to the shipper. In the case at bar, it was the shipper which filed a claim against the insurer. The ruling in Filipino Merchants should apply only to suits against the carrier filed either by the shipper, the consignee or the insurer. The obligation under an all-risk policy prescribes in ten years, in accordance with Article 1144 of the New Civil Code. Petition GRANTED.

Malayan Ins Co v. Cruz Arnaldo 154 SCRA 672 MALAYAN INSURANCE CO., INC. (MICO), petitioner, vs. GREGORIA CRUZ ARNALDO, in her capacity as the INSURANCE COMMISSIONER, and CORONACION PINCA, respondents. FACTS:

• On June 7, 1981, the petitioner (hereinafter called (MICO) issued to the private respondent, Coronacion Pinca, Fire Insurance Policy No. F-001-17212 on her property for the amount of P100,000.00, effective July 22, 1981, until July 22, 1982.

• On October 15, 1981, MICO allegedly cancelled the policy for non-payment, of the premium and sent the corresponding notice to Pinca.

• On December 24, 1981, payment of the premium for Pinca was received by Domingo Adora, agent of MICO.

• On January 15, 1982, Adora remitted this payment to MICO, together with other payments.

• On January 18, 1982, Pinca's property was completely burned • On February 5, 1982, Pinca's payment was returned by MICO to Adora on the

ground that her policy had been cancelled earlier. But Adora refused to accept it.

• In due time, Pinca made the requisite demands for payment, which MICO rejected. She then went to the Insurance Commission. It is because she was ultimately sustained by the public respondent that the petitioner has come to us for relief.

ISSUE: Whether there is an existing insurance policy at the time of the loss sustained by Pinca

HELD:

Yes. The Court does not agree in MICO's view that there was no existing insurance at the time of the loss sustained by Pinca because her policy never became effective for non-payment of premium. Payment was in fact made, rendering the policy operative as of June 22, 1981, and removing it from the provisions of Article 77. Thereafter, the policy could be cancelled on any of the supervening grounds enumerated in Article 64 (except "non-payment of premium") provided the cancellation was made in accordance therewith and with Article 65.

Section 64 reads as follows:

"SEC. 64.No policy of insurance other than life shall be cancelled by the insurer except upon prior notice thereof to the insured, and no notice of cancellation shall be effective unless it is based on the occurrence, after the effective date of the policy, of one or more of the following: "(a)non-payment of premium; "(b)conviction of a crime arising out of acts increasing the hazard insured against; "(c)discovery of fraud or material misrepresentation; "(d)discovery of willful or reckless acts or commissions increasing the hazard insured against; "(e)physical changes in the property insured which result in the property becoming uninsurable; or "(f)a determination by the Commissioner that the continuation of the policy would violate or would place the insurer in violation of this Code."

As for the method of cancellation, Section 65 provides as follows:

"SEC. 65.All notices of cancellation mentioned in the preceding section shall be in writing, mailed or delivered to the named insured at the address shown in the policy, and shall state (a) which of the grounds set forth in section sixty-four is relied upon and (b) that, upon written request of the named insured, the insurer will furnish the facts on which the cancellation is based."

A valid cancellation must, therefore, require concurrence of the following conditions:

(1)There must be prior notice of cancellation to the insured; (2)The notice must be based on the occurrence, after the effective date of the policy, of one or more of the grounds mentioned. (3)The notice must be (a) in writing, (b) mailed, or delivered to the named insured, (c) at the address shown in the policy; (4)It must state (a) which of the grounds mentioned in Section 64 is relied upon and (b) that upon written request of the insured, the insurer will furnish the facts on which the cancellation is based.

MICO claims it cancelled the policy in question on October 15, 1981, for non-payment of premium. On the other hand, there is the flat denial of Pinca, who says she never received the claimed cancellation and who, of course, did not have to prove such denial Considering the strict language of Section 64 that no insurance policy shall be cancelled except upon prior notice, it behooved MICO to make sure that the cancellation was actually sent to and received by the insured.

Page 62: Insurance Case Digests (1)

Insurance Case Digests Atty. Bathan-Basuel

2-S, 2013-2014 62

It stands to reason that if Pinca had really received the said notice, she would not have made payment on the original policy on December 24, 1981. Instead, she would have asked for a new insurance, effective on that date and until one year later, and so taken advantage of the extended period. The Court finds that if she did pay on that date, it was because she honestly believed that the policy issued on June 7, 1981, was still in effect and she was willing to make her payment retroact to July 22, 1981, its stipulated commencement date. As it has not been shown that there was a valid cancellation of the policy, there was consequently no need to renew it but to pay the premium thereon. Payment was thus legally made on the original transaction and it could be, and was, validly received on behalf of the insurer by its agent Adora. Adora, incidentally, had not been informed of the cancellation either and saw no reason not to accept the said payment.

WHEREFORE, the petition is DENIED.

7. Warranties

American Home Assurance Company v. Tantuco Enterprises, Inc. 366 SCRA 740 American Home Assurance Company v. Tantuco Enterprises, Inc. G.R. No. 138941. October 8, 2001 FACTS: Tantuco Enterprises (later to be referred to as Tantuco) is engaged in the coconut oil milling and refining industry. It owns two oil mills which are located at its factory compound at Iyam, Lucena City. The two oil mills were separately covered by fire insurance policies issued by American Home Assurance Co., Philippine Branch. The first oil mill was insured for three million pesos (P3,000,000.00), while the new oil mill was insured for six million pesos (P6,000,000.00). A fire that broke out in the early morning of September 30,1991 and consumed the new oil mill. Respondent immediately notified the petitioner of the incident. American Home rejected respondent’s claim for the insurance proceeds on the ground that no policy was issued by it covering the burned oil mill. It stated that the description of the insured establishment referred to another building. American Home also raised the issue that Tantuco violated the express terms of the Fire Extinguishing Appliances Warranty stated in the policy. A complaint for specific performance and damages was consequently instituted by the Tantuco with the RTC Lucena City. RTC Lucena – in favor of Tantuco CA – affirmed RTC's decision ISSUE: Whether there was a violation of the Fire Extinguishing Appliances Warranty stated in the policy resulting to the avoidance of the policy.

HELD: There was NO violation of the Fire Extinguishing Appliances Warranty in the policy. The warranty provides:

“WARRANTED that during the currency of this Policy, Fire Extinguishing Appliances as mentioned below shall be maintained in efficient working order on the premises to which insurance applies:

- PORTABLE EXTINGUISHERS - INTERNAL HYDRANTS - EXTERNAL HYDRANTS - FIRE PUMP - 24-HOUR SECURITY SERVICES

BREACH of this warranty shall render this policy null and void and the Company shall no longer be liable for any loss which may occur.”

Petitioner argues that the warranty clearly obligates the insured to maintain all the appliances specified therein. The breach occurred when the respondent failed to install internal fire hydrants inside the burned building as warranted. This fact was admitted by the oil mill’s expeller operator, Gerardo Zarsuela. The aforementioned warranty did not require respondent to provide for all the fire extinguishing appliances enumerated therein. Additionally, we find that neither did it require that the appliances are restricted to those mentioned in the warranty. In other words, what the warranty mandates is that respondent should maintain in efficient working condition within the premises of the insured property, fire fighting equipments such as, but not limited to, those identified in the list, which will serve as the oil mill’s first line of defense in case any part of it bursts into flame. To be sure, respondent was able to comply with the warranty. Within the vicinity of the new oil mill can be found the following devices: numerous portable fire extinguishers, two fire hoses, fire hydrant, and an emergency fire engine. All of these equipments were in efficient working order when the fire occurred. It ought to be remembered that not only are warranties strictly construed against the insurer, but they should, likewise, by themselves be reasonably interpreted. That reasonableness is to be ascertained in light of the factual conditions prevailing in each case. Here, we find that there is no more need for an internal hydrant considering that inside the burned building were: (1) numerous portable fire extinguishers, (2) an emergency fire engine, and (3) a fire hose which has a connection to one of the external hydrants.

Ang Giok Chip v. Sprinfiled Fire & Maritime Ins. Co., 56 Phil 375 FACTS:

Page 63: Insurance Case Digests (1)

Insurance Case Digests Atty. Bathan-Basuel

2-S, 2013-2014 63

Ang Giok Chip doing business under the name and style of Hua Bee Kong Si was formerly the owner of a warehouse situated at No. 643 Calle Reina Regente, City of Manila. The contents of the warehouse were insured with the three insurance companies for the total sum of P60,000. One insurance policy, in the amount of P10,000, was taken out with the Springfield Fire & Marine Insurance Company. The warehouse was destroyed by fire on January 11, 1928, while the policy issued by the latter company was in force.

Predicated on this policy the plaintiff instituted action in the Court of First Instance of Manila against the defendant to recover a proportional part of the loss coming to P8,170.59. Four special defenses were interposed on behalf of the insurance company, one being planted on a violation of warranty F fixing the amount of hazardous goods which might be stored in the insured building.

Ten thousand pesos Philippine Currency. — On general non-hazardous merchandise, chiefly consisting of chucherias, also produce, Cacao, Flour, all the property of the Insured, or held by them in trust, on commission or on joint account with others, or for which he is responsible, while contained during the currency of this policy in the godown, situate No. 643 Calle Reina Regent. . . .

This policy is subject to the hereon attached "Ordinary Short Period Rate Scale"

Warranties A & F, Co-insurances Clause "and Three Fourths Loss Clause," which are forming part of same. Co-insurance declared:

"P20,000. — Sun Insurance Office Ltd. (K & S)." (Emphasis inserted.) Securely pasted on the left hand margin of the face of the policy are five warranties and special clauses. One of them is warranty F, specially referred to on the face of the policy, reading in part as follows:

ISSUE: whether a warranty referred to in the policy as forming part of the contract of insurance and in the form of a rider to the insurance policy, is null and void because not complying with the Philippine Insurance Act.

HELD:

"Every express warranty, made at or before the execution of a policy, must be contained in the policy itself, or in another instrument signed by the insured and referred to in the policy, as making a part of it." As the Philippine law was taken verbatim from the law of California, in accordance with well settled canons of statutory construction, the court should follow in fundamental points, at least, the construction placed by California courts on a California law.

"Any express warranty or condition is always a part of the policy, but, like any other part of an express contract, may be written in the margin, or contained in proposals or documents expressly referred to in the policy, and so made a part of it."

We are given to understand, and there is no indication to the contrary, that we have here a standard insurance policy. We are further given to understand, and there is no indication to the contrary, that the issuance of the policy in this case with its attached rider conforms to well established practice in the Philippines and elsewhere. We are further given to understand, and there is no indication to the contrary, that there are no less than sixty- nine insurance companies doing business in the Philippine Islands with outstanding policies more or less similar to the one involved in this case, and that to nullify such policies would place an unnecessary hindrance in the transaction of insurance business in the Philippines. These are matters of public policy. We cannot believe that it was ever the legislative intention to insert in the Philippine Law on Insurance an oddity, an incongruity, entirely out of harmony with the law as found in other jurisdiction, and destructive of good business practice.

We have studied this case carefully and having done so have reached the definite conclusion that warranty F, a rider attached to the face of the insurance policy, and referred to in contract of insurance, is valid and sufficient under section 65 of the Insurance Act. Accordingly, sustaining the first and fourth errors assigned, and it being unnecessary to discuss the remaining errors, the result will be to reverse the judgment appealed from and to order the dismissal of the complaint, without special pronouncement as to costs in either instance.

Young v. Midland Textile Insurance Co., 30 Phil 617 FACTS: The purpose of the present action is to recover the sum of P3,000 upon an insurance policy. The lower court rendered a judgment in favor of the plaintiff. The undisputed facts upon which said action is based are as follows: 1. The plaintiff conducted a candy and fruit store on the Escolta and occupied a building as a residence and bodega (storehouse). 2 The defendant entered into a contract of insurance with the plaintiff. 3. On the conditions of said contract of insurance is found in "warranty B" and is as follows: "Waranty B. — It is hereby declared and agreed that during the pendency of this policy no hazardous goods stored or kept for sale, and no hazardous trade or process be carried on, in the building to which this insurance applies, or in any building connected therewith." 4. The plaintiff placed in said residence and bodega three boxes which belonged to him and which were filed with fireworks. 5. Said residence and bodega and the contents thereof were partially destroyed by fire. 6. Said fireworks had been given to the plaintiff by the former owner of the Luneta Candy Store; that the plaintiff intended to use the same in the celebration of the

Page 64: Insurance Case Digests (1)

Insurance Case Digests Atty. Bathan-Basuel

2-S, 2013-2014 64

Chinese new year; that the authorities of the city of Manila had prohibited the use of fireworks on said occasion, and that the plaintiff then placed the same in said bodega, where they remained from the 4th or 5th of February, 1913, until after the fire of the 18th of March, 1913. 7. Both of the parties agree that said fireworks come within the phrase "hazardous goods," mentioned in said "warranty B" of the policy. 8. That said fireworks were found in a part of the building not destroyed by the fire; that they in no way contributed to the fire, or to the loss occasioned thereby. ISSUE: Whether or not the placing of said fireworks in the building insured, under the conditions above enumerated, they being "hazardous goods," is a violation of the terms of the contract of insurance and especially of "warranty B." HELD: The plaintiff contends that under all the facts and circumstances of the case, they were not "stored" in said building, and that the placing of them in the building was not a violation of the terms of the contract. This leads us to a consideration of the meaning of the accord "stored" as used in said "warranty B. The author of the Century Dictionary defines the world "store" to be a deposit in a store or warehouse for preservation or safe keeping; o place in a warehouse or other place of deposit for safe keeping. Said definitions, of course, do not include a deposit in a store, in small quantities, for daily use. "Daily use" precludes the idea of a deposit for preservation or safe keeping, as well as a deposit for future consumption, or safe keeping. In the present case no claim is made that the "hazardous goods" were placed in the bodega for present or daily use. It is admitted that they were placed in the bodega "for future use," or for future consumption, or for safe keeping. It seems clear to us that the "hazardous goods" in question were "stored" in the bodega, as that word is generally defined. If the "warranty" is a term of the contract, will not its violation cause a breach and justify noncompliance or a repudiation? Contracts of insurance are contracts of indemnity upon the terms and conditions specified in the policy. The parties have a right to impose such reasonable conditions at the time of the making of the contract as they may deem wise and necessary. he terms of the policy constitute the measure of the insurer's liability, and in order to recover the insured must show himself within those terms; and if it appears that the contract has been terminated by a violation, on the part of the insured, of its conditions, then there can be no right of recovery. The compliance of the insured with the terms of the contract is a condition precedent to the right of recovery. If the insured has violated or failed to perform the conditions of the contract, and such a violation or want of performance has not been waived by the insurer, then the insured cannot recover. The conditions of contracts of insurance, when plainly expressed in a policy, are binding

upon the parties and should be enforced by the courts, if the evidence brings the case clearly within their meaning and intent. The appellant argues, however, that in view of the fact that the "storing" of the fireworks on the premises of the insured did not contribute in any way to the damage occasioned by the fire, he should be permitted to recover — that the "storing" of the "hazardous goods" in no way caused injury to the defendant company. The violation of the terms of the contract, by virtue of the provisions of the policy itself, terminated, at the election of either party, he contractual relations. Certainly it cannot be denied that the placing of the firecrackers in the building insured increased the risk. The plaintiff had not paid a premium based upon the increased risk, neither had the defendant issued a policy upon the theory of a different risk. The plaintiff was enjoying, if his contention may be allowed may be allowed, the benefits of an insurance policy upon one risk, whereas, as a matter of fact, it was issued upon an entirely different risk. The defendant had neither been paid nor had issues a policy to cover the increased risk. An increase of risk which is substantial and which is continued for a considerable period of time, is a direct and certain injury to the insurer, and changes the basis upon which the contract of insurance rests. Therefore and for the foregoing reasons, the judgment of the lower court is hereby revoked and the defendant is hereby relieved from any responsibility under said complaint, and, without any finding as to costs, it is so ordered.

Qua Chee Gan v. law Unoin & Rock Ins. Co. Ltd, G.R. No. L-4611, December 17, 1955

FACTS: Qua Chee Gan, a merchant of Albay, instituted this action in 1940, in the Court of First Instance of said province, seeking to recover the proceeds of certain fire insurance policies totalling P370,000, issued by the Law Union & Rock Insurance Co., Ltd., upon certain bodegas and merchandise of the insured that were burned on June 21, 1940. The records of the original case were destroyed during the liberation of the region, and were reconstituted in 1946. After a trial that lasted several years, the Court of First Instance rendered a decision in favor of the plaintiff. The record shows that before the last war, plaintiff-appellee owned four warehouses or bodegas (designated as Bodegas Nos. 1 to 4) in the municipality of Tabaco, Albay, used for the storage of stocks of copra and of hemp, baled and loose, in which the appellee dealth extensively. They had been, with their contents, insured with the defendant Company since 1937, and the lose made payable to the Philippine National Bank as mortgage of the hemp and crops, to the extent of its interest. TOTAL 370,000. BODEGA 1, 2, 3, 4 etc.

Fire of undetermined origin that broke out in the early morning of July 21, 1940, and lasted almost one week, gutted and completely destroyed Bodegas Nos. 1, 2 and 4, with the merchandise stored theren. Plaintiff-appellee informed the insurer by telegram on the same date;; and on the next day, the fire adjusters engaged by appellant insurance company arrived and proceeded to examine and photograph the premises, pored over the books of the insured and conducted an extensive investigation. The

Page 65: Insurance Case Digests (1)

Insurance Case Digests Atty. Bathan-Basuel

2-S, 2013-2014 65

plaintiff having submitted the corresponding fire claims, totalling P398,562.81 (but reduced to the full amount of the insurance, P370,000), the Insurance Company resisted payment, claiming violation of warranties and conditions, filing of fraudulent claims, and that the fire had been deliberately caused by the insured or by other persons in connivance with him.

With counsel for the insurance company acting as private prosecutor, Que Chee Gan, with his brother, Qua Chee Pao, and some employees of his, were indicted and tried in 1940 for the crime of arson, it being claimed that they had set fire to the destroyed warehouses to collect the insurance. They were, however, acquitted by the trial court in a final decision dated July 9, 1941 (Exhibit WW). Thereafter, the civil suit to collect the insurance money proceeded to its trial and termination in the Court below, with the result noted at the start of this opinion. The Philippine National Bank's complaint in intervention was dismissed because the appellee had managed to pay his indebtedness to the Bank during the pendecy of the suit, and despite the fire losses.

ISSUE: Whether the policy is avoided on the ground of breach of warranty, specifically appearing on a rider pasted on the face of the policies

HELD: Appellant is barred by waiver (or rather estoppel) to claim violation of the so-called fire hydrants warranty, for the reason that knowing fully all that the number of hydrants demanded therein never existed from the very beginning, the appellant neverthless issued the policies in question subject to such warranty, and received the corresponding premiums. It would be perilously close to conniving at fraud upon the insured to allow appellant to claims now as void ab initio the policies that it had issued to the plaintiff without warning of their fatal defect, of which it was informed, and after it had misled the defendant into believing that the policies were effective.

The insurance company was aware, even before the policies were issued, that in the premises insured there were only two fire hydrants installed by Qua Chee Gan and two others nearby, owned by the municipality of Tabaco, only two fire hydrants installed by Qua Chee Gan and two others nearby, owned by the municipality of TAbaco,

contrary to the requirements of the warranty in question. Such fact appears from positive testimony for the insured that appellant's agents inspected the premises;; and the simple denials of appellant's representative (Jamiczon) can not overcome that proof. That such inspection was made is moreover rendered probable by its being a prerequisite for the fixing of the discount on the premium to which the insured was entitled, since the discount depended on the number of hydrants, and the fire fighting equipment available (See "Scale of Allowances" to which the policies were expressly made subject). The law, supported by a long line of cases, is expressed by American Jurisprudence (Vol. 29, pp. 611-612) to be as follows:

It is usually held that where the insurer, at the time of the issuance of a policy of insurance, has knowledge of existing facts which, if insisted on, would invalidate the contract from its very inception, such knowledge constitutes a waiver of conditions in

the contract inconsistent with the facts, and the insurer is stopped thereafter from asserting the breach of such conditions. The law is charitable enough to assume, in the absence of any showing to the contrary, that an insurance company intends to executed a valid contract in return for the premium received;; and when the policy contains a condition which renders it voidable at its inception, and this result is known to the insurer, it will be presumed to have intended to waive the conditions and to execute a binding contract, rather than to have deceived the insured into thinking he is insured when in fact he is not, and to have taken his money without consideration.

To allow a company to accept one's money for a policy of insurance which it then knows to be void and of no effect, though it knows as it must, that the assured believes it to be valid and binding, is so contrary to the dictates of honesty and fair dealing, and so closely related to positive fraud, as to the abhorent to fairminded men.

We see no reason why the prohibition of keeping gasoline in the premises could not be expressed clearly and unmistakably, in the language and terms that the general public can readily understand, without resort to obscure esoteric expression

his rigid application of the rule on ambiguities has become necessary in view of current business practices. The courts cannot ignore that nowadays monopolies, cartels and concentrations of capital, endowed with overwhelming economic power, manage to impose upon parties dealing with them cunningly prepared "agreements" that the weaker party may not change one whit, his participation in the "agreement" being reduced to the alternative to take it or leave it" labelled since Raymond Baloilles" contracts by adherence" (con tracts d'adhesion), in contrast to these entered into by parties bargaining on an equal footing, such contracts (of which policies of insurance and international bills of lading are prime examples) obviously call for greater strictness and vigilance on the part of courts of justice with a view to protecting the weaker party from abuses and imposition, and prevent their becoming traps for the unwarry

RTC DECISION

Bachrach v. British American Assurance Co., 17 Phil 555 FACTS: Fire insurance policy issued by the defendant to the plaintiff. Properties destroyed by fire.

IN DEFENDANT’S ANSWER:

First. That the plaintiff maintained a paint and varnish shop in the said building where the goods which were insured were stored.

Second. That the plaintiff transferred his interest in and to the property covered by the policy to H. W. Peabody & Co. to secure certain indebtedness due and owing to said

Page 66: Insurance Case Digests (1)

Insurance Case Digests Atty. Bathan-Basuel

2-S, 2013-2014 66

company, and also that the plaintiff had transferred his interest in certain of the goods covered by the said policy to one Macke, to secure certain obligations assumed by the said Macke for and on behalf of the insured. That the sanction of the said defendant had not been obtained by the plaintiff, as required by the said policy.

Third. That the plaintiff, on the 18th of April, 1908, and immediately preceding the outbreak of the alleged fire, willfully placed a gasoline can containing 10 gallons of gasoline in the upper story of said building in close proximity to a portion of said goods, wares, and merchandise, which can was so placed by the plaintiff as to permit the gasoline to run on the floor of said second story, and after so placing said gasoline, he, the plaintiff, placed in close proximity to said escaping gasoline a lighted lamp containing alcohol, thereby greatly increasing the risk of fire.

Fourth. That the plaintiff made no proof of the loss within the time required by condition five of said policy, nor did the insured file a statement with he municipal or any other judge or court of the goods alleged to have been in said building at the time of the alleged fire, nor of the goods saved, nor the loss suffered.

LOWER COURT’S DECISION:

It is claimed that either gasoline or alcohol was kept in violation of the policy in the bodega containing the insured property. The testimony on this point is somewhat conflicting, but conceding all of the defendant's claims, the construction given to this claim by American courts would not justify the forfeiture of the policy on that ground. The property insured consisted mainly of household furniture kept for the purpose of sale. The preservation of the furniture in a salable condition by retouching or otherwise was incidental to the business. The evidence offered by the plaintiff is to the effect that alcohol was used in preparing varnish for the purpose of retouching, though he also says that the alcohol was kept in store and not in the bodega where the furniture was. It is well settled that the keeping of inflammable oils on the premises, though prohibited by the policy, does not void it if such keeping is incidental to the business. Thus, where a furniture factory keeps benzine for the purposes of operation (Davis vs. Pioneer Furniture Company, 78 N. W. Rep., 596;; Faust vs. American Fire Insurance Company, 91 Wis., 158), or where it is used for the cleaning machinery (Mears vs. Humboldt Insurance Company, 92 Pa. St., 15;; 37 Am. Rep., 647), the insurer can not on that ground avoid payment of loss, though the keeping of the benzine on the premises is expressly prohibited. These authorities also appear sufficient to answer the objection that the insured automobile contained gasoline and that the plaintiff on one occasion was seen in the bodega with a lighted lamp. The first was incidental to the use of the insured article and the second being a single instance falls within the doctrine of the case last cited.

ISSUE: WHETHER BACHRACH CAN CLAIM PROCEEDS

HELD: It may be added that there was no provision in the policy prohibiting the keeping of paints and varnishes upon the premises where the insured property was stored. If the

company intended to rely upon a condition of that character, it ought to have been plainly expressed in the policy.

As to the claim that the loss occurred through the voluntary act of the insured, we consider it unnecessary to review the evidence in detail. That was done by another branch of this court in disposing of the criminal prosecution brought against the insured, on the same ground, based mainly on the same evidence. And regardless of whether or not the judgment in that proceeding is res adjudicata as to anything here, we are at least of the opinion that the evidence to establish this defense should not be materially less convincing than that required in order to convict the insured of the crime of arson. (Turtell vs. Beamount, 25 Rev. Rep., 644.) In order to find that the defense of incendiarism was established here, we would be obliged, therefore, in effect to set aside the findings of the judge and assessors in the criminal cause, and this we would be loath to do even though the evidence now produced were much stronger than it is.

It is claimed that the execution of a chattel mortgage on the insured property violated what is known as the "alienation clause," which is now found in most policies, and which is expressed in the policies involved in cases 6496 and 6497 by a purchase imposing forfeiture if the interest in the property pass from the insured. (Cases 6496 and 6497, in which are involved other action against other insurance companies for the same loss as in the present action.)

This clause has been the subject of a vast number of judicial decisions (13 Am. & Eng. Encyc. of Law, 2d ed., pp. 239 et seq.), and it is held by the great weight of authority that the interest in property insured does not pass by the mere execution of a chattel mortgage and that while a chattel mortgage is a conditional sale, there is no alienation within the meaning of the insurance law until the mortgage acquires a right to take possession by default under the terms of the mortgage. No such right is claimed to have accrued in the case at bar, and the alienation clause is therefore inapplicable.

Geagonia V. CA 214 SCRA 152 (1995) FACTS: The petitioner is the owner of Norman's Mart located in the public market of San Francisco, Agusan del Sur. On 22 December 1989, he obtained from the private respondent fire insurance policy for P100,000. The period of the policy was from 22 December 1989 to 22 December 1990 and covered the following: "Stock-in-trade consisting principally of dry goods such as RTW's for men and women wear and other usual to assured's business. The petitioner declared in the policy under the subheading entitled CO-INSURANCE that Mercantile Insurance Co., Inc. was the co-insurer for P50,000.00. From 1989 to 1990, the petitioner had in his inventory stocks amounting to P392,130.50. The policy contained the following condition:

Page 67: Insurance Case Digests (1)

Insurance Case Digests Atty. Bathan-Basuel

2-S, 2013-2014 67

3. The insured shall give notice to the Company of any insurance or insurances already affected, or which may subsequently be effected, covering any of the property or properties consisting of stocks in trade, goods in process and/or inventories only hereby insured, and unless such notice be given and the particulars of such insurance or insurances be stated therein or endorsed in this policy pursuant to Section 50 of the Insurance Code, by or on behalf of the Company before the occurrence of any loss or damage, all benefits under this policy shall be deemed forfeited, provided however, that this condition shall not apply when the total insurance or insurances in force at the time of the loss or damage is not more than P200,000.00. On 27 May 1990, fire of accidental origin broke out at around 7:30 p.m. at the public market of San Francisco, Agusan del Sur. The petitioner's insured stock-in-trade were completely destroyed prompting him to file with the private respondent a claim under the policy. On 28 December 1990, the private respondent denied the claim because it found that at the time of the loss the petitioner's stocks-in-trade were likewise covered by fire insurance policies No. GA-28146 and No. GA-28144, for P100,000.00 each, issued by the Cebu Branch of the Philippines First Insurance Co., Inc. (hereinafter PFIC). 3 These policies indicate that the insured was "Messrs. Discount Mart (Mr. Armando Geagonia, Prop.)" with a mortgage clause reading:

MORTGAGE: Loss, if any shall be payable to Messrs. Cebu Tesing Textiles, Cebu City as their interest may appear subject to the terms of this policy. CO-INSURANCE DECLARED: P100,000. — Phils. First CEB/F 24758. 4

The basis of the private respondent's denial was the petitioner's alleged violation of Condition 3 of the policy. PETITIONER’S CONTENTION: at the time he obtained the private respondent's fire insurance policy he knew that the two policies issued by the PFIC were already in existence; however, he had no knowledge of the provision in the private respondent's policy requiring him to inform it of the prior policies; this requirement was not mentioned to him by the private respondent's agent; and had it been mentioned, he would not have withheld such information. RESPONDENT’S CONTENTION: denied the allegations in the complaint and set up as its principal defense the violation of Condition 3 of the policy. INSURANCE COMMISSION: petitioner did not violate Condition 3 as he had no knowledge of the existence of the two fire insurance policies obtained from the PFIC. CA: reversed the decision of I.C. ISSUES: (a) whether the petitioner had prior knowledge of the two insurance policies issued by the PFIC when he obtained the fire insurance policy from the private respondent, thereby, for not disclosing such fact, violating Condition 3 of the policy, and [YES] (b) if he had, whether he is precluded from recovering therefrom. [NO]

HELD: We agree with the Court of Appeals that the petitioner knew of the prior policies issued by the PFIC. His letter of 18 January 1991 to the private respondent conclusively proves this knowledge. His testimony to the contrary before the Insurance Commissioner and which the latter relied upon cannot prevail over a written admission made ante litem motam. It was, indeed, incredible that he did not know about the prior policies since these policies were not new or original. Policy No. GA-28144 was a renewal of Policy No. F-24758, while Policy No. GA-28146 had been renewed twice, the previous policy being F-24792. Condition 3 of the private respondent's Policy No. F-14622 is a condition which is not proscribed by law. Its incorporation in the policy is allowed by Section 75 of the Insurance Code which provides: "[a] policy may declare that a violation of specified provisions thereof shall avoid it, otherwise the breach of an immaterial provision does not avoid the policy." Such a condition is a provision which invariably appears in fire insurance policies and is intended to prevent an increase in the moral hazard. It is commonly known as the additional or "other insurance" clause and has been upheld as valid and as a warranty that no other insurance exists. Its violation would thus avoid the policy. However, in order to constitute a violation, the other insurance must be upon same subject matter, the same interest therein, and the same risk.” As to a mortgaged property, the mortgagor and the mortgagee have each an independent insurable interest therein and both interests may be one policy, or each may take out a separate policy covering his interest, either at the same or at separate times. A mortgagor may, however, take out insurance for the benefit of the mortgagee, which is the usual practice. The mortgagee may be made the beneficial payee in several ways. He may become the assignee of the policy with the consent of the insurer; or the mere pledgee without such consent; or the original policy may contain a mortgage clause; or a rider making the policy payable to the mortgagee "as his interest may appear" may be attached; or a "standard mortgage clause," containing a collateral independent contract between the mortgagee and insurer, may be attached; or the policy, though by its terms payable absolutely to the mortgagor, may have been procured by a mortgagor under a contract duty to insure for the mortgagee's benefit, in which case the mortgagee acquires an equitable lien upon the proceeds. It is a cardinal rule on insurance that a policy or insurance contract is to be interpreted liberally in favor of the insured and strictly against the company, the reason being, undoubtedly, to afford the greatest protection which the insured was endeavoring to secure when he applied for insurance. It is also a cardinal principle of law that forfeitures are not favored and that any construction which would result in the forfeiture of the policy benefits for the person claiming thereunder, will be avoided, if it is possible to construe the policy in a manner which would permit recovery, as, for example, by finding a waiver for such forfeiture.

Page 68: Insurance Case Digests (1)

Insurance Case Digests Atty. Bathan-Basuel

2-S, 2013-2014 68

With these principles in mind, we are of the opinion that Condition 3 of the subject policy is not totally free from ambiguity and must, perforce, be meticulously analyzed. Such analysis leads us to conclude that (a) the prohibition applies only to double insurance, and (b) the nullity of the policy shall only be to the extent exceeding P200,000.00 of the total policies obtained. The first conclusion is supported by the portion of the condition referring to other insurance "covering any of the property or properties consisting of stocks in trade, goods in process and/or inventories only hereby insured," and the portion regarding the insured's declaration on the subheading CO-INSURANCE that the co-insurer is Mercantile Insurance Co., Inc. in the sum of P50,000.00. A double insurance exists where the same person is insured by several insurers separately in respect of the same subject and interest. As earlier stated, the insurable interests of a mortgagor and a mortgagee on the mortgaged property are distinct and separate. Since the two policies of the PFIC do not cover the same interest as that covered by the policy of the private respondent, no double insurance exists. The non-disclosure then of the former policies was not fatal to the petitioner's right to recover on the private respondent's policy. When a property owner obtains insurance policies from two or more insurers in a total amount that exceeds the property's value, the insured may have an inducement to destroy the property for the purpose of collecting the insurance. The public as well as the insurer is interested in preventing a situation in which a fire would be profitable to the insured. PETITION GRANTED. CA DECISION SET ASIDE. 8. Premium

Arce v. Capital Insurance & Surety Co., Inc 117 SCRA 63

FACTS:The appellee owned a residential house in Tondo Manila which was insured with the appellant COMPANY since 1961. In November 1965, the COMPANY sent to the INSURED a Renewal Certificate to cover the period from December 5, 1965 to December 5,1966, and requested payment of the corresponding premium. Anticipating that the premium could not be paid on time, the INSURED, thru his wife asked for an extension which was on January 4, 1996. The request was granted by the COMPANY. After the lapse of the requested extension, INSURED still failed to pay the premium. Thereafter, the house of the INSURED was totally destroyed by fire. Upon INSURED's presentation of claim for indemnity, he was told that no indemnity was due because the premium was not paid. The INSURED sued the COMPANY for indemnity. The trial court held the COMPANY liable to indemnify the INSURED on the ground that since the COMPANY could have demanded payment of the premium, mutuality of obligation required that it should be liable on the policy. Hence, this appeal by the COMPANY on question of law. ISSUE: Whether or not payment of premiums necessary for effectivity of policy?

HELD : YES. The Supreme Court reversed the decision of the trial court.It held that Section 72 of the Insurance Act as amended by R.A.. 3540 states that "no policy issued by an insurance company is valid and binding unless and until the premium thereof has been paid." It is obvious from both the Insurance Act, as amended, and the stipulation of the parties that time is of the essence in respect to the payment of the insurance premium so that if it is not paid the contract does not take effect unless there is still another stipulation to the contrary. In the instant case, the INSURED was given a grace period to pay the premium but the period having expired with no payment made, he cannot insist that the COMPANY is nonetheless obligated to him.

Acme Shoe Rubber & Plastic Corporation v. CA 134 SCRA 155 Facts: Since 1946, ACME had been insuring yearly against fire its building, machines and general merchandise, located at Caloocan City, with respondent Domestic Insurance Company of the Philippines. On May 14, 1962, ACME continued to insure its properties with Domestic and was issued a policy in the amount of P200,000.00 for the period May 15, 1962 up to May 15, 1963. Domestic issued a renewal receipt to cover the period May 15, 1963 to May 15, 1964. On January 8, 1964, ACME paid P3,331.26 as premium.Domestic applied the payment as renewal premium for the period May 15, 1963 to May 15, 1964. A receipt was issued for the renewal premium of P3,331.26 for the period May 15, 1964 to May 15, 1965. Stamped on the receipt was the:

“"Note: Subject to 'Receipt of Payment Clause' and 'Credit Agreement' attached hereto and forming part hereof."

The clauses mentioned, which were attached as riders to Renewal Receipt No. 30127, respectively read as follows:

"RECEIPT OF PAYMENT CLAUSE "IT IS HEREBY DECLARED AND AGREED that notwithstanding anything to the contrary contained in the within policy, this insurance will be deemed valid and binding upon the Company only when the premium and documentary stamps therefor have actually been paid in full and duly acknowledged in an official receipt signed by an authorized official/representative of the Company" "CREDIT AGREEMENT "The premium corresponding to the first ninety days of the term of this policy or any renewal thereof is hereby considered paid for the purpose only of making this Policy valid and binding during said portion of the term. Thereafter, this Policy shall automatically become void and ineffective (without prejudice to the obligation of the Insured to pay the corresponding short period premium for the said 90 days) unless prior to the expiration of said period the Insured shall have actually paid to the Company the total premium and the documentary stamps stipulated in this Policy." (Exhibit 'E-2')”

ACME thru its president, also signed a promissory note, promising to pay Domestic within 90 days from the effective date of the policy (May 15, 1964) the premium and

Page 69: Insurance Case Digests (1)

Insurance Case Digests Atty. Bathan-Basuel

2-S, 2013-2014 69

the documentary stamps in the sum of P3,331.26. Otherwise, the policy will be automatically cancelled. ACME's properties were completely destroyed by fire on October 13, 1964. ACME filed its insurance claim but the INSURER disclaimed liability on the ground that as of the date of loss, the properties burned were not covered by insurance. Issue: WON there is no insurance contract since Domestic accepted a one-year premium on January 8, 1964 Held: Yes, there is no insurance contract. By the express terms of the Promissory Note signed by its President, ACME was fully aware that the policy would be automatically cancelled on August 13, 1964, the 90th day from March 14, 1964, if it did not pay the premium before the former date. There is also evidence to the effect that various reminders by the INSURER for payment remained unheeded (Exhibit "10"). Not having paid the 1964-1965 premium within the extension granted, and pursuant to R.A. No. 3540, the policy was automatically cancelled and there was no insurance coverage to speak of as of the date of the fire on October 13, 1964. The pertinent provision of Republic Act No. 3540, approved on June 20, 1963, and put into effect by the Office of the Insurance Commissioner beginning October 1, 1963 (Exhibit "11"), reads:

"Sec. 72.An insurer is entitled to payment of the premium as soon as the thing insured is exposed to the peril insured against, unless there is clear agreement to grant the insured credit extension of the premium due. No policy issued by an insurance company is valid and binding unless and until the premium thereof has been paid."

Since Republic Act No. 3540 was approved only on June 20, 1963 and was put into effect only beginning October 1, 1963, it could not retroactively affect the renewal of the insurance policy on May 15, 1963, or prior to the Act's effective date. ACME's premium payment of January 8, 1964, therefore, was properly applied to the 1963-1964 premium.

ACME's claim that the INSURER would unjustly enrich itself if it were to be allowed to apply the one-year premium it received to a past period when the policy was void and the INSURER had incurred no risk, is flawed for the reason already stated that Renewal Receipt No. 22989 for 1963-1964 had been issued on May 14, 1963 before R.A. No. 3540 was approved on June 20, 1963 and implemented on October 1, 1963.What became automatically cancelled by R.A. No. 3540 was the 1964-1965 policy for ACME's failure to pay the premium within the 90-day extension granted, and in accordance with the express terms of the Promissory Note that it had signed.

Valenzuela vs. CA 191 SCRA 1 FACTS:

1) Arturo P. Valenzuela as a General Agent of Philippine American General Insurance Company, Inc. (Philamgen) since 1965, was authorized to solicit and sell in behalf of Philamgen all kinds of non-life insurance, and in consideration of services rendered was entitled to receive the full agent's commission of 32.5% from Philamgen under the scheduled commission rates. From 1973 to 1975, Valenzuela solicited marine insurance from one of his clients, the Delta Motors, Inc. (Division of Electronics Airconditioning and Refrigeration) in the amount of P4.4M from which he was entitled to a commission of 32%. However, Valenzuela did not receive his full commission which amounted to P1.6M from the P4.4 M insurance coverage of the Delta Motors. During the period 1976 to 1978, premium payments amounting to P1,946,886.00 were paid directly to Philamgen and Valenzuela's commission to which he is entitled amounted to P632,737.00. 2) In 1977, Philamgen started to become interested in and expressed its intent to share in the commission due Valenzuela on a 50-50 basis. Valenzuela refused. 3) On February 8, 1978 Philamgen and its President, Bienvenido M. Aragon insisted on the sharing of the commission with Valenzuela. This was followed by another sharing proposal dated June 1, 1978. On June 16,1978, Valenzuela firmly reiterated his objection to the proposals. Because of the refusal of Valenzuela, Philamgen and its officers, namely: Bienvenido Aragon, Carlos Catolico and Robert E. Parnell took drastic action against Valenzuela. They: (a) reversed the commission due him by not crediting in his account the commission earned from the Delta Motors, Inc. insurance; (b) placed agency transactions on a cash and carry basis; (c) threatened the cancellation of policies issued by his agency; and (d) started to leak out news that Valenzuela has a substantial account with Philamgen. All of these acts resulted in the decline of his business as insurance agent. Then on December 27, 1978, Philamgen terminated the General Agency Agreement of Valenzuela. TRIAL COURT: Ruled in favor of Valenzuela – “Since defendants are not justified in the termination of Arturo P. Valenzuela as one of their General Agents, defendants shall be liable for the resulting damage and loss of business of Valenzuela.” CA: Ruled in favor of Philamgen, ordered Valenzuela to pay Philamgen the amount corresponding to the unpaid and uncollected premiums. ISSUE: Whether or not the Valenzuela is liable to Philamgen for the unpaid and uncollected premiums? HELD: NO, Valenzuela is not liable to Philamgen for the unpaid and uncollected premiums.

Ø Section 77 of the Insurance Code: [N]otwithstanding any agreement to the contrary, no policy or contract of insurance is valid and binding unless and until the premiums thereof have been paid except in the case of a life or industrial life policy whenever the grace period provision applies (P.D. 612, as amended otherwise known as the Insurance Code of 1974).

Ø Perforce, since admittedly the premiums have not been paid, the policies issued have lapsed. The insurance coverage did not go into effect or did not continue and the obligation of Philamgen as insurer ceased. Hence, for Philamgen which had no more liability under the lapsed and inexistent policies

Page 70: Insurance Case Digests (1)

Insurance Case Digests Atty. Bathan-Basuel

2-S, 2013-2014 70

to demand, much less sue Valenzuela for the unpaid premiums would be the height of injustice and unfair dealing. In this instance, with the lapsing of the policies through the nonpayment of premiums by the insured there were no more insurance contracts to speak of.

Ø In Phil ippine Phoenix Surety and Insurance, Inc. v. Woodworks, Inc.,

SC held that the non-payment of premium does not merely suspend but puts an end to an insurance contract since the time of the payment is peculiarly of the essence of the contract. And in Arce v. The Capital Insurance and Surety Co. Inc., SC reiterated the rule that unless premium is paid, an insurance contract does not take effect.

Makati Tuscany Condominium Corp. V. CA 215 SCRA 462 FACTS

Ø 1982: The American Home Assurance Co. issued in favor of Makati Tuscany Condominium Corp. an insurance policy on the latter’s building and premises. The premium was paid on instalments, all of which were accepted by the insurer.

Ø 1983: The insurer issued to the Makati Tuscany another policy which replaced and renewed the previous polcy. The premium was again paid on instalments, all of which were accepted by the insurer.

Ø 1984: The policy was again renewed and insurer issued to insured another policy. On this renewed policy, the insured made two instalment payments, both accepted by the insurer. Thereafter, the insured refused to pay the balance of the premium.

Ø The insurer thus filed an action to recover the unpaid balance. Ø Insured explained that it discontinued the payment because the policy did not

contain a credit clause in its favor and the receipts for the instalment payments covering the policy for 1984-1985 as well as the two previous policies stated the ff. reservations: (2) Acceptance of this payment shall not waive any of the company rights to deny liability on any claim under the policy arising before such payments or after the expiration of the credit clause of the policy; (3) Subject to no loss prior to premium payment. If there be any loss such is not covered.

Ø Insured further claimed that the policy was never binding and no risk attached to the policy. It pleaded for a counterclaim for the premiums already paid from 1982 to 1985.

Ø Trial court dismissed the complaint and the counterclaim. Both appeals. Ø CA ordered the insured to pay the balance of the premiums. It explained that

the obligation to pay when due is ordinarily an indivisible obligation to pay the entire premium. Here, the parties agreed to make the premiums payable in INSTALLMENTS. It said that while it may be true under Section 77 that the parties may not agree to make the insurance contract valid and binding w/o payment of premiums, THERE IS NOTHING IN SAID SECTION WHICH SUGGESTS

THAT THE PARTIES MAY NOT AGREE TO ALLOW PAYMENT OF THE PREMIUMS IN INSTALLMENTS, OR TO CONSIDER THE CONTRACT AS VALID AND BINDING UPON PAYMENT OF THE FIRST PREMIUM. Otherwise, we would allow the insurer to renege on its liability under the contract had a loss occurred before the completion of payment of the entire premium despite the insurer’s voluntary acceptance of partial payments. THE INSURANCE CONTRACT BECAME VALID AND BINDING UPON PAYMENT OF THE FIRST PREMIUM AND THE INSURER COULD NOT HAVE DENIED LIABILITY ON THE GROUND THAT PAYMENT WAS NOT MADE IN FULL FOR THE REASON THAT IT AGREED TO ACCEPT INSTALLMENT PAYMENTS.

Ø The insured now asserts that its payment by instalment of the premiums for the insurance policies for 1982, 1092 and 1094 invalidates said policies because of Sec. 77 and by the conditions stipulated by the insurer in the receipts, disclaiming liability for loss occurring before payment of premiums. The insured concludes that there could not be a perfected contract of insurance upon mere partial payment of the premiums.

ISSUE Whether payment by instalment of the premiums due on an insurance policy invalidates the contract of insurance in view of Section 77 of PD 612 RULING The policies are valid even if the premiums were paid on instalments

(1) Parties intended subject insurance polices to be binding and effective notwithstanding the staggered payment of the premiums. Basic principles of equity and fairness would not allow the insurer to continue collecting and accepting the premiums although paid on instalments and later deny liability on the lame excuse that the premiums were not prepaid in full.

(2) While the import of Sec. 77 is that prepayment of premiums is strictly required as a condition to the validity of the contract, we are not prepared to rule that the request to make instalment payments duly approved by insurer would prevent the entire contract from going into effect despite payment and acceptance of the first instalment/ initial premium. SECTION 78 allows waiver by the insurer of the condition of prepayment by making an acknowledgment in the insurance policy of receipt of premium as conclusive evidence of payment so far as to make the policy binding despite the fact that premium is actually unpaid.

(3) Insured not entitled to a refund of the premiums. Parties intended to make the three insurance contracts valid. Hence, petitioner may not be allowed to renege on its obligation to pay the balance after the expiration of the term of the third policy. Where the risk is entire and the contract is indivisible, the insured is not entitled to a refund of the premiums paid if the insurer was exposed to the risk insured for any period, however brief or momentary.

UCPB General Insurance Co., INc. v. Masagana Telmart Inc., G.R. No. 137172 April

4, 2001

Page 71: Insurance Case Digests (1)

Insurance Case Digests Atty. Bathan-Basuel

2-S, 2013-2014 71

FACTS: Masagana Telamart, Inc. (Telamart) obtained from UCPB Gen. Insurance Co., Inc. (UCPB) 5 insurance policies on its properties in Pasay City and Manila which all reflect the effectivity term: “from 4PM of May 22, 1991 to 4PM of May 22, 1992”. On June 13, 1992, Telamart’s properties located in Taft Avenue, Pasay City were razed by fire. On July 13, 1992, Telamart tendered and UCPB accepted 5 Equitable Bank Manager’s Checks in the total amount of P225,753.45 as renewal premium payments for which Official Receipt Direct Premium No. 62926 was issued by UCPB. On July 14, 1992, Telamart made its formal demand for indemnification for the burned insured properties. UCPB returned the 5 manager’s checks stating in its letter that it was rejecting Telamart’s claim on the ff. grounds: a. The policies expired last May 22, 1992 and were not renewed for another term; b. UCPB had put Telamart and its alleged broker on notice of non-renewal earlier; and c. The properties covered by the said policies were burned in a fire that took place

last June 13, 1992, or before tender of premium payment. Hence, Telamart filed this case. The trial court (a) allowed Telamart to consign P225,753.95 as full payment of the premiums for the renewal of the 5 insurance policies on Telamart’s properties; (b) declared the replacement renewal policies effective and binding from May 22, 1992 until May 22, 1993; and (c) ordered UCPB to pay Telamart P18,645,000 as indemnity for the burned properties covered by the renewal-replacement policies. CA reversed and set aside the decision (1) deleting the declaration that 3 of the policies were in force from August 1991 to August 1992; and (2) reducing the award of the attorney’s fees from 25% to 10% of the total amount due to Telamart; because the tender of payment of the premiums on July 13, 1992 did not result in the renewal of the policies, having been made beyond the effective date of renewal as provided under Policy Condition No. 26, which states: 26. Renewal Clause. -- Unless the company at least forty five days in advance of the end of the policy period mails or delivers to the assured at the address shown in the policy notice of its intention not to renew the policy or to condition its renewal upon reduction of limits or elimination of coverages, the assured shall be entitled to renew the policy upon payment of the premium due on the effective date of renewal. Both the trial court and the CA found that sufficient proof exists that Telamart which had procured insurance coverage from UCPB for a number of years had been granted a 60- to 90-day credit for the renewal of the policies. Such a practice had existed up to the time the claims were filed. Also, the CA ruled that that no timely notice of non-renewal was made by UCPB.

On June 15, 1999, the SC ruled that the fire insurance policies issued by UCPB to Telamart covering the period from May 22, 1991 to May 22, 1992 had NOT been extended nor renewed by an implied credit arrangement though actual payment of premium was tendered on a later date and after the occurrence of the (fire) risk insured against, pursuant to Sec. 77 of the Insurance Code. Telemart filed an MR. No notice of non-renewal was made within 45 days before 22 May 1992, or before the expiration date of the fire insurance policies. Thus, the policies in question were renewed by operation of law and were effective and valid on 30 June 1992 when the fire occurred, since the premiums were paid within the 60- to 90-day credit term. The trial court and the CA established the ff. facts: 1. For years, UCPB had been issuing fire policies to Telamart, and these policies were

annually renewed. 2. UCPB had been granting Telamart a 60- to 90-day credit term within which to pay

the premiums on the renewed policies. 3. There was no valid notice of non-renewal of the policies in question, as there is no

proof at all that the notice sent by ordinary mail was received by Telamart and the copy thereof allegedly sent to Zuellig was ever transmitted to Telamart.

4. The premiums for the policies in question in the aggregate amount of P225,753.95 were paid by Telamart within the 60- to 90-day credit term and were duly accepted and received by UCPB’s cashier.

ISSUE: Whether Sec. 77 of the Insurance Code of 1978 (PD 1460) must be strictly applied to UCPB’s advantage despite its practice of granting a 60- to 90-day credit term for the payment of premiums. HELD: NO. Section 77 of the Insurance Code of 1978 provides: SEC. 77. An insurer is entitled to payment of the premium as soon as the thing insured is exposed to the peril insured against. Notwithstanding any agreement to the contrary, no policy or contract of insurance issued by an insurance company is valid and binding unless and until the premium thereof has been paid, except in the case of a life or an industrial life policy whenever the grace period provision applies. There are, however, exceptions to Sec. 77: 1. In case of a life or industrial life policy whenever the grace period provision applies

(Sec. 77) 2. Any acknowledgement in a policy or contract of insurance of the receipt of

premium is conclusive evidence of its payment so far as to make the policy binding, notwithstanding any stipulation therein that it shall not be binding until premium is actually paid. (Sec. 78)

3. If the parties agreed to the payment in installments of the premium and partial payment has been made at the time of loss.

4. The insurer may grant credit extension for the payment of the premium. 5. Estoppel.

Page 72: Insurance Case Digests (1)

Insurance Case Digests Atty. Bathan-Basuel

2-S, 2013-2014 72

While the import of Section 77 is that prepayment of premiums is strictly required as a condition to the validity of the contract, the request to make installment payments duly approved by the insurer would NOT prevent the entire contract of insurance from going into effect despite payment and acceptance of the initial premium or first installment. Section 78 of the Insurance Code in effect allows waiver by the insurer of the condition of prepayment by making an acknowledgment in the insurance policy of receipt of premium as conclusive evidence of payment so far as to make the policy binding despite the fact that premium is actually unpaid. Section 77 merely precludes the parties from stipulating that the policy is valid even if premiums are not paid, but does not expressly prohibit an agreement granting credit extension, and such an agreement is not contrary to morals, good customs, public order or public policy. So is an understanding to allow insured to pay premiums in installments not so prescribed. At the very least, both parties should be deemed in estoppel to question the arrangement they have voluntarily accepted. Hence, if the insurer has granted the insured a credit term for the payment of the premium and loss occurs before the expiration of the term, recovery on the policy should be allowed even though the premium is paid after the loss but within the credit term. There is nothing in Section 77 which prohibits the parties in an insurance contract to provide a credit term within which to pay the premiums. That agreement is not against the law, morals, good customs, public order or public policy. The agreement binds the parties. Article 1306 of the Civil Code provides: ART. 1306. The contracting parties may establish such stipulations clauses, terms and conditions as they may deem convenient, provided they are not contrary to law, morals, good customs, public order, or public policy. It would be unjust and inequitable if recovery on the policy would not be permitted against UCPB, which had consistently granted a 60- to 90-day credit term for the payment of premiums despite its full awareness of Section 77. Estoppel bars it from taking refuge under said Section, since Respondent relied in good faith on such practice. WHEREFORE, the Decision in this case of 15 June 1999 is RECONSIDERED and SET ASIDE, and a new one is hereby entered DENYING the instant petition for failure of Petitioner to sufficiently show that a reversible error was committed by the Court of Appeals in its challenged decision, which is hereby AFFIRMED in toto.

Phil. Pryce Assurance Corp. v. CA 230 SCRA 164 FACTS: Respondent Gegroco, Inc. filed a complaint for collection of sum of money against petitioner, Interworld Assurance Corporation (the company now carries the corporate

name Philippine Pryce Assurance Corporation). The complaint alleged that Pryce Corp. issued two surety bonds in behalf of its principal Sagum General Merchandise for (P500,000.00) PESOS and (1,000,000.00) PESOS, respectively. Pryce admitted having executed the said bonds, but denied liability because allegedly 1) the checks which were to pay for the premiums bounced and were dishonored hence there is no contract to speak of between petitioner and its supposed principal; and 2) that the bonds were merely to guarantee payment of its principal's obligation, thus, excussion is necessary. Petitioner Pryce failed to attend the pre-trial on several occasions and was therefor held in default and respondent was allowed to present evidence ex-parte. Both RTC and CA ruled in favor of respondent Gegroco Inc. ISSUE: Whether Pryce should be held liable for the surety bond which it issued HELD: Yes. The Insurance Code states that: "SECTION 177.The surety is entitled to payment of the premium as soon as the contract of suretyship or bond is perfected and delivered to the obligor. No contract of suretyship or bonding shall be valid and binding unless and until the premium therefor has been paid, except where the obligee has accepted the bond, in which case the bond becomes valid and enforceable irrespective of whether or not the premium has been paid by the obligor to the surety. . . ." (emphasis added) The above provision outrightly negates petitioner's first defense. In a desperate attempt to escape liability, petitioner further asserts that the above provision is not applicable because the respondent allegedly had not accepted the surety bond, hence could not have delivered the goods to Sagum Enterprises. The above provision outrightly negates petitioner's first defense. In a desperate attempt to escape liability, petitioner further asserts that the above provision is not applicable because the respondent allegedly had not accepted the surety bond, hence could not have delivered the goods to Sagum Enterprises. This statement clearly intends to muddle the facts as found by the trial court and which are on record. cdrep In the first place, petitioner, in its answer, admitted to have issued the bonds subject matter of the original action. 19 Secondly, the testimony of Mr. Leonardo T. Guzman, witness for the respondent, reveals the following: "Q.What are the conditions and terms of sales you extended to Sagum General Merchandise? A.First, we required him to submit to us Surety Bond to guaranty payment of the spare parts to be purchased. Then we sell to them on 90 days credit. Also, we required them to issue post-dated checks.

Page 73: Insurance Case Digests (1)

Insurance Case Digests Atty. Bathan-Basuel

2-S, 2013-2014 73

Q.Did Sagum General Merchandise comply with your surety bond requirement? A.Yes. They submitted to us and which we have accepted two surety bonds. On the other hand, petitioner's defense that it did not have authority to issue a Surety Bond when it did is an admission of fraud committed against respondent. No person can claim benefit from the wrong he himself committed. A representation made is rendered conclusive upon the person making it and cannot be denied or disproved as against the person relying thereon.

Capital Ins. & Surety Cor., Inc v. Plastic Era Co., Inc. 65 SCRA 134 FACTS

Ø Capital Insurance & Surety Inc. delivered to Plastic Era Manufacturing Co. Inc. an open fire policy wherein the former undertook to insure the latter’s building, equipment, raw materials, products, and accessories located at Sheridan Street, Mandaluyong, Rizal.

Ø The policy expressly provides that if the property insured would be destroyed or damaged by fire after the payment of the premiums, the insurance company shall make good all such loss/damage in an amount not exceeding 100K.

Ø When the policy was delivered, Plastic Era failed to pay the corresponding insurance premium.

Ø HOWEVER, its duly authorized representative executed the an acknowledgment receipt.

Ø Subsequently, in partial payment of the premium, Plastic Era delivered to Capital Insurance a check postdated January 16, 1961 (date when premium is due) payable to the order of the latter. However, Capital tried to deposit the check only on Feb. 20, 1961 and the same was dishonoured for lack of funds.

Ø On January 18, 1961, 2 days after the insurance premium became due, the property insured was destroyed by fire. Plastic Era notified the insurer of the loss and filed its claim for indemnity. The loss was estimated to be P283,875. However, the records of the same property show that it has been insured by Plastic Era with the PhilamGen Insurance Company for 200K.

Ø In less than a month, Plastic era demanded from Capital the payment of 100K as indemnity for the loss but the latter refused because Plastic failed to pay the insurance premium.

Ø Plastic filed its complaint against Capital for the recovery of 100K. Trial court ruled in favor of Plastic. Capital appealed to CA which affirmed the trial court decision. Hence, this petition.

ISSUE Whether or not a contract of insurance has been duly perfected. The insurance policy provides it is only upon payment of the premiums by Plastic that Capital agrees to insure the properties of the former. The crux of the problem then is

whether at the time the insurance policy was delivered to Plastic, the latter was able to pay the premium. It appears on record that Plastic did not, but instead executed an acknowledgment receipt of the policy. In said receipt, Plastic promised to pay the premium w/in 30 days from the effectivity date of the policy and Capital Insurance accepted it. What then is the effect of accepting such acknowledgment receipt? Did capital insurance mean to agree to make good its undertaking under the policy if the premium could not be paid on or before Jan 16? What would be the effect of the delivery to Capital of the postdated check? Could not this have been considered a valid payment? #damingtanong RULING

(1) Under Article 1249, the mere delivery of a BOE in payment of a debt does not immediately effect payment. Tender of draft or check to effect payment that would extinguish the liability should actually be cashed. If the delivery of the check were to be viewed in light of the foregoing, NO PAYMENT OF THE PREMIUM HAD BEEN EFFECTED, for it is only when the check is cashed that it is said to effect payment.

(2) By accepting the promise of Plastic to pay the premium w/in 30 days from the effective date of the policy, Capital implicitly agreed to MODIFY THE TENOR OF THE POLICY and in effect WAIVED THE PROVISION THEREIN THAT IT WOULD ONLY PAY FOR THE LOSS OR DAMAGE IN CASE THE SAME OCCURS AFTER THE PAYMENT OF THE PREMIUM. Considering that the policy is silent as to the mode of payment, Capital is deemed to have accepted the promissory note in payment of the premium. This rendered the policy IMMEDIATELY OPERATIVE ON THE DATE IT WAS DELIVERED. In US cases: Although one of the conditions of an insurance policy is that it shall not be valid until the payment of the first premium, if the policy is silent as to the mode of payment, promissory notes received by the company must be deemed to have been accepted in payment of the premium. In other words, a requirement for the payment of the 1st or initial premium in advance or actual cash may be waived by acceptance of a promissory note. This is what happened in this case when Capital accepted the receipt promising to pay w/in 30 days from Dec. 17. Hence, when the damage or loss of the insured property occurred, the insurance policy was in full force and effect. THE FACT THAT THE CHECK WAS LATER ON DISHONORED DID NOT OPERATE AS A FOREFEITURE OF ITS RIGHTS UNDER THE POLICY, there being no express stipulation therein to that effect. If the check is accepted as payment of the premium even though it turns out be worthless, there is payment which will prevent forfeiture.

(3) By accepting the promise of Plastic, Capital had in effect extended credit to Plastic. The payment of the premium therefore became an independent obligation the non fulfilment of which would entitle Capital to recover. It could just deduct the premium due and unpaid upon the satisfaction of the loss

Page 74: Insurance Case Digests (1)

Insurance Case Digests Atty. Bathan-Basuel

2-S, 2013-2014 74

under the policy. It did not have the right to cancel the policy for non-payment except by putting Plastic Era in default and giving it personal notice to that effect. This Capital failed to do. Where credit is given by the insurer for the payment of the premium, it has no right to cancel the policy for non-payment except by putting the insured in default and giving him personal notice. On the other hand, Capital had accepted a check from Plastic in partial payment of the premium. Having held the check for 35 days before presenting it for payment (an unreasonable period of time), Capital was stopped from claiming a forfeiture even if the check had been dishonoured later. Where the check is held for an unreasonable time before presenting it for payment, the insurer may be held estopped from claiming a forfeiture if the check is dishonoured.